You are on page 1of 155

Digitally signed by

kamel hassan
DN: cn=kamel hassan,

kamel hassan c=AD,


o=msd,
ou=1,
Reason: I attest to the accuracy and integrity of this document

MCQs PEDIATRICS

MCQs
PEDIATRICS

Prepared by
Dr.Kamel Hassan MD
C.E.S de Pediatrie, C.P.B in Pediatrics
Palestine – Gaza
E-mail: kyh10557@yahoo.com

Dr.Kamel Youssef Hassan, Pediatrician Consultant , Palestine - Gaza


E-mail: kyh10557@yahoo.com 1
MCQs PEDIATRICS

1- DEVELOPMENT
1-Theme : STATURE
A. Achondroplasia
B. Anorexia nervosa
C. Constitutional delay
D. Cornelia-de-Lange syndrome
E. Familial short stature
F. Growth hormone deficiency
G. Primordial dwarfism
H. Psychosocial poor growth
I. Prader-Willi syndrome
J. Russell-silver dwarfism
Select the most appropriate diagnosis from the above options to explain the
following presentations:
1) A child has short stature, a triangular face and facial and limb length
asymmetry.
J. Russell-silver dwarfism
Note:
describes a child with Russell –silver syndrome. A condition of short stature,
hemi-hypertrophy and the child often has triangular facies, frontal bossing and
clinodactyly of the 5th finger.

2) A child has short stature and is noted to have a large head with a prominent
forehead.
A. Achondroplasia
Note:
describes a child with achondroplasia. Short stature, rhizomelia (short limbs
especially the proximal part). Affected individuals usually have a large head with
hydrocephalus being a complication.

3) A baby is born on the 50th centile for height and weight. By the age of 2 her
length has fallen to the 2nd centile and weight remains on the 50th. She has a
rounded face with small hands and feet.
F. Growth hormone deficiency
Note:
describes a growth hormone deficient child. Early recognition and initiation of
treatment is required to reach satisfactory adult height.

Dr.Kamel Youssef Hassan, Pediatrician Consultant , Palestine - Gaza


E-mail: kyh10557@yahoo.com 2
MCQs PEDIATRICS

2-Theme : STATURE
A. Achondroplasia
B. Anorexia nervosa
C. Constitutional delay
D. Cornelia-de-Lange syndrome
E. Familial short stature
F. Growth hormone deficiency
G. Primordial dwarfism
H. Psychosocial poor growth
I. Prader-Willi syndrome
J. Russell-silver dwarfism
Select the most appropriate diagnosis from the above options to explain the
following presentations:
1) A 9 year old boy statemented for special educational needs is investigated for
short stature. He has a weight over the 91st centile. He is noted to have small
hands and feet and hypogonadism.
I. Prader-Willi syndrome
Note:
describes a child with Prader-Willi syndrome. A condition characterised by obesity after the
first year of life. Hypotonia, hypogonadism and learning difficulties due to a deletion of
chromosome 15 (the deletion occurring in the paternal gene, if the deletion of chromosome 15
in the child was inherited from the mother the child would have the phenotype of Angelman's
syndrome).
2) A 15 year old girl in foster care is referred to outpatients because of delayed
puberty. On examination her weight is on the 2nd centile and her height is on the
25th centile. She has poor dentition and her temperature is measured at 35°C.
B. Anorexia nervosa
Note:
describes an adolescent girl with anorexia nervosa. A condition due to fear of becoming
obese or disturbed body image. There is usually a tendency to deny hunger, excessive
dieting, laxative abuse or excessive physical activity. The diagnosis is classified as weight of
15% below that expected for age and height and is associated with physical characteristics
including amenorrhoea, bradycardia, hypothermia and electrolyte disturbances.
3) A 7 year old girl presents with short stature with weight being on the 9th
centile and height less than 0.4th centile. Examination is normal as well as all
investigations including a bone age, which is compatible with her chronological
age. Her mother is 5”1 (134cm), her father is 5”5 (143cm).
E. Familial short stature
Note:
describes a child with familial short stature. From the measurements of the
parents the child's height is compatible with this diagnosis. It is expected that
most children will reach a height within 8cms of the mid-parental height centile.

Dr.Kamel Youssef Hassan, Pediatrician Consultant , Palestine - Gaza


E-mail: kyh10557@yahoo.com 3
MCQs PEDIATRICS

2- CARDIOLOGY
1-Theme : EMERGENCY MEDICINE
A. Adenosine
B. Adrenaline
C. Atropine
D. DC shock
E. Dobutamine
F. Diving reflex
G. Endotracheal intubation
H. Intraosseous line
I. IV Morphine
J. Naloxone
Select the most appropriate emergency treatment for the following children:
1) A 4-year-old child is rescued from a house fire. She is admitted tachypnoeic
and tacchycardic. She has soot in her nostrils.
G. Endotracheal intubation
Note:
Item 1 relates to burns. Most deaths following house fires occur secondary to
smoke inhalation. Amongst the indicators of inhaled smoke injury is deposits
around the mouth and nose. Oedema follows thermal injury and therefore any
suspicion of airway compromise should result in endotracheal intubation.

2) A 3 month old baby is admitted with a history of poor feeding. On arrival he


has a pulse rate of 220 beats per minute.
F. Diving reflex
Note:
Supraventricular tachycardia is the diagnosis in Item 3. Vagal stimulation is the
treatment of choice and the diving reflex is the simple procedure elicited by
submerging the baby's face in to ice or placing an ice bag over the face. The
diving reflex increases vagal tone, slows AV conduction interrupting the
tachycardia.

3) A 13 year old boy is admitted with meningococcaemia. He is in shock.


Peripheral cannulation is difficult.
H. Intraosseous line
Note:
Item 2 relates to a child in shock. In many life-threatening conditions venous
cannulation is difficult. It is important to obtain vascular access very quickly and
therefore intraosseous infusion is recommended.

Dr.Kamel Youssef Hassan, Pediatrician Consultant , Palestine - Gaza


E-mail: kyh10557@yahoo.com 4
MCQs PEDIATRICS

2-Theme : SYNDROMES ASSOCIATED WITH CONGENITAL HEART


DISEASE
A. Charcot-Marie- Tooth syndrome
B. Down syndrome
C. Fragile X syndrome
D. Marfan's syndrome
E. Noonan syndrome
F. Sturge-Weber syndrome
G. Tetrology of Fallot
H. Tourette syndrome
I. Turner's syndrome
J. William's syndrome
For each of the cardiac lesions described below, choose the most commonly
associated syndrome from the above list of options. Each option may be used
once or not at all.
1) Supravalvular aortic stenosis
J. William's syndrome
Note:
William's syndrome  is characterized by short stature, characteristic facies,
supravalvular aortic stenosis, mild to moderate learning difficulties and transient
neonatal hypercalcemia.

2) Coarctation of Aorta
I. Turner's syndrome
Note:
Turner's syndrome  This is characterized by 45, X genotype, ovarian
dysgenesis leading to infertility, short stature, webbing of the neck, wide carrying
angles and wide spaced nipples. However they have normal intellectual
development.

3) Dilatation of aortic root/aortic regurgitation


D. Marfan's syndrome
Note:
Marfan's syndrome  This is an autosomal dominant disorder. The clinical
features are tall stature, arachno-dactyly, high arched plate and increase in
length of the lower segment of the body compared to the upper segment. The
cardiac manifestations include dilated aortic root, aortic incompetence, mitral
valve prolapse and mitral incompetence.

Dr.Kamel Youssef Hassan, Pediatrician Consultant , Palestine - Gaza


E-mail: kyh10557@yahoo.com 5
MCQs PEDIATRICS

4) Cardiac cushion defects (leading to ASD, VSD)


B. Down syndrome
Note:
Down syndrome  Features include characteristic facies, hypotonia, severe
learning difficulties and small stature. About 40% of patients have cardiac
anomalies mainly endocardial cushion defects leading to ASD and VSD.

5) Infundibular pulmonary stenosis


G. Tetrology of Fallot
Note:
Tetrology of Fallot This is a cyanotic heart disease and the cardinal features
include:
1. Infundibular pulmonary stenosis
2. VSD
3. Right ventricular hypertrophy
Over-riding of the aorta

Comments:
Charcot-Marie-Tooth syndrome Autosomal dominant peroneal muscular
dystrophy.

Fragile X syndrome Moderate learning difficulty, macrocephaly, characteristic


facies (long face, large ears, prominent mandible and forehead)
Noonan syndrome Facies, mild learning difficulties, short webbed neck, short
stature and congenital heart disease (pulmonary valvular stenosis, ASD, left
ventricular hypertrophy)

Sturge-Weber syndrome Haemangiomas in the distribution of trigeminal nerve


and in the brain. Sometimes can have intractable epilepsy.

Tourette syndrome Tics, compulsive utterances of obscene words (coprolalia)

Dr.Kamel Youssef Hassan, Pediatrician Consultant , Palestine - Gaza


E-mail: kyh10557@yahoo.com 6
MCQs PEDIATRICS

3-Theme : EMERGENCY TREATMENTS


A. Adenosine
B. Adrenaline
C. Atropine
D. DC shock
E. Dobutamine
F. Diving reflex
G. Endotracheal intubation
H. Intraosseous line
I. IV Morphine
J. Naloxone
Select the most appropriate emergency treatment for the following children:
1) A 5 year old boy is brought to the hospital with 15% scalds to his chest.
I. IV Morphine
Note:
Item 1 relates to a child with serious burns. Children who have been burnt are in
severe pain and therefore IV Morphine is the analgesic of choice.

2) A 14-year-old girl with a history of previous overdoses is admitted to A&E


apnoeic and unconsciousness. ECG shows ventricular fibrillation. CPR is
commenced.
D. DC shock
Note:
Item 2 describes a child in ventricular fibrillation. This is uncommon in childhood
although may occur as a result of tricyclic antidepressant overdose and
hypothermia. If the arrest is witnessed a precordial thump is carried out
otherwise electrical de-fibrillation at 2 joules per kilogram.

3) A term baby is born in poor condition. Apgar scores 3 at 1 minute and 5 at 5


minutes. CPR is commenced. At 10 minutes he remains bradycardic.
B. Adrenaline
Note:
Item 3 describes a baby born in poor condition. A bradycardia in an unstable
newborn requires oxygenation, ventilation and cardiac compressions. IV
adrenaline is administered as Atropine is ineffective in this age group.

Dr.Kamel Youssef Hassan, Pediatrician Consultant , Palestine - Gaza


E-mail: kyh10557@yahoo.com 7
MCQs PEDIATRICS

4-Theme : GENETICS - CARDIAC ABNORMALITIES IN GENETIC


DISORDERS
A. Angelman's syndrome
B. Beckwith-Wiedemann syndrome
C. Congenital Rubella syndrome
D. Down syndrome
E. Foetal alcohol syndrome
F. Glycogen storage disease
G. Marfan's syndrome
H. Noonan's syndrome
I. Turner's syndrome
J. Williams syndrome

Match each of the following cardiovascular abnormalities to the single most


likely associated genetic disorder.
1) Dilation of the aorta with aneurysms.
G. Marfan's syndrome
Note:
In Marfan's syndrome dilatation of the ascending aorta is often seen with or
without aneurysms. Less commonly the thoracic abdominal aorta or pulmonary
arteries are affected with secondary aortic regurgitation and mitral valve
prolapse.

2) Supra-valvular aortic stenosis.


J. Williams syndrome
Note:
In Williams syndrome supra-valvular aortic stenosis is the most common cardiac
lesion. Septal defects also occur as well as peripheral branch pulmonary artery
stenosis.

3) Pulmonary stenosis.
H. Noonan's syndrome
Note:
In Noonan's syndrome pulmonary valve stenosis due to a dysplastic or thick
valve is seen often associated with left ventricular hypertrophy. Branch stenosis
of the pulmonary artery also is found in Noonan's syndrome.

Dr.Kamel Youssef Hassan, Pediatrician Consultant , Palestine - Gaza


E-mail: kyh10557@yahoo.com 8
MCQs PEDIATRICS

5-Theme : CONGENITAL CARDIAC DEFECTS


A. Angelman's syndrome
B. Beckwith-Wiedemann syndrome
C. Congenital Rubella syndrome
D. Down syndrome
E. Foetal alcohol syndrome
F. Glycogen storage disease
G. Marfan's syndrome
H. Noonan's syndrome
I. Turner's syndrome
J. Williams syndrome
Match each of the following cardiovascular abnormalities to the single most
likely associated disorder.
1) Endocardial cushion defect.
D. Down syndrome
Note:
In Down syndrome approximately 40% of children have a congenital heart
disease. The most common being endocardial cushion defects although VSDs,
ASDs and PDA also occur.

2) Coarctation of the aorta.


I. Turner's syndrome
Note:
In Turner's syndrome cardiac defects are common. 30% include bicuspid aortic
valves with the second most common heart defect being coarctation of the aorta.
Aortic stenosis, mitral valve prolapse and hypertension are also found.

3) Septal defects.
E. Foetal alcohol syndrome
Note:
In foetal alcohol syndrome individuals have poor growth, developmental delay
and usually characteristic facial features including microcephaly and a short
smooth philtrum. The most common cardiac lesion in these children are septal
defects primarily ventricular septal defects.

Dr.Kamel Youssef Hassan, Pediatrician Consultant , Palestine - Gaza


E-mail: kyh10557@yahoo.com 9
MCQs PEDIATRICS

6-Theme : BREATHLESSNESS
A. Asthma
B. Hyperventilation
C. Tuberculosis
D. Cystic Fibrosis
E. Pneumocystis carinii
F. Ventricular septal defect
G. Gastroesophageal reflux
H. Atrial septal defect
I. Bronchiolitis
J. Mitral stenosis
For each of these patients with breathlessness, select the most likely diagnosis
1) A thirteen-year-old girl who has intermittent episodes of breathlessness,
which tend to occur in crowded shops. She feels the need to take deep breaths
and then breathes very quickly, complaining of pins & needles around her mouth
and in her hands. Her chest is clear and her blood gases show a normal pO2
and low pCO2.
B. Hyperventilation
Note:
This description is one of anxiety. The blood gas picture is one of
hyperventilation.
2) A three-year-old boy who presents with worsening cough and breathlessness of 3 weeks’
duration. His mother was an intravenous drug abuser. He has always been prone to
infections. When he was 2 years old he had chicken pox for 4 weeks. On examination, he has
an emaciated appearance, his weight is below the 0.4th centile, he has a temperature of
37.6°C and he has generalized crepitations on auscultation of his chest. A blood count shows
severe lymphopenia.
E. Pneumocystis carinii
Note:
This has resulted from congenitally acquired HIV. PCP has an insidious onset
and often there are no chest signs in children. Lymphopenia is consistent.
Treatment is with septrin or nebulised pentamidine as second line.
3) A 4-month-old baby has not gained much weight since birth and only takes
small milk feeds, as he appears to become breathless on feeding. He is
tachypnoeic, sweaty and has a tachycardia. His liver is enlarged and he has a
harsh grade 2-pansystolic murmur at the left lower sternal edge
F. Ventricular septal defect
Note:
Poor feeding is a symptom of heart failure in babies. A VSD has a pan systolic murmur and if
large may have a lower grade as there is less resistance to flow. Hepatomegaly is an early
sign of heart failure in infants.

Dr.Kamel Youssef Hassan, Pediatrician Consultant , Palestine - Gaza


E-mail: kyh10557@yahoo.com 10
MCQs PEDIATRICS

7-Theme : CONGENITAL HEART DISEASE


A. Ostium secundum atrial septal defect
B. Ventricular septal defect
C. Transposition of the great arteries
D. Total anomalous pulmonary venous drainage
E. Atrioventricular septal defect
F. Patent ductus arteriosus
G. Pulmonary valve stenosis
H. Coarctation of the aorta
I. Tetralogy of Fallot
J. Hypoplastic left heart syndrome

Which of the above is the most likely diagnosis in the following cases.
1) A 13-year-old girl is referred for evaluation of her short stature. She is
prepubertal. On auscultation she has an ejection systolic murmur in the second
and third left intercostals spaces radiating to the back, but is a symptomatic.
G. Pulmonary valve stenosis
Note:
The murmur describes pulmonary stenosis, which could also be a left peripheral
pulmonary stenosis. She is short and has delayed puberty and coupled with the
cardiac findings would suggest Noonan’s syndrome.

2) A 7-week-old infant presents with breathlessness on feeding and failure to


thrive. On examination his femoral pulses are difficult to feel but present. Chest
radiograph shows cardiomegaly and increased vascular markings.
H. Coarctation of the aorta
Note:
Absent or weak femoral pulses suggest coarctation. Remember association with
Turner’s syndrome.

3) An infant is seen for his 6-week check and found to have a loud ejection
systolic murmur in the third left intercostal space and a single second heart
sound on examination. There is no obvious cyanosis but a suggestion of mild
desaturation. On the chest X ray there is a concavity on the left heart border and
decreased pulmonary vascular markings.
I. Tetralogy of Fallot
Note:
Tetralogy of Fallot may present later than in the neonatal period. The ejection
systolic murmur is from the infundibular stenosis. The desaturation results from
the right to left shunt across the VSD.

Dr.Kamel Youssef Hassan, Pediatrician Consultant , Palestine - Gaza


E-mail: kyh10557@yahoo.com 11
MCQs PEDIATRICS

8-Theme : CONGENITAL HEART DISEASE


A. Atrioventricular septal defect
B. Coarctation of the aorta
C. Hypoplastic left heart syndrome
D. Ostium secundum atrial septal defect
E. Patent ductus arteriosus
F. Pulmonary valve stenosis
G. Tetralogy of Fallot
H. Total anomalous pulmonary venous drainage
I. Transposition of the great arteries
J. Ventricular septal defect

Which is the most likely diagnosis in the following cases?


1) An infant is found profoundly cyanosed and lethargic in his cot on day 2. On
auscultation there is a soft systolic murmur heard inconsistently at the left sternal
edge and a single second sound. The chest X ray shows a narrow upper
mediastinum, hypertrophied right ventricle and increased pulmonary vascular
markings. The ECG shows a normal neonatal pattern.
I. Transposition of the great arteries
Note:
Cyanosis on the second day is suggestive of a duct-dependent lesion. The rest
of the answer describes TGA.
2) A 3-week-old premature infant born at 27 weeks gestation remains ventilated
following surfactant deficient respiratory distress syndrome. On auscultation of
his chest a systolic murmur is heard at the left sternal edge and pulses are very
easy to feel. There is pulmonary plethora on chest X ray.
E. Patent ductus arteriosus
Note:
PDA is a relatively common problem in premature babies. The left to right shunt
results in excess blood flow through the lungs and frequently oxygen
dependency and difficulty in weaning from the ventilator. A loud systolic murmur
radiating to the back with easily palpable pulses are typical.
3) A 7-year-old boy is examined for a chest infection. An incidental finding of a
short systolic murmur with fixed splitting of the second heart sound is detected.
His blood pressure is normal and all pulses are normal.
D. Ostium secundum atrial septal defect
Note:
Ostium primum defects are unlikely to present incidentally but rather with heart
failure or pulmonary hypertension. Fixed splitting is typical of ostium secundum
defects

Dr.Kamel Youssef Hassan, Pediatrician Consultant , Palestine - Gaza


E-mail: kyh10557@yahoo.com 12
MCQs PEDIATRICS

3- DERMATOLOGY
1-Theme : SKIN AND HAIR ABNORMALITIES
A. Alopecia areata
B. Dermatitis artefacta
C. Trichotillomania
D. Aplasia cutis
E. Sutural alopecia
F. Ectodermal dysplasia
G. Acrodermatitis enteropathica
H. Menkes Kinky hair
I. Trichorrhexis nodosa
J. Telogen Effluvium
Select one option from the list above that is most suitable for the following
patients
1) A genetic syndrome resulting in abnormal dentition and ichthyosis
F. Ectodermal dysplasia
Note:
Ectodermal dysplasia is an X linked recessive condition with features of
alopecia, hypodontia and a defect in sweating. These individuals have
characteristic facies.

2) A 15 year old girl with a history of emotional difficulties presents with linear
markings to her forearms and an inconsistent history.
B. Dermatitis artefacta
Note:
Dermatitis artefacta, skin lesions are deliberately produced by patients who
conceal this fact from their doctors. The condition is usually found in individuals
with emotional difficulties arising from a disorder of personality.

3) A congenital diffuse cause of hair loss due to an abnormality of the hair shaft
with a child with intractable seizures.
H. Menkes Kinky hair
Note:
Menkes, kinky hair syndrome is a progressive cerebral deterioration associated
with seizures and twisted fractured hair. It is X linked recessive and the
underlying aetiology is an abnormality of Copper transport resulting in low
Copper and low Caeruloplasmin.

Dr.Kamel Youssef Hassan, Pediatrician Consultant , Palestine - Gaza


E-mail: kyh10557@yahoo.com 13
MCQs PEDIATRICS

2-Theme : GENETICS - CLINICAL ABNORMALITIES OF LIMBS


A. Bloom's syndrome
B. Cockayne's syndrome
C. Down's syndrome
D. Ehlers Danlos syndrome
E. Hunter's syndrome
F. Prader-Willi syndrome
G. Rubinstein-Taybi syndrome
H. Russell Silver syndrome
I. Sotos' syndrome
J. Williams syndrome
In a child with a suspected genetic disorder and the following clinical
abnormalities choose the single most likely diagnosis from the list of options.
1) Simian creases
C. Down's syndrome
Note:
Simian creases are found in approximately 45% of babies with Down's
syndrome. This is a single horizontal palmer crease. 85% of affected individuals
have a distal positioning of the palmer axial tri-radius. Also a syndrome 'Sandal
gap' is found. This is wide gap between the first and second toes. Plantar
creases between the first and second toes are usually deep and there is
characteristic dermal ridge pattern.
2) Broad thumbs and toes.
G. Rubinstein-Taybi syndrome
Note:
Children with Rubinstein-Taybi syndrome are short in stature, which has a post-
natal onset. They tend to have downward slanting palpable fissures and a
hypoplastic mandible. Examination of the limbs may reveal broad thumbs and
toes, flat feet and deep plantar creases.
3) Large hands and feet and a high arched palate.
I. Sotos' syndrome
Note:
Sotos' syndrome is also called cerebral gigantism. Birth length is usually over
the 90th centile and there is rapid linear growth there on. Individuals have large
hands and feet and macrocephaly. They have downward slanting eyes and a
high arched palate and most children with Sotos' syndrome have learning
difficulties.
Comments:
Examination of the hand sand feet are important in the assessment of a
dysmorphic infant.

Dr.Kamel Youssef Hassan, Pediatrician Consultant , Palestine - Gaza


E-mail: kyh10557@yahoo.com 14
MCQs PEDIATRICS

3-Theme : CUTANEOUS MANIFESTATIONS OF DISEASE


A. Koplik Spots
B. Erythema Multiforme
C. Erythema nodosum
D. Dermatitis Herpetiformis
E. Vitiligo
F. Gottron's papules
G. Alopecia areata
H. Mycosis Fungoides
I. Dermatitis artefacta
J. Café au lait spots
For each of the following diseases choose the most specific mucuous or
cutaneous manifestation from the list of options.
1) Addison's disease
E. Vitiligo
Note:
Vitiligo is a disorder resulting from insufficiency of melanocytes deposited in the
skin. It is associated with auto-immune conditions.

2) Cutaneous T-cell lymphoma


H. Mycosis Fungoides
Note:
Mycosis Fungoides is acute alias T-cell lymphoma often presenting as
superficial patchy dermatitis which is then developed in to tumours.

3) Coeliac disease
D. Dermatitis Herpetiformis
Note:
Dermatitis herpetiformis is a pruritic fascicular rash found on elbows, shoulder,
buttocks and knees and is associated with Coeliac disease.

Dr.Kamel Youssef Hassan, Pediatrician Consultant , Palestine - Gaza


E-mail: kyh10557@yahoo.com 15
MCQs PEDIATRICS

4-Theme : SKIN AND HAIR ABNORMALITIES


A. Alopecia areata
B. Dermatitis artefacta
C. Trichotillomania
D. Aplasia cutis
E. Sutural alopecia
F. Ectodermal dysplasia
G. Acrodermatitis enteropathica
H. Menkes Kinky hair
I. Trichorrhexis nodosa
J. Telogen Effluvium
Select one option from the list above that is most suitable for the following
patients
1) A congenital cause for localised hair loss.
D. Aplasia cutis
Note:
Aplasia cutis is a localised defect with the scalp. This can be an isolated finding
although has been associated with Trisomies, particularly Patau syndrome.

2) An acquired localised area of hair loss related to Tinea capitis


A. Alopecia areata
Note:
Alopecia areata is a local area of hair loss in this case secondary to underlying
ringworm.

3) An acquired course of hair loss resulting from an emotionally disturbed


teenager.
C. Trichotillomania
Note:
Trichotillomania is a term for hair loss as a result of pulling of his/her own hair
out, there is often a history of emotional difficulties.

Dr.Kamel Youssef Hassan, Pediatrician Consultant , Palestine - Gaza


E-mail: kyh10557@yahoo.com 16
MCQs PEDIATRICS
5- Theme : CUTANEOUS MANIFESTATIONS OF DISEASE
A. Koplik Spots
B. Erythema Multiforme
C. Erythema nodosum
D. Dermatitis Herpetiformis
E. Vitiligo
F. Gottron's papules
G. Alopecia areata
H. Mycosis Fungoides
I. Dermatitis artefacta
J. Café au lait spots
For each of the following diseases choose the most specific mucuous or
cutaneous manifestation from the list of options.
1) Measles
A. Koplik Spots
Note:
Koplik spots are of pathognomonic measles found on the buccal mucosa
membrane and resembles coarse granules of salt.

2) Dermatomyositis
F. Gottron's papules
Note:
Gottron's papules are pathognomonic for Dermatomyositis. They are
inflammatory papules found over the dorsal interphalangeal joints and they
become violaceous in colour and flattened before they atrophy to leave.

3) Stevens-Johnson syndrome
B. Erythema Multiforme
Note:
Hypopigmentation – Erythema multiforme major is also known as Stevens-
Johnson syndrome. It is a serious systemic disease which involve the two
mucous membranes plus skin. Often follows respiratory symptoms and is
associated with a conjunctivitis, uveitis and bullae. It may result in fluid loss,
weakness as well as anaemia and neutropenia.

Dr.Kamel Youssef Hassan, Pediatrician Consultant , Palestine - Gaza


E-mail: kyh10557@yahoo.com 17
MCQs PEDIATRICS

4- GENETICS
1-Theme : DIAGNOSIS OF CHROMOSOMAL DISORDERS
A. Di -George Anomaly
B. Fragile X
C. Karyotype 45XO
D. Karyotype 46XX
E. Karyotype 47XYY
F. Klinefelter syndrome
G. Triploidy
H. Trisomy 9
I. Trisomy 18
J. Trisomy 21
For each of the following descriptions of clinical abnormalities choose the single
most likely genetic disorder from the list of options
1) A 13 year old boy with learning difficulties, undescended testicles and
gynaecomastia.
F. Klinefelter syndrome
Note:
Klinefelter syndrome (XXY). Individuals with Klinefelter’s usually have learning
difficulties. Phenotypically they are relatively tall and slim individuals with small
penis and testicles, gynaecomastia and infertility.
2) Gross foetal oedema, a cystic hygroma and ultrasound appearances of
female foetus.
C. Karyotype 45XO
Note:
Turner’s syndrome (45XO) affects females. Features include short stature and gonadal
agenesis. Ultrasound scan of the foetus in the utero may confirm lymphoedema. Other
features include a webbed neck, a broad shield like chest, wide spaced nipples and cubitus
valgus. Cardiac anomalies are common in Turner’s syndrome the most common being a
bicuspid aortic valve and coaptation of the aorta. Renal anomalies such as horse shoe
kidneys are often seen
3) A cardiac abnormality in a child with immuno deficiency and mild – moderate
learning difficulties
A. Di -George Anomaly
Note:
Di -George syndrome, which is a defect of development of the thymus, parathyroid as well as
great vessels. They tend to have characteristic facies with short palpable fissures,
micrognathia and a short philtrum. They have reduced cellular immunity and
hypoparathyroidism leads to hypocalcaemia and seizures. Cardiac anomalies are also
common for example aortic arch defects.

Dr.Kamel Youssef Hassan, Pediatrician Consultant , Palestine - Gaza


E-mail: kyh10557@yahoo.com 18
MCQs PEDIATRICS

2-Theme : DIAGNOSIS OF CHROMOSOMAL DISORDERS


A. Di -George Anomaly
B. Fragile X
C. Karyotype 45XO
D. Karyotype 46XX
E. Karyotype 47XYY
F. Klinefelter syndrome
G. Triploidy
H. Trisomy 9
I. Trisomy 18
J. Trisomy 21
For each of the following descriptions of clinical abnormalities choose the single
most likely genetic disorder from the list of options.
1) Abnormal ears and facies, rocker bottom feet and cardiac abnormalities
I. Trisomy 18
Note:
Trisomy 18 or Edwards syndrome, which has a very poor prognosis with 50% of
affected individuals dying within the first week of life. Growth is poor and cardiac
anomalies are severe. Babies with Edwards syndrome tend to have clenched
hands with overlapping fingers and their feet are usually deformed with a “rocker
bottom appearance” and equinovarus.

2) Eye manifestations such as squints, refractive errors and Brushfield’s spots.


J. Trisomy 21
Note:
The eye manifestations of squints, refractive errors and Brushfield's spors
relates to Trisomy 21 or Down syndrome. Affected individuals have learning
difficulties. Eye manifestations include strabismus, refractive errors, epicanthic
folds, Brushfield’s spots which are flecks on the iris. They tend to have almond
shaped eyes with the outer aspect of the palpable fissure being upwardly
slanting.

3) Learning difficulties, prominent ears and testicular enlargement.


B. Fragile X
Note:
Fragile X syndrome, which can affect both sexes. Males with Fragile X have
macro-orchidism, (testicular enlargement), prominent ears, moderate learning
difficulties and often speech delay.

Dr.Kamel Youssef Hassan, Pediatrician Consultant , Palestine - Gaza


E-mail: kyh10557@yahoo.com 19
MCQs PEDIATRICS

3-Theme : GENETICS - CLINICAL FEATURES OF GENETIC DISORDER


SYNDROMES
A. Angelman's syndrome
B. Cri-du-chat syndrome
C. Cornelia de-Lange syndrome
D. Edwards syndrome
E. Kartagener's syndrome
F. Marfan's syndrome
G. Noonan's syndrome
H. Patau's syndrome
I. Turner's syndrome
J. Williams syndrome
For each of the following descriptions of clinical abnormalities choose the single
most likely genetic disorder from the list of options.
1) Synophrys, limb defects and marked retardation of growth.
C. Cornelia de-Lange syndrome
Note:
Cornelia de-Lange – Small stature, moderate to severe learning difficulties. Limb
abnormalities for example oligodactyly and phocomelia. Facial dysmorphic
features included synophrys, bushy eyebrows and a long philtrum
2) Learning difficulties, pectus excavatum and heart defects.
G. Noonan's syndrome
Note:
Noonan’s syndrome is a condition which has a similar phenotype to Turner's
syndrome. Individuals have learning difficulties, a webbed neck, pectus
excavatum and cardiac anomalies include pulmonary valve stenosis and
branched stenosis of the pulmonary artery.
3) An elf like face, learning difficulties and hypocalcaemia.
J. Williams syndrome
Note:
Williams syndrome – The disorder including learning difficulties an unusual
characteristic, elf like face with prominent lips. Epicanthic folds and peri-orbital
puffiness. Hypercalcaemia is an associated finding and cardiovascular
anomalies mainly supra-valvular aortic stenosis has been found in children with
Williams syndrome.

Dr.Kamel Youssef Hassan, Pediatrician Consultant , Palestine - Gaza


E-mail: kyh10557@yahoo.com 20
MCQs PEDIATRICS

4-Theme : GENETICS - CLINICAL FEATURES OF GENETIC DISORDER


SYNDROMES
A. Angelman's syndrome
B. Cri-du-chat syndrome
C. Cornelia de-Lange syndrome
D. Edwards syndrome
E. Kartagener's syndrome
F. Marfan's syndrome
G. Noonan's syndrome
H. Patau's syndrome
I. Turner's syndrome
J. Williams syndrome
For each of the following descriptions of clinical abnormalities choose the single
most likely genetic disorder from the list of options.
1) Microcephaly, dysmorphic features and an abnormal cry.
B. Cri-du-chat syndrome
2) Micropthalmia, scalp defects, polydactyly.
H. Patau's syndrome
3) Arachnodactyly, high arched palate and lens dislocation.
F. Marfan's syndrome
Comments:
Cri-du-chat syndrome is due to a deletion of the short arm of chromosome 5.
Affected individuals have a low birth weight, microcephaly and learning
difficulties. They have a characteristic high pitch cry and facial dysmorphic
features including low set ears and epicanthic folds. Congenital heart disease is
common. Patau syndrome is Trisomy 13 affected individuals have a poor
prognosis with 80% dying within the first month of life. There is an incomplete
development of the forebrain and optic nerves and skull defects are common for
example cutis aplasia, microphthalmia and colobomata seen as well as cleft
palate and polydactyly. Cardiac abnormalities are present in approximately 80%
of babies with Patau’s syndrome. Marfan's syndrome – phenotypically children
with Marfan’s syndrome have tall stature, slim limbs, arachnodactyly and joint
hyper-extensibility. Cardiac anomalies are associated for example dilation of the
aorta, aorta regurgitation and mitral valve prolapse. Eye lens dislocation is often
seen.

Dr.Kamel Youssef Hassan, Pediatrician Consultant , Palestine - Gaza


E-mail: kyh10557@yahoo.com 21
MCQs PEDIATRICS

5- INFECTIOUS DISEASES
1) DIARRHOEA
A. Coeliac disease
B. Crohns disease
C. Ulcerative colitis
D. Cows milk protein intolerance
E. Toddlers diarrhoea
F. Cystic fibrosis
G. Lactose intolerance
H. Irritable bowel syndrome
I. Abdominal migraine
J. Haemolytic uraemic syndrome
For the following children presenting with diarrhoea which is the most likely
diagnosis:
1) An 11 month infant presents with chronic diarrhoea and failure to thrive since
breast feeding was discontinued at 9 months. He has been treated by his GP
with antibiotics on two occasions for chest infections. Initial investigations reveal
low serum potassium, chloride and metabolic alkalosis.
F. Cystic fibrosis
Note:
Failure to thrive, chest infections and diarrhoea (the latter results from failure of pancreatic
exocrine function) are indicative of CF. The abnormal biochemistry is caused by salt loss and
may be described as pseudo-bartters syndrome

2) A 15 year old boy presents with delayed puberty and short stature. He gives a
history of intermittent abdominal pain and diarrhoea. He recently presented to
his family doctor with a painful red swelling on his shin which has now resolved.
B. Crohns disease
Note:
Crohn’s disease may present in this way, the skin lesions are erythema
nodosum which are associated with inflammatory bowel disease.

3) A 8 month old infant has recurrent diarrhoea. Mother dates the start of
symptoms from the an episode of gastroenteritis. The diarrhoea is explosive in
nature
G. Lactose intolerance
Note:
Secondary lactose intolerance is a known sequelae of viral gastroenteritis. It is
not prevented by regrading milk following the episode of viral infection.
Dr.Kamel Youssef Hassan, Pediatrician Consultant , Palestine - Gaza
E-mail: kyh10557@yahoo.com 22
MCQs PEDIATRICS

2) CONTRA-INDICATIONS TO VACCINES
A. BCG
B. Diphtheria Tetanus Polio
C. Hepatitis B
D. Haemophilus Influenzae B
E. Influenza
F. Measles Mumps Rubella
G. Polio
H. Rubella
I. Tetanus
J. Tuberculin
Which of the vaccines listed above would be contra-indicated in the following
scenarios:
1) Contra-indication if known to have a hypersensitive reaction to egg.
E. Influenza
Note:
relates to hypersensitivity to egg, which contra-indicates the Influenza vaccine
as there is residual egg protein present.
2) Contra-indication if known to have an allergy to gelatin.
F. Measles Mumps Rubella
Note:
refers to contra-indications to MMR, which include children with allergies to
Gelatin, Neomycin or kanamycin as well as children with untreated malignant
disease or altered immunity. Those receiving immuno-suppressive drugs or
radiotherapy are also contraindicated. Children who have received another live
vaccine by injection within 3 weeks should not be given MMR vaccine. MMR
should also not be given within 3 months of receiving an Immunoglobulin
injection.
3) Contra-indication if known to have an allergy to kanamycin.
F. Measles Mumps Rubella
Note:
refers to contra-indications to MMR, which include children with allergies to
Gelatin, Neomycin or kanamycin as well as children with untreated malignant
disease or altered immunity. Those receiving immuno-suppressive drugs or
radiotherapy are also contraindicated. Children who have received another live
vaccine by injection within 3 weeks should not be given MMR vaccine. MMR
should also not be given within 3 months of receiving an Immunoglobulin
injection.

Dr.Kamel Youssef Hassan, Pediatrician Consultant , Palestine - Gaza


E-mail: kyh10557@yahoo.com 23
MCQs PEDIATRICS

3) CHILDHOOD CHEST INFECTIONS


A. Chlamydia
B. Cytomegalovirus
C. Group B Streptococcus
D. Haemophilus Influenza
E. Mycoplasma pneumonia
F. Parainfluenza
G. Respiratory syncytial virus
H. Rhinovirus
I. Staphylococcus aureus
J. Ureaplasma
Select the most appropriate pathogen from the above list that would account for
the following presentations:
1) The commonest cause of pneumonia in the 2 month - 6month age group.
G. Respiratory syncytial virus
2) A 9 year old boy presents with a cough and fever. Chest x-ray demonstrates a
reticular nodular pattern with high adenopathy and a small pleural effusion.
E. Mycoplasma pneumonia
3) A 5 year old boy presents with painful red swellings of his shins. Examination
of his throat reveals an exudative tonsillitis.
C. Group B Streptococcus
Comments:
The commonest cause of pneumonia in the 2month – 6 month age group is
Respiratory syncytial virus, which results in bronchiolitis and occurs in
epidemics. Mycoplasma pneumonia mainly affects older children (between 5
and 15 years) resulting in community acquired pneumonias. Tonsillitis can occur
at any age although is frequently seen in the 4-7 age group. The majority of
episodes are caused by viruses although beta haemolytic streptococcal infection
is also responsible and has systemic reactions such as erythema nodosum,
rheumatic fever and glomerular nephritis

Dr.Kamel Youssef Hassan, Pediatrician Consultant , Palestine - Gaza


E-mail: kyh10557@yahoo.com 24
MCQs PEDIATRICS

4) CHILDHOOD DIARRHOEA
A. Adenovirus
B. Cytomegalovirus
C. Epstein-Barr virus
D. E. Coli
E. Giardia Lamblia
F. Hepatitis A
G. Norwalk virus
H. Rotavirus
I. Salmonella species
J. Staphylococcus aureus

Select one option from the list above that is most suitable for the following
patients
1) Accepted as the most common cause of infectious diarrhoea in children in the
developed society.
H. Rotavirus
Note:
Rotavirus is the most common virus responsible for diarrhoea worldwide. It is a
double stranded RNA virus, especially prevalent in the winter months. It causes
fever and watery diarrhoea.

2) Can follow ingestion of dust containing dried faecal material.


F. Hepatitis A
Note:
Hepatitis A is an RNA virus typically transmitted by the faecal-oral route.

3) Attaches the small intestinal border and releases extoxin.


D. E. Coli
Note:
E-coli and enteropathogen is one of the bacterial causes of diarrhoea. Many of
its effects are a consequence of an exotoxin. It should be suspected if there is a
history of travel or a history of ingestion of poorly prepared food. Bacterial
diarrhoea usually results in bloody diarrhoea.

Dr.Kamel Youssef Hassan, Pediatrician Consultant , Palestine - Gaza


E-mail: kyh10557@yahoo.com 25
MCQs PEDIATRICS

5) URINARY TRACT DISEASE


A. Haemolytic uraemic syndrome
B. Henoch Schonlein purpura
C. Nephrotic syndrome
D. Polycystic kidneys
E. Pyelonephritis
F. Renal Calculi
G. Renal tubular acidosis
H. Renal vein thrombosis
I. Systemic lupus erythematosus
J. Wilm's tumour
Select the most appropriate diagnosis from the above list of options that would
best explain the following cases:
1) A 2 year old boy is admitted with a history of bloody diarrhoea, abdominal pain and
puffiness around his eyes. He is found to be slightly jaundiced. His urea is 12.6 and Creatinine
163 mmol / liter
A. Haemolytic uraemic syndrome
Note:
describes a child with haemolytic uraemic syndrome, which is of unknown aetiology. Various
agents have been implicated including E.coli (0157) Salmonella, Shigella and viruses. It
usually presents with a prodromal symptoms of an upper respiratory tract infection or
gastroenteritis and later develops in to pallor, oliguria and hypertension.
Laboratory investigations may reveal microangiopathic haemolytic anaemia,
thrombocytopenia and urinalysis may reveal proteinuria.
2) A 9 day old baby is admitted with a 2 day history of vomiting and diarrhoea. During the
admission haematuria is noted and he has had 2 convulsions. A mass is palpable in the left
hypochondrium.
H. Renal vein thrombosis
Note:
describes a child with renal vein thrombosis. In neonates it is associated with dehydration,
asphyxia, shock or sepsis.
Manifestations include sudden onset of haematuria and identification of an enlarged kidney. If
both kidneys are involved it may result in acute renal failure.

3) A 5 year old girl presents with puffiness around the eyes. Urinalysis confirms
proteinuria with a trace of blood.
C. Nephrotic syndrome
Note:
relates to a child with Nephrotic syndrome. This occurs when there is proteinuria resulting in
hypoalbuminaemia and oedema with an unknown aetiology. Peri-orbital or dependant
oedema is usually noted first. There may also be a history of abdominal pain, vomiting and
diarrhoea. The child needs to be monitored carefully as hypovolaemia and circulatory
collapse. Steroids are the treatment of choice with careful management of fluids.

Dr.Kamel Youssef Hassan, Pediatrician Consultant , Palestine - Gaza


E-mail: kyh10557@yahoo.com 26
MCQs PEDIATRICS

6) NEONATAL JAUNDICE

A. Physiological jaundice
B. Biliary atresia
C. Hypothyroidism
D. Rhesus incompatibility
E. Congenital spherocytosis
F. Congenital cytomegalo virus infection
G. Galactosaemia
H. Glucose-6-phosphate dehydrogenase deficiency
I. Cystic fibrosis
J. Fructose intolerance

For each of the following jaundiced babies, select the most likely cause:

1) A 13 day old infant who was noted to have an umbilical hernia and has very
dry skin presents with jaundice. She is a floppy baby. Her bilirubin is checked
and is found to be elevated and mainly unconjugated. The community midwife
has been unable to gain access to the home for the last week.
C. Hypothyroidism
Note:
Umbilical hernia, dry skin, hypotonia and jaundice are features of congenital
hypothyroidism.

2) A caucasian infant has required surgery at the regional neonatal unit for meconium ileus
and has developed jaundice. The serum conjugated bilirubin is 65 micromol/L. The diagnosis
is eventually confirmed from the neonatal screening tests.
I. Cystic fibrosis
Note:
Meconium ileus and jaundice may be presenting features of CF in the neonatal
period. The screening test is immunoreactive trypsin.
3) A 12 day old male baby has a conjugated and unconjugated hyperbilirubinaemia. He is
breast fed and has become more disinterested in feeds. He was investigated for sepsis and
blood cultures demonstrated E-coli septicaemia, urine cultures were clear. His clotting is
deranged.
G. Galactosaemia
Note:
Galactosaemia typically presents around 2 weeks of age with jaundice. E coli
septicaemia is a feature. Treatment is by removing galactose from the diet.
Cataracts are a later feature even if treatment instituted early.

Dr.Kamel Youssef Hassan, Pediatrician Consultant , Palestine - Gaza


E-mail: kyh10557@yahoo.com 27
MCQs PEDIATRICS

7) VIRAL INFECTIONS
A. Measles
B. Rubella
C. Chicken Pox
D. Herpes Simplex
E. Mumps
F. Glandular fever
G. Pertussis
H. Polio myelitis
I. Hepatitis A
For each description below choose the single most likely viral infection from the
list of options.
1) May result in Giant cell pneumonia.
A. Measles
Note:
Measles infection is uncommon due to world-wide immunisation. A child with
measles usually has a fever, upper respiratory tract symptoms and a
morbilliform rash. Serious complications include a giant cell pneumonia and
encephalitis (SSPE).

2) May result in a Keratoconjuctivitis.


D. Herpes Simplex
Note:
Herpes simplex infection may be transmitted verdantly to an infant from their
mother's genital tract. It may cause isolated skin lesions, a Keratoconjunctivitis
or a paronychia. More seriously it is also responsible for encephalitis.

3) Caused by an RNA virus with no known carrier state.


I. Hepatitis A
Note:
Hepatitis A, which is caused by an RNA virus, is usually transmitted by the oral
route. It has an incubation period of between 15-50 days and treatment is
usually symptomatic only.

Dr.Kamel Youssef Hassan, Pediatrician Consultant , Palestine - Gaza


E-mail: kyh10557@yahoo.com 28
MCQs PEDIATRICS

8) URINARY TRACT DISEASE


A. Haemolytic uraemic syndrome
B. Henoch Schonlein purpura
C. Nephrotic syndrome
D. Polycystic kidneys
E. Pyelonephritis
F. Renal Calculi
G. Renal tubular acidosis
H. Renal vein thrombosis
I. Systemic lupus erythematosus
J. Wilm's tumour
Select the most appropriate diagnosis from the above list of options that would
best explain the following cases:
1) An 11 year old with a previous history of chronic glomerulonephritis presents
with bruising and epistaxis. A full blood count confirms a pancytopenia.
I. Systemic lupus erythematosus
Note:
describes a child with glomerulonephritis and bone marrow failure suggestive of
a connective tissue disease such as SLE.

2) A 14 year old boy with a history of recurrent urinary tract infections present
with severe abdominal pain radiating to his back. Dysuria and haematuria.
F. Renal Calculi
Note:
relates to Urolithiasis, Renal Calculi. Children present with abdominal pain,
voiding abnormalities, dysuria, haematuria may be present. Avoidance of
dehydration is important. Treatment may require lithotripsy.

3) A 1 year old girl with a 3 month history of vomiting is investigated for failure to
thrive. She is found to be mildly acidotic.
G. Renal tubular acidosis
Note:
Renal tubular acidosis is the answer to Item 3. It is a clinical state of systemic
hyperchloraemic acidosis resulting from impaired urinary acidification. Three types exist. Type
1 distal RTA, type 2 proximal RTA and type 4 mineralocorticoid deficiency. Type 3 is thought
to be a variant of type 1. Types 1, 2 and 4 each have several causes. Children with isolated
forms of proximal or distal commonly present with growth failure; gastrointestinal symptoms
are also common. Nephro-calcinosis and hypercalciuria may complicate distal RTA and goals
of treatment are to correct the acidosis and to maintain normal Bicarbonate and Potassium
levels.

Dr.Kamel Youssef Hassan, Pediatrician Consultant , Palestine - Gaza


E-mail: kyh10557@yahoo.com 29
MCQs PEDIATRICS

9) MUCOCUTANEOUS INFECTIONS AND INFESTATIONS


A. Enterobiasis vermicularis
B. Chlamydia
C. Candidiasis
D. Tinea pedis
E. Pityriasis rosea
F. Scabies
G. Ringworm
H. Lichen sclerosis
I. Napkin dermatitis
J. Lichen planus
For each of the following children who present with an itch, select the most
appropriate diagnosis from the list of options:
1) A child complains of an itchy rash over his arms. His brother and sister have
similar symptoms.
F. Scabies
Note:
Scabies is caused by the mite Sarcoptes scabiei hominis. Transmission is
through close body contact, the adult mites lay their eggs in burrows in the skin
and it results in an eczematous rash with parotitis skin.

2) A baby has an excoriated perineal rash involving the flexures.


C. Candidiasis
Note:
Candidiasis is caused by Candida albicans (yeast). In babies it presents as a
perineal rash which usually affects the flexures. Satellite lesions may also be
seen.

3) A boy has an itchy rash over the trunk with a solitary large oval lesion on the
back.
E. Pityriasis rosea

Note:
Pityriasis rosea is a benign condition of the skin resulting in oval pink / brown
scaly lesions over the trunk, which are usually preceded by a herald patch (a
solitary large lesion usually between 1 –10cm). No treatment is required.

Dr.Kamel Youssef Hassan, Pediatrician Consultant , Palestine - Gaza


E-mail: kyh10557@yahoo.com 30
MCQs PEDIATRICS

10) CHILDHOOD RESPIRATORY DISEASES


A. Asthma
B. Bronchiolitis
C. Croup
D. Cystic fibrosis
E. Diphtheria
F. Epiglottis
G. Pneumonia
H. Influenza
I. Retropharyngeal abscess
J. Whooping cough
For each patient below, choose the SINGLE most probable diagnosis from the
above list of options. Each option may be used once, more than once or not at
all.
1) A six month old baby presents with high fever, breathlessness, cough and
feeding difficulties. Chest examination reveals dull percussion note over the right
base posteriorly with bronchial breath sounds on auscultation.
G. Pneumonia
Note:
Pneumonia may occur at any age. Patients present with fever, tachypnoea,
feeding difficulties and cyanosis. Examination reveals bronchial breath sounds
and crepitations. Chest X ray may show consolidation. Common organisms are
pneumococcus, haemophilus, staphylococcus, mycoplasma, TB and viruses.

2) A 1 year old baby boy is wheezy, coughing, cyanosed and breathless with
intercostal recession.
B. Bronchiolitis
Note:
Acute bronchiolitis is very common in infancy. In winter epidemics of respiratory
syncytial virus infection are the commonest cause. Wheeze, cough, fever and
respiratory distress are common. Chest X ray shows hyperinflation.

3) A 4 year old non immunized boy presents with bouts of coughing ending in
vomiting. He has an absolute lymphocytosis.
J. Whooping cough
Note:
Whooping cough is caused by Bordetella pertussis infection. Bouts of coughing
ending in vomiting, especially at night and after feeding suggest the diagnosis.
The characteristic whoop, forced inspiration through a closed glottis may or may

Dr.Kamel Youssef Hassan, Pediatrician Consultant , Palestine - Gaza


E-mail: kyh10557@yahoo.com 31
MCQs PEDIATRICS

not be present. Absolute lymphocytosis is common. Complications include CNS


haemorrhages, rectal prolapse and bronchiectasis.

4) A nine month old baby girl is upset and has stridor. Her voice is hoarse and
has a barking cough. She has a low grade fever.
C. Croup
Note:
Croup usually occurs in epidemics in autumn or spring. Causative viruses are
Parainfluenza (types 1, 2, or 3), respiratory syncytial viruses and measles virus.
Onset is over a few days; stridor is harsh and occurs only when child is upset. A
barking cough, harsh voice and ability to swallow secretions are typical.
5) A 2 year old boy is very unwell. His temperature is 39°C and he is unable to
swallow his secretions.
F. Epiglottis
Note:
Acute epiglottis is due to Haemophilus influenza type B infection. It is
characterised by sudden onset, high fever, continuous stridor and drooling of
secretions. Intravenous antibiotics, anaesthetic support are usually indicated.

Dr.Kamel Youssef Hassan, Pediatrician Consultant , Palestine - Gaza


E-mail: kyh10557@yahoo.com 32
MCQs PEDIATRICS

11) TREATMENT OF INFECTIOUS DISEASE


A. Acyclovir
B. Acyclovir plus Cefotaxime
C. Ampicillin plus Gentamicin
D. Benzylpenicillin
E. Ceftriaxone
F. Ciprofloxacin
G. Flucloxacillin
H. Netilmicin
I. Symptomatic treatment
J. Teicoplanin
For each case of infectious disease described below choose the single best
treatment option from the list.
1) A 4 year old with a 3 day history of vomiting and diarrhoea.
I. Symptomatic treatment
Note:
A child with a history of gastroenteritis is a very common childhood illness.
Conservative treatment only is required.

2) A 2 year old child with an immune deficiency develops Chicken Pox.


A. Acyclovir
Note:
A child who is immuno-deficient and therefore must be provided with Zoster
Immune Globulin if exposed to Chicken pox through contact. If the Chicken pox
develops, treatment with Acyclovir is required.
3) A 7 year old child with widespread impetigo.
G. Flu-cloxacillin
Note:
Impetigo is a common staphylococcal skin manifestation, which is highly
contagious. Most frequent sites being nostrils and the peri-oral area.
Flucloxacillin is the treatment of choice.

Dr.Kamel Youssef Hassan, Pediatrician Consultant , Palestine - Gaza


E-mail: kyh10557@yahoo.com 33
MCQs PEDIATRICS

12) EXANTHEM
A. Staphylococcal scalded skin syndrome
B. Rubella
C. Measles
D. Kawasaki’s disease
E. Impetigo
F. Scarlatina
G. Infectious mononucleosis
H. Henoch – Schonlein purpura
I. Meningococcal infection
J. Stills disease (systemic onset juvenile chronic arthritis)
Match the following descriptions of rash with the illness for which they are the
most typical exanthem
1) A 5 year old boy has a bright, red, punctate, erythematous rash which
blanches on pressure, beginning in the axillae with some perioral pallor and
relative facial sparing. The skin feels like “sandpaper”. The rash fades and
desquamates on the hands and feet. A thick white exudate develops on the
tongue which peels leaving a “strawberry tongue” with prominent papillae.
F. Scarlatina(scarlet fever)
Note:
This description is typical of Scarlatina i.e. beta haemolytic streptococcal
infection. The rash may be confused with that of Kawasaki disease. However,
desquamation occurs in late in Kawasaki’s disease.

2) A maculopapular rash develops in a child with sore throat and fever who has
been treated with ampicillin.
G. Infectious mononucleosis
Note:
This is a known effect of giving ampicillin during EBV infection.

3) A 3 year old child presents with high fever for 7 days, conjunctival injection,
fissuring of the lips and strawberry tongue, erythema followed by desquamation
of the hands and feet, and a macular rash over the trunk with cervical
lymphadenopathy.
D. Kawasaki’s disease
Note:
There are major and minor features of Kawasaki disease. High fever and
desquamation are typical.

Dr.Kamel Youssef Hassan, Pediatrician Consultant , Palestine - Gaza


E-mail: kyh10557@yahoo.com 34
MCQs PEDIATRICS

13) PRESENTATION OF INFECTIOUS DISEASE IN CHILDHOOD


A. Chicken Pox
B. Herpes simplex
C. Infectious mononucleosis
D. Measles
E. Mumps
F. Mycoplasma
G. Pertussis
H. Rubella
I. Tuberculosis
For each presentation of infectious disease choose the single most likely
diagnosis from the list of options.
1) May be complicated by cerebellar ataxia.
A. Chicken Pox
Note:
Chicken pox is a common childhood illness caused by Varicella. Vesicles
usually appear as crops over the trunk. Viral cerebellitis is a complication.
2) May present with apnoeas during infancy.
G. Pertussis
Note:
Pertussis is caused by Bordetella pertussis. In infancy it can be dangerous often
presenting with apnoeas. Signs of an upper respiratory tract infection are also
common with a paroxysmal cough (whoop). Babies may also present with
vomiting and cyanosis. A blood count will reveal a lymphocytosis. Complications
include broncho-pneumonia and bronchiectasis as sequelae is also associated
though uncommon.

3) Associated with a maculo-papular rash that typically starts on face and


extends to rest of body
H. Rubella
Note:
Rubella infection is uncommon now due to widespread MMR vaccination.
Typically causes a mild illness which may go undetected in 25-50% of cases.
Typical symptoms are transient macular papular rash that starts on the face and
extends to body and a low grade fever. Complications include arthritis,
encephalitis and thrombocytopenia.

Dr.Kamel Youssef Hassan, Pediatrician Consultant , Palestine - Gaza


E-mail: kyh10557@yahoo.com 35
MCQs PEDIATRICS

14) RASHES
A. Staphylococcal scalded skin syndrome
B. Rubella
C. Measles
D. Kawasaki’s disease
E. Impetigo
F. Scarlatina
G. Infectious mononucleosis
H. Henoch – Schonlein purpura
I. Meningococcal infection
J. Stills disease (systemic onset juvenile chronic arthritis)
Match the following descriptions of rash with the illness for which they are the
most typical exanthem.
1) A salmon-coloured, reticulate macular rash develops mainly over the extensor
surfaces of the limbs in a 5 year old boy with swinging temperature; hot, swollen,
painful knees and left elbow and palpable spleen. The ESR is 95. The blood
count, C-reactive protein and chest X-ray are normal.
J. Stills disease (systemic onset juvenile chronic arthritis)
Note:
“Salmon-coloured” is the description used to describe the rash of Still’s disease.
The distribution is not that of HSP which covers typically the buttocks and limbs
and is a purpuric rash.

2) A 12 year old boy develops petechiae and papules, some of which become
purpuric over his buttocks and legs, associated with painful swollen knees.
There is microscopic haematuria on testing. The platelet count is normal.
H. Henoch – Schonlein purpura
Note:
This is a description of HSP and the well recognised complication of HSP
nephritis. A proportion of these patients will develop progressive nephritis and
end stage renal failure.

3) A 5 day old girl has a high temperature and is irritable. She has areas of desquamation
over her finger tips and in the axillae. Her carer notices that her skin blisters easily following
minimal contact.
A. Staphylococcal scalded skin syndrome
Note:
SSSS results from infection with staphylococci with the exofoliative toxin A and B. These
exotoxins cause disruption to the epidermal layer by interfering with intercellular junctions.
Mortality is up to 3% in children. A positive Nikolsky sign (slippage of the superficial layer of the
epithelium on gentle pressure) The desquamation occurs concomitantly with the illness unlike
Kawaskis disease and Kawasaki disease does not occur in this age group. There may be a
history of minimal skin trauma which provides a port of entry for the organism.

Dr.Kamel Youssef Hassan, Pediatrician Consultant , Palestine - Gaza


E-mail: kyh10557@yahoo.com 36
MCQs PEDIATRICS

15) DIARRHOEA
A. Coeliac disease
B. Crohns disease
C. Ulcerative colitis
D. Cows milk protein intolerance
E. Toddlers diarrhoea
F. Cystic fibrosis
G. Lactose intolerance
H. Irritable bowel syndrome
I. Abdominal migraine
J. Haemolytic uraemic syndrome
For the following children presenting with diarrhoea which is the most likely
diagnosis:
1) A 6 year old boy with Downs’s syndrome has between 3 and 4 loose stools a
day. He is pale but otherwise the examination is unremarkable. When plotting
his growth on a chart specific for Downs syndrome children it is clear that both
height and weight have fallen across 2 centiles.
A. Coeliac disease
Note:
Coeliac disease is more occurs more frequently in Downs’s syndrome. To
screen, anti-TTG antibodies taken with serum IgA must be evaluated.

2) A 3 year old has intermittent diarrhoea with stools of varying consistency and
sometimes undigested food particles. His growth is satisfactory. His mother had
tried to reduce his intake of diary produce but dietary manipulation has had no
impact on the symptoms
E. Toddlers diarrhoea
Note:
Toddler diarrhoea (thought to be a normal variant) occurs mainly in preschool
children and is evidenced by undigested food observed in the faeces in a well
child. Poor growth would indicate a likely pathological cause.

3) A 4 month baby girl has severe atopic eczema and is regularly reviewed in the
paediatric clinic. She had now developed diarrhoea and her weight has been
static in recent weeks. A full blood count shows a slight eosinophilia.
D. Cows milk protein intolerance
Note:
CMPI is associated with eczema and eosinophilia. Cow’s milk should be
excluded from the diet and reintroduced between 1-2 years if tolerated. An
alternative milk source such as soy milk can be used until this is done.

Dr.Kamel Youssef Hassan, Pediatrician Consultant , Palestine - Gaza


E-mail: kyh10557@yahoo.com 37
MCQs PEDIATRICS

16) CHILDHOOD INFECTIONS


A. Adenovirus
B. Cytomegalovirus
C. Epstein-Barr virus
D. Escherichia coli
E. Giardia lamblia
F. Haemophilus influenzae type B
G. Listeria monocytogenes
H. Mycobacterium tuberculosis
I. Mycoplasma pneumoniae
J. Neisseria meningitidis
K. Pseudomonas aeruginosa
L. Pneumocystis carinii
M. Respiratory syncytial virus
N. Rotavirus
O. Staphylococcus aureus
P. Staphylococcus epIdermidis
Q. Streptococcus agalactiae
R. Streptococcus pneumoniae
S. Streptococcus pyogenes
T. Toxoplasma gondii
All of the infectious diseases described below occur in children. For each one,
select the most likely underlying causative agent from the list of options
1) A 3 year old boy is referred to hospital with a two day history of lethargy,
irritability and poor feeding. On examination, he is pyrexial, drowsy and has a
purpuric rash on his trunk and extremities. CSF obtained from a lumbar puncture
is cloudy and contains 540 white cells/mm3 (90% polymorphs) and 5 red blood
cells/mm 3 .
J. Neisseria meningitides

2) A 6 year old girl presents with a one week history of febrile illness with sore
throat and headache. One day prior to hospital admission, the patient awoke
with pain and swelling in the right ankle. On examination, she has a warm
swollen right ankle and a systolic heart murmur, consistent with mitral
regurgitation.
S. Streptococcus pyogenes

3) A new born infant is found to be lethargic and has a distended abdomen


immediately after birth. On examination, the infant is jaundiced and has
hepatosplenomegaly. A cranial CT scan reveals periventricular calcification.

Dr.Kamel Youssef Hassan, Pediatrician Consultant , Palestine - Gaza


E-mail: kyh10557@yahoo.com 38
MCQs PEDIATRICS

B. Cytomegalovirus

4) A 7 year old male child is referred to hospital by the general practitioner with
acute renal failure. The child had bloody diarrhoea and a low grade fever a week
ago; both resolved with rehydration.
D. Escherichia coli

5) A 4 month old female infant is brought to the hospital with severe respiratory
distress. Five days previously, she had a cough and rhinitis. On examination her
temperature is 38.9 oC, pulse 180/min and the respiratory rate 80/min. She had
subcostal retractions and nasal flaring. On auscultation, there are rhonchi and
wheezes all over her chest.
M. Respiratory syncytial virus
Comments:
1. The diagnosis of Meningococcal meningitis is clear. 2. Rheumatic fever
results from immune-mediated post Group A streptococcal infection,
Streptococcus pyogenes being a common pathogen. 3. Congenital CMV
infection is associated with petechiae, choroidoretinitis, hepatosplenomegaly,
intracerebral calcification which may lead to CNS damage with long term
sequaelae. 4. E. coli infection has resulted in Haemolytic Uraemic Syndrome
with renal failure. 5. Respiratory Syncytial Virus is the commonest cause of
lower respiratory tract infections in children worldwide, and is the leading cause
of bronchiolitis and pneumonia in children.

Dr.Kamel Youssef Hassan, Pediatrician Consultant , Palestine - Gaza


E-mail: kyh10557@yahoo.com 39
MCQs PEDIATRICS

17) VIRAL INFECTIONS


A. Measles
B. Rubella
C. Chicken Pox
D. Herpes Simplex
E. Mumps
F. Glandular fever
G. Pertussis
H. Polio myelitis
I. Hepatitis A
For each description below choose the single most likely viral infection from the
list of options
1) Caused by a gram negative pleomorphic bacillus
G. Pertussis
2) Is caused by a paramyxovirus.
E. Mumps
3) This infection is often followed by a transient immuno-deficiency.
F. Glandular fever
Comments:
Whooping cough is not uncommon in infancy. It typically presents with apnoeic
episodes or cyanotic episodes during infancy. In the older child upper respiratory
tract infections and a paroxysmal cough with a whoop is characteristic. Mumps
is caused by a paramyxovirus. Glandular fever virus infects the B lymphocytes
which results in an immuno-deficiency which is usually self limiting.

Dr.Kamel Youssef Hassan, Pediatrician Consultant , Palestine - Gaza


E-mail: kyh10557@yahoo.com 40
MCQs PEDIATRICS

18) CHILDHOOD DIARRHOEA


A. Adenovirus
B. Cytomegalovirus
C. Epstein-Barr virus
D. E. Coli
E. Giardia Lamblia
F. Hepatitis A
G. Norwalk virus
H. Rotavirus
I. Salmonella species
J. Staphylococcus aureus
Select one option from the list above that is most suitable for the following
patients
1) Causes diarrhoea by invading the brush border of the small intestine and
causes vacuolation.
I. Salmonella species
Note:
Salmonella may contaminate foods improperly foods cooked or stored and
invades the brush border of the small intestine. Symptoms include nausea,
vomiting and diarrhoea approximately between 1-8 hours after ingestion.

2) Spread is due to personal contact as well as by contaminated water supply.


E. Giardia Lamblia
Note:
Giardia Lamblia is a protozoa and may contaminate water or be transmitted by
the faecal or oral route or person to person contact. Children are often
asymptomatic however diarrhoea and cramps and weight loss may be features
of the disease.

3) An organism which is not only associated with gastroenteritis but is also the
most common cause of osteomyelitis and arthritis in children.
J. Staphylococcus aureus
Note:
Staphylococcus aureus is a gram positive coccus and the most common cause
of osteo-myelitis and arthritis in children. It may contaminate skin resulting in
impetigo, cellulitis, folliculitis and furunculosis. It may cause pneumonias and
may contaminate food resulting in enterotoxins being released in to the intestinal
tract.

Dr.Kamel Youssef Hassan, Pediatrician Consultant , Palestine - Gaza


E-mail: kyh10557@yahoo.com 41
MCQs PEDIATRICS

19) CONTRA-INDICATIONS TO VACCINES


A. BCG
B. Diphtheria Tetanus Pertussis
C. Hepatitis B
D. Haemophilus Influenzae B
E. Influenza
F. Measles Mumps Rubella
G. Polio
H. Rubella
I. Tetanus
J. Tuberculin
Which of the vaccines listed above would be contra-indicated in the following
scenarios:
1) Contra-indicated if known to be allergic to Neomycin.
F. Measles Mumps Rubella
Note:
refers to contra-indications to MMR, which include children with allergies to
Gelatin, Neomycin or kanamycin as well as children with untreated malignant
disease or altered immunity. Those receiving immuno-suppressive drugs or
radiotherapy. Children who have received another live vaccine by injection
within 3 weeks and children should not be given MMR within 3 months of an
Immunoglobulin injection.

2) Contra-indicated in HIV positive patients.


A. BCG
Note:
relates to vaccines in HIV infection. The department of health has advised that
HIV positive subjects with or with out symptoms should not receive BCG, yellow
fever or typhoid vaccinations.

3) Contra-indicated in subjects with progressive neurological conditions.


B. Diphtheria Tetanus Pertussis
Note:
relates to DTP (and the pertussis component) which should not be given to
subjects with any progressive neurological disorder particularly epilepsy and
immunisation should be delayed until the condition is stable.

Dr.Kamel Youssef Hassan, Pediatrician Consultant , Palestine - Gaza


E-mail: kyh10557@yahoo.com 42
MCQs PEDIATRICS

20) CHILDHOOD RESPIRATORY DISEASES


A. Asthma
B. Bronchiolitis
C. Croup
D. Cystic fibrosis
E. Diphtheria
F. Epiglottis
G. Pneumonia
H. Influenza
I. Retropharyngeal abscess
J. Whooping cough
For each patient below, choose the SINGLE most probable diagnosis from the
above list of options. Each option may be used once, more than once or not at
all.
1) A six month old baby presents with high fever, breathlessness, cough and
feeding difficulties. Chest examination reveals dull percussion note over the right
base posteriorly with bronchial breath sounds on auscultation.
G. Pneumonia
Note:
Pneumonia may occur at any age. Patients present with fever, tachypnoea,
feeding difficulties and cyanosis. Examination reveals bronchial breath sounds
and crepitations. Chest X ray may show consolidation. Common organisms are
pneumococcus, haemophilus, staphylococcus, mycoplasma, TB and viruses.

2) A 1 year old baby boy is wheezy, coughing, cyanosed and breathless with
intercostal recession.
B. Bronchiolitis
Note:
Acute bronchiolitis is very common in infancy. In winter epidemics of respiratory
syncytial virus infection are the commonest cause. Wheeze, cough, fever and
respiratory distress are common. Chest X ray shows hyperinflation.

3) A 4 year old non immunized boy presents with bouts of coughing ending in
vomiting. He has an absolute lymphocytosis.
J. Whooping cough
Note:
Whooping cough is caused by Bordetella pertussis infection. bouts of coughing
ending in vomiting, especially at night and after feeding suggest the diagnosis.
The characteristic whoop, forced inspiration through a closed glottis may or may

Dr.Kamel Youssef Hassan, Pediatrician Consultant , Palestine - Gaza


E-mail: kyh10557@yahoo.com 43
MCQs PEDIATRICS

not be present. Absolute lymphocytosis is common. Complications include CNS


haemorrhages, rectal prolapse and bronchiectasis.

4) A nine month old baby girl is upset and has stridor. Her voice is hoarse and
has a barking cough. She has a low grade fever.
C. Croup
Note:
Croup usually occurs in epidemics in autumn or spring. Causative viruses are
Parainfluenza (types 1, 2, or 3), respiratory syncytial viruses and measles virus.
Onset is over a few days; stridor is harsh and occurs only when child is upset. A
barking cough, harsh voice and ability to swallow secretions are typical.

5) A 2 year old boy is very unwell. His temperature is 39oC and he is unable to
swallow his secretions
F. Epiglottis
Note:
Acute epiglottis is due to Haemophilus influenza type B infection. It is
characterised by sudden onset, high fever, and continuous stridor and drooling
of secretions. Intravenous antibiotics, anaesthetic support are usually indicated.

Dr.Kamel Youssef Hassan, Pediatrician Consultant , Palestine - Gaza


E-mail: kyh10557@yahoo.com 44
MCQs PEDIATRICS

21) CONGENITAL AND NEONATAL DEFECTS DUE TO MATERNAL


INFECTIONS
A. AIDS
B. Cytomegalovirus (CMV)
C. Coxsackie group B
D. Hepatitis B
E. Herpes simplex
F. Listeriosis
G. Rubella
H. Syphilis
I. Toxoplasmosis
J. Varicella
For each patient below, choose the SINGLE most probable diagnosis from the
above list of options. Each option may be used once, more than once or not at
all.
1) A newborn baby presents with rudimentary digits, limb hypoplasia and
convulsions.
J. Varicella
Note:
Chicken pox infection within the first 20 weeks of pregnancy may result in the
congenital varicella syndrome. This is characterised by cerebral cortical and
cerebellar hypoplasia, microcephaly, convulsions, limb hypoplasia and
rudimentary digits. Prevention is by administering varicella vaccine even before
pregnancy. Varicella immunoglobulin is administered to pregnant women who
are exposed to infection. Infection during pregnancy is treated with acylovir.

2) A six week old baby is confirmed to have cataracts, cardiac abnormalities,


thrombocytopenia and cerebral calcification.
G. Rubella
Note:
This baby has congenital rubella. It occurs in children of non immunised women.
Symptoms are absent in 50% of mothers. The foetus is most vulnerable in the
first 16 weeks of pregnancy. Cataracts are associated with infections in weeks 8-
9, deafness at 5-7 weeks and cardiac lesions from 5-10 weeks. Diagnosis is
based on rising antibody titres in blood taken 10 days apart and the presence of
IGM antibodies at 4-5 weeks from incubation period.

3) A pre term neonate has multi-organ disease with granulomas on his skin. His
mother had a special liking for soft cheese during her pregnancy.
F. Listeriosis

Dr.Kamel Youssef Hassan, Pediatrician Consultant , Palestine - Gaza


E-mail: kyh10557@yahoo.com 45
MCQs PEDIATRICS

Note:
Maternal listeriosis is usually a mild infection but transplacental infection and
premature labour may occur in about 5% of cases. Avoidance of partially cooked
meats, soft cheeses and unpasteurised milk should avoid this infection.
Treatment is with ampicillin and gentamicin. Neonatal infection is usually multi-
organ and granulomas may be found on the skin and the pharynx.

4) A two week old baby has microcephaly, seizures and chorioretinitis.


I. Toxoplasmosis
Note:
Maternal and foetal toxoplasma infection may be avoided by advising pregnant
women to wear gloves when gardening or handling cat litter and to thoroughly
cook meat. Affected babies are treated with pyrimethamine, sulphadiazine and
folic acid.

5) A new born baby is very unwell with jaundice, hepatosplenomegaly and


microcephaly.
B. Cytomegalovirus (CMV)
Note:
Maternal CMV infection is usually mild and asymptomatic. 5 in 1000 live births
are affected, 5% will develop cytomegalic inclusion disease. The foetus is most
at risk in early pregnancy. There is no effective prevention.

Dr.Kamel Youssef Hassan, Pediatrician Consultant , Palestine - Gaza


E-mail: kyh10557@yahoo.com 46
MCQs PEDIATRICS

22) CHILDHOOD CHEST INFECTIONS

A. Chlamydia
B. Cytomegalovirus
C. Group B Streptococcus
D. Haemophilus Influenza
E. Mycoplasma pneumonia
F. Parainfluenza
G. Respiratory syncytial virus
H. Rhinovirus
I. Staphylococcus aureus
J. Ureaplasma
Select the most appropriate pathogen from the above list that would account for
the following presentations:
1) A 4 year old boy presents with a rapid history of high fever and drooling.
D. Haemophilus Influenza
2) A 4 year old girl presents with a 24 hour history of rhinitis, a barking cough
and hoarseness.
F. Parainfluenza
3) The commonest cause of pneumonia in the neonate.
C. Group B Streptococcus
Comments:
Item 1 describes a child with acute epiglottitis. This is a rare infection mainly
caused by Haemophilus influenza. Airway obstruction can develop rapidly due to
oedema around the epiglottis. Item 2 describe a child with croup. Acute
laryngotracheal bronchitis. It is almost exclusively viral in origin. Mainly
Parainfluenza. Streptococcus pneumonia is a common pathogen in the lung and
the commonest cause of pneumonia in the neonate.

Dr.Kamel Youssef Hassan, Pediatrician Consultant , Palestine - Gaza


E-mail: kyh10557@yahoo.com 47
MCQs PEDIATRICS

23) MUCOCUTANEOUS INFECTIONS AND INFESTATIONS


A. Enterobiasis vermicularis
B. Chlamydia
C. Candidiasis
D. Tinea pedis
E. Pityriasis rosea
F. Scabies
G. Ringworm
H. Lichen sclerosis
I. Napkin dermatitis
J. Lichen planus
For each of the following children who present with an itch, select the most
appropriate diagnosis from the list of options:
1) A child has itching in the vulva region mostly at night.
A. Enterobiasis vermicularis
Note:
Enterobiasis vermicularis is also known as threadworms. It is a common
infestation in children and present with nocturnal anal pruritus and a perianal
irritation.

2) A child presents with annular regions over the trunk.


G. Ringworm
Note:
Ringworm also known as tinea corporis presents with plaques of scaling eczema
which are characteristically itchy.
3) A diabetic child presents with a vulva rash.
C. Candidiasis
Note:
Candidiasis is caused by Candida albicans (yeast). In babies it presents as a
perineal rash which usually affects the flexures. Satellite lesions may also be
seen.

Dr.Kamel Youssef Hassan, Pediatrician Consultant , Palestine - Gaza


E-mail: kyh10557@yahoo.com 48
MCQs PEDIATRICS

24) PRESENTATION OF INFECTIOUS DISEASE IN CHILDHOOD


A. Chicken Pox
B. Herpes simplex
C. Infectious mononucleosis
D. Measles
E. Mumps
F. Mycoplasma
G. Pertussis
H. Rubella
I. Tuberculosis
For each presentation of infectious disease choose the single most likely
diagnosis from the list of options.

1) Commonly causes an acute gingivostomatitis.


B. Herpes simplex
Note:
Herpes simplex - the majority of children have benign manifestations of primary
infection with Herpes simplex, for example a gingival stomatitis. The virus is
readily spread by direct contact especially to damaged skin e.g. eczema.

2) Causing an acute parotitis


E. Mumps
Note:
Mumps infection is now uncommon due to the vaccination. It is caused by a
paramyxovirus and usually causes minimal symptoms. The most common
manifestation being an acute parotitis although severe infection with mumps
may result in meningoencephalitis plus deafness.

3) Causing an exudative tonsillitis.


C. Infectious mononucleosis
Note:
Glandular fever is also called infectious mononucleosis. It is caused by Epstein-
Barr virus and usually presents with an exudative pharyngitis or tonsillitis and
cervical lymphadenopathy. It may cause a transient impairment of cellular and
humeral immunity, which is usually self limiting.

Dr.Kamel Youssef Hassan, Pediatrician Consultant , Palestine - Gaza


E-mail: kyh10557@yahoo.com 49
MCQs PEDIATRICS

25) CHILDHOOD RESPIRATORY INFECTIONS


A. Allergic Bronchopulmonary aspergillosis
B. Aspergilloma
C. Bordetella pertussis
D. Chlamydia
E. Coxsackie B
F. Klebsiella pneumonia
G. Mycoplasma pneumonia
H. Pneumocystis carinii
I. Pseudomonas aeruginosa
J. Tuberculosis
From the list above, select the most appropriate diagnosis for the following
presentations:
1) A 5 year old boy with cystic fibrosis undergoes a routine chest x-ray which
reveals an apical round lesion on the left of his chest.
B. Aspergilloma
Note:
Aspergilloma is a fungus which may complicate conditions such as asthma and cystic fibrosis.
Children present with a cough and wheeze and there may be a positive skin test to aspergillus
with an eosinophilia and an elevated IgE. Chest x-ray may reveal an apical round mass.

2) A 5 year old girl with a high temperature presents with painful blisters on the
palms and soles of her feet
E. Coxsackie B
Note:
describes a child with hand foot and mouth, which is typically caused by Coxsackie B.
Characteristically individuals develop papules which then progress to vesicles. They are
painful and pruritic and typically affect acral areas. It is often associated with a high
temperature.

3) A 14 year old boy presents with fever, anorexia and loss of weight of 3
months duration. Scattered crepitations are heard over both lungs. Chest x-ray
is abnormal with generalised mottling.
J. Tuberculosis
Note:
TB- Mycobacterium Tuberculosis is a primary infection which may occur in the lung, gut or
skin. The local infection spreads to the surrounding lymph nodes, which constitutes the
primary complex in the lungs. Progression of the primary complex may result in
bronchopneumonia or bronchial obstruction secondary to enlargement of the lymph nodes.
Pleural effusions may also occur as well as cavitations. Primary TB may spread to the blood
stream resulting in TB. Examination may reveal hepatosplenomegaly and fundoscopy may
reveal choroid tubercles.

Dr.Kamel Youssef Hassan, Pediatrician Consultant , Palestine - Gaza


E-mail: kyh10557@yahoo.com 50
MCQs PEDIATRICS

26) TREATMENT OF INFECTIOUS DISEASE


A. Acyclovir
B. Acyclovir plus Cefotaxime
C. Ampicillin plus Gentamicin
D. Benzylpenicillin
E. Ceftriaxone
F. Ciprofloxacin
G. Flucloxacillin
H. Netilmicin
I. Symptomatic treatment
J. Teicoplanin
For each case of infectious disease described below choose the single best
treatment option from the list

1) A 7 year old child develops an exudative tonsillitis and lethargy, monospot is


positive.
I. Symptomatic treatment
Note:
Glandular fever where the treatment is symptomatic / conservative only.

2) A 1 year old child presents with neck stiffness and a purpuric rash.
D. Benzylpenicillin
Note:
A child with meningococcaemia presented to the surgery and Benzylpenicillin
needs to be given urgently prior to transfer to hospital.

3) A 4 year old child presents with encephalopathy. The EEG shows high
amplitude, abnormal waveforms. The MRI scan is also abnormal. A diagnosis of
encephalitis is made.
B. Acyclovir plus Cefotaxime
Note:
A child with encephalitis. The majority of cases are secondary to viruses
although toxic and metabolic causes need to be considered. Of the viruses
Herpes simplex is the most common agent causing encephalitis. When
encephalitis is suspected both antibiotics and Acyclovir must be commenced
immediately.

Dr.Kamel Youssef Hassan, Pediatrician Consultant , Palestine - Gaza


E-mail: kyh10557@yahoo.com 51
MCQs PEDIATRICS

27) CUTANEOUS MANIFESTATIONS OF INFECTIOUS DISEASE


A. Rheumatic fever
B. TB
C. Lyme's disease
D. Chicken pox
E. Histoplasmosis
F. Cat scratch disease
G. Measles
H. Parvovirus
I. Hepatitis B
J. Herpes simplex
For each cutaneous manifestation described below choose the single most likely
associated infectious disease.
1) Erythema infectiosum.
H. Parvovirus
Note:
Erythema infectiosum also known as Fifths disease caused by human
parvovirus, often results in a low grade temperature, slapped cheek appearance
to the face and a reticular lacy rash to the arms.

2) Gianotti-Crosti.
I. Hepatitis B
Note:
Gianotti-Crosti is a syndrome of non-pruritic erythematous papules on the face,
buttocks and extremities. Characteristically related to Hepatitis B infection
although other viruses for example EBV may be associated.

3) Koplik spots.
G. Measles
Note:
Koplik spots are white coarse granules found on the buccal mucosa opposite the
back molars and is pathognomonic for measles.

Dr.Kamel Youssef Hassan, Pediatrician Consultant , Palestine - Gaza


E-mail: kyh10557@yahoo.com 52
MCQs PEDIATRICS

28) CONGENITAL AND NEONATAL DEFECTS DUE TO MATERNAL


INFECTION
A. AIDS
B. Coxsackie group B
C. Cytomegalovirus (CMV)
D. Hepatitis B
E. Rubella
F. Herpes simplex
G. Listeriosis
H. Syphilis
I. Varicella
J. Toxoplasmosis
For each patient below, choose the SINGLE most probable diagnosis from the
above list of options. Each option may be used once, more than once or not at
all.
1) A newborn baby presents with rudimentary digits, limb hypoplasia and
convulsions.
I. Varicella

2) A six week old baby is confirmed to have cataracts, cardiac abnormalities,


thrombocytopenia and cerebral calcification.
E. Rubella

3) A pre term neonate has multi-organ disease with granulomas on his skin. His
mother had a special liking for soft cheese during her pregnancy.
G. Listeriosis

4) A two week old baby has microcephaly, seizures and chorioretinitis.


J. Toxoplasmosis

5) A new born baby is very unwell with jaundice, hepatosplenomegaly and


microcephaly.
C. Cytomegalovirus (CMV)

Comments:
Rudimentary digits and limb hypopkasia suggest congenital chicken pox infection. The
presence of cataracts, cardiac abnormalities, thrombocytopaenia and cerebral calcification
suggest a diagnosis of Congenital rubella. The presence of Choroido-retinitis strongly
suggests a diagnosis of Toxoplasmosis. The baby born with granulomas and whose mother
has a penchant for soft cheeses suggests Listeria Monocytogenes infection. Chorio-retinitis
suggests The presence of Jaundice, HSM and microcephaly suggests CMV infection.

Dr.Kamel Youssef Hassan, Pediatrician Consultant , Palestine - Gaza


E-mail: kyh10557@yahoo.com 53
MCQs PEDIATRICS

29) CHILDHOOD VIRAL INFECTIONS


A. Adenovirus
B. Coxsackie
C. Cytomegalovirus
D. Epstein barr
E. Measles
F. Molluscum contagiosum
G. Mumps
H. Rotavirus
I. Rubella
J. Varicella
For each patient with the group of symptoms listed below, choose the SINGLE
most probable causative agent from the above list of options. Each option may
be used once, more than once or not at all.
1) A two year old infant boy is admitted to hospital with vomiting, none bloody
watery diarrhoea and is dehydrated. It emerges other children from his play
group have developed a similar illness.
H. Rotavirus
Note:
Rotavirus is the most common cause of severe viral gastroenteritis worldwide.
Infection is via the faeco-oral route and often occurs in children aged between
six months to six years. This RNA virus replicates in the intestinal mucosal cells
damages transport mechanisms leading to salt and water depletion which
results in diarrhorea and vomiting. Diagnosis is made from clinical features and
culture of virus from stools and also by Polymerase chain reaction techniques.
Treatment is mainly re-hydration and correction of any electrolyte imbalance.

2) A two year old boy is mildly unwell. His mother has noticed vesicles in his
mouth, palms and soles of his feet.
B. Coxsackie
Note:
Coxsackie A16 virus is the cause of hand, foot and mouth disease characterized
by fever, sore throat and ulcerating vesicles in palms, orophaynx and on soles.
Incubation period is 5-7 days and these heal without crusting. Treatment is
symptomatic.

3) A Three-year baby girl presents with a macular confluent rash which


appeared initially behind the ears and is spreading. Over the previous five days
she has had a low grade fever, catarrh and conjunctivitis. Her mother is vague
about her immunization history.

Dr.Kamel Youssef Hassan, Pediatrician Consultant , Palestine - Gaza


E-mail: kyh10557@yahoo.com 54
MCQs PEDIATRICS

E. Measles
Note:
Measles is caused by and RNA paramyxovirus and occurs worldwide.
Outbreaks are common in areas with high numbers of non immunized children.
Infection is transmitted via respiratory droplets and incubation period is 10-21
days. The prodromal stage fever conjunctivitis, runny nose and coughing lasts
for five days. Koplik’s spots are bright red lesions with a central white dot which
appear on the buccal mucosa. These are virtually diagnostic. The typical
macular confluent rash appears on the face from day 3-5 and spreads to the rest
of the body. Diagnosis is made from clinical features, viral culture from lesions
and a grater than 4-fold rise in antibody titres. Otitis media, pneumonia,
meningitis and very rarely several years after primary infection subacute
sclerosing panencephalitis (SSPE).

4) A Ten year old girl develops an itchy rash on her body which is spreading to
her body. She traveled to see her grandmother who was unwell with a painful
rash three weeks ago.
J. Varicella
Note:
Varicella (Chicken pox) is transmitted by respiratory droplets and contact with
somebody with shingles. Incubation period is 14-21 days and following a brief
period of malaise, an itchy papulovesicular rash appears appears on trunk and
spreads to the head and the extremities. The rash evolves from papules to
vesicles, pustules and finally crusts. Antiviral therapy is reserved for systemic
disease in the immunocompromised.

5) The four month old baby daughter of an HIV positive mother is admitted to
hospital with seizures. She has neonatal jaundice and microcephaly.
C. Cytomegalovirus
Note:
Cytomegalovirus inclusion disease is the result of infection of the foetus. Many
organs may be affected and congenital abnormalities result. Microcephaly,
seizures, neonatal jaundice, hepatosplenomegaly, deafness and mental
retardation are some of the features that may occur.

Dr.Kamel Youssef Hassan, Pediatrician Consultant , Palestine - Gaza


E-mail: kyh10557@yahoo.com 55
MCQs PEDIATRICS

30) CUTANEOUS MANIFESTATIONS OF INFECTIOUS DISEASE


A. Rheumatic fever
B. TB
C. Lyme's disease
D. Chicken pox
E. Histoplasmosis
F. Cat scratch disease
G. Measles
H. Parvovirus
I. Hepatitis B
J. Herpes simplex
For each cutaneous manifestation described below choose the single most likely
associated infectious disease.
1) Erythema nodosum in presence of abnormal chest x-ray.
B. TB
Note:
Erythema nodosum may be associated with streptococcal reactions, rheumatic
fever and Tuberculosis. In the presence of an abnormal chest X-ray TB is the
most likely answer.

2) Erythema marginatum
A. Rheumatic fever
Note:
Erythema marginatum is one of the five major criteria to make a diagnosis of
rheumatic fever. It is a pink rash with pale centres and a serpiginous margin,
found on the trunk and proximal limbs.

3) Erythema chronicum migrans.


C. Lyme's disease
Note:
Erythema chronicum migrans found in Lyme's disease. A febrile illness caused
by Borrelia burgdorferi transmitted by bites of animal tic. The rash is
characterised by red margins and central clearing.

Dr.Kamel Youssef Hassan, Pediatrician Consultant , Palestine - Gaza


E-mail: kyh10557@yahoo.com 56
MCQs PEDIATRICS

6- PEDIATRICS
1) Theme: ABDOMINAL PAIN
A. Peritonitis
B. Appendicitis
C. Constipation
D. Urinary tract infection
E. Mesenteric lymphadenitis
F. Migraine
G. Intussusception
H. Henoch-Schonlein vasculitis
I. Psychosomatic
J. Shigella dysentery
For the following patients with abdominal pain, choose the most likely diagnosis.

1) A previously well 10 month old baby presenting with drawing up of her legs
with crying and some bloody runny stools.
G. Intussusception
Note:
This is the right age group for intussception. It is often preceded by an URTI and
it is thought that mesenteric lymphadenopathy may precipitate intussception. A
red currant jelly-like stool is a late sign. In the older child, the cause of
intussception may be HSP or lymphoma.
2) A 4-year-old child, with poor eating habits, iron deficiency anaemia and
intermittent colicky abdominal pain.
C. Constipation
Note:
Constipation is common in children and should be managed aggressively to
establish regular bowel habit quickly.
3) A 2-year-old girl with nephrotic syndrome who has developed a fever and
abdominal pain.
A. Peritonitis
Note:
The causes of abdominal pain in nephrotic syndrome are peritonitis (usually
pneumococcal), renal vein thrombosis and hypovolaemia.

Dr.Kamel Youssef Hassan, Pediatrician Consultant , Palestine - Gaza


E-mail: kyh10557@yahoo.com 57
MCQs PEDIATRICS

2) Theme: Skin lesions


A. Café au lait spots
B. Capillary haemangioma
C. Port-wine stain
D. Shagreen patch
E. Adenoma sebaceum
F. Cutis Marmorata
G. Herald's patch
H. Peri-oral pigmentation
I. Hypo pigmentation
J. Tuber
Select one option from the list above that is most suitable for the following
patients

1) An 11 year old boy with learning difficulties has an acne type rash over the
nose
E. Adenoma sebaceum
Note:
relates to the condition known as Tuberous sclerosis. It is a condition where by
hamartomatous lesions become evident in early childhood. Seizures and
learning difficulties are common and skin manifestations include fibrous
angiomatous lesions in the naso labial folds known as adenoma sebaceum.
White macules are seen over the trunk known as Ash leaf macules and
Shagreen patches resemble goose like flesh. Subungual fibromata are also
common.

2) A 4 year old boy with a history of focal seizures and a birth mark
C. Port-wine stain
Note:
relates to Sturge-Weber syndrome. The association and localization of aberrant
vasculature in the facial skin, eyes and meninges. Most commonly in the
trigeminal nerve distribution.

3) An infant with Down's syndrome has mottled appearance to the skin.


F. Cutis Marmorata
Note:
relates to Cutis Marmorata, a mottled, marbled type appearance to the skin.
Common in Down's syndrome as well as other Trisomies, hypothyroidism and
Cornelia-de-Lange syndrome.

Dr.Kamel Youssef Hassan, Pediatrician Consultant , Palestine - Gaza


E-mail: kyh10557@yahoo.com 58
MCQs PEDIATRICS

3) Theme: Poisoning
A. Ant psychotics
B. Benzodiazepines
C. Ethylene glycol
D. Methanol
E. Opiates
F. Organophosphates
G. Salicylates
H. Sympathomimetic
I. Tricyclic antidepressants
J. Volatile solvents

Select the most appropriate agent from the above list that would elicit the
following toxic effects:
1) Coma, pinpoint pupils, hyperventilation
E. Opiates

2) Hyper salivation, broncho-rrhoea, broncho- spasms, perspiration,


neuromuscular paralysis
F. Organophosphates

3) Tachypnoea, metabolic acidosis, Haematemesis, Reyes Syndrome.


G. Salicylates

Comments:
A pinpoint pupil suggests  opiates (Heroin – Morphine – codeine). The hyper
salivation, tachypnoea and sweating suggest Organophosphate poisoning. This
drug inhibits ant-cholinesterase activity promoting Ach effects. Treatment
consists of anticholinergics  (Atropine dose i.v 0.05 mg/kg repeated every 5-10
min as needed adverse effect tachycardia, dry mouth, blurred vision, and urinary
retention). Finally, the use of salicylates in children is well-recognized cause of
Reyes syndrome, a condition associated with fulminant hepatic failure.
Suspected poisoning in children result in thousands of attendances at A&E
department each year. Various drugs cause specific signs when taken in
overdose. Careful examination of the child should follow A, B, C protocol.

Dr.Kamel Youssef Hassan, Pediatrician Consultant , Palestine - Gaza


E-mail: kyh10557@yahoo.com 59
MCQs PEDIATRICS

4) Theme: Chemotherapy
Select the most appropriate chemotherapeutic agent from the above list that
corresponds with the following actions and side effects:

1) A drug disrupting microtubule leading to loss of deep tendon reflexes.


J. Vincristine

2) A drug inhibiting initiation of DNA synthesis resulting in conjunctivitis and


cerebellar toxicity.
G. Cytosine arabinoside

3) A drug resulting in dose related lung damage.


D. Bleomycin

Comments:
All cytotoxic agents have adverse effects. General side-effects include nausea, vomiting
and bone marrow suppression, alopecia and stomatitis. More specific side-effects are
listed in the questions above. Alkylating agents such as Cyclophosphamide may cause a
haemorrhagic cystitis. Asparaginase results in a dose related Pancreatitis.
The anthracyclines e.g. Doxorubicin and Daunorubicin are cardiotoxic, which is often
very difficult to detect. Monitoring with echocardiograms is advised. Vincristine is an
alkaloid agent and results in sensory motor neuropathy with long-term use. It also has an
affect on the autonomic system resulting in severe constipation and paralytic ileus. It
may also result in sensory changes with parathesis progressing to loss of tendon reflexes.
Bleomycin is the main drug resulting in lung damage and occurs in up to 10% of
patients. The damage is dose related.

Dr.Kamel Youssef Hassan, Pediatrician Consultant , Palestine - Gaza


E-mail: kyh10557@yahoo.com 60
MCQs PEDIATRICS

5) Theme: Childhood malignancies


A. Burkitt's nasopharyngeal carcinoma
B. Ewing sarcoma
C. Hepatoblastoma
D. Hodgkin's lymphoma
E. Melanoma
F. Nesidio-blastosis
G. Neuroblastoma
H. Teratoma
I. Wilm's tumour
J. Xeroderma pigmentosum
Select the most appropriate oncological diagnosis from the above list of options
to explain the following scenarios
1) A 13-year-old boy presents with abdominal pain. An abdominal mass is found.
This boy has a past medical history of biliary atresia.
C. Hepatoblastoma

2) A malignant transformation of a skin resulting from an autosomal recessive


condition.
J. Xeroderma pigmentosum

3) A baby on the neonatal unit is found to be hypoglycaemic.


F. Nesidio-blastosis

Comments:
Hepatoblastoma is a malignant liver tumour. It may present with anorexia,
abdominal mass and jaundice. It is associated with cirrhosis as well as biliary
cirrhosis secondary to biliary atresia. Xeroderma pigmentosa is an autosomal
recessive condition of defective DNA repair. Skin damage arise and malignant
transformation results at multiple sites. Nesidio-blastosis is also known as beta
cell endocrine benign hyperplasia and affects the pancreas. Diagnosis is made
by hypoglycaemia with high serum Insulin and treatment is by Pancreatectomy.

Dr.Kamel Youssef Hassan, Pediatrician Consultant , Palestine - Gaza


E-mail: kyh10557@yahoo.com 61
MCQs PEDIATRICS

6) Theme: SYNDROMES
A. Down's syndrome
B. Turner's syndrome
C. Noonan's syndrome
D. Klinefelter's syndrome
E. Treacher-Collins syndrome
F. Marfan's syndrome
G. Sotos syndrome
H. Klippel-feil syndrome
I. Laurence Moon Biedl syndrome
J. MCune Albright syndrome
Which is the most applicable diagnosis for the following children.
1) A 3-year-old boy with delayed motor development is reviewed regularly in the
child development clinic. He attends for routine annual blood tests to check his
thyroid function.
A. Down's syndrome
Note:
Children with Down’s syndrome are screened yearly for the development of
hypothyroidism. They should also be regularly assessed for the development of
myopia and hearing loss. These children have a predisposition to acute
lymphoblastic leukaemia (ALL), Alzheimer’s-like dementia and Atlanto-axial
subluxation.

2) A 16-year-old girl is being investigated for delayed menarche and short stature.
In her notes it is recorded that she had swollen feet at birth but she was lost to
follow-up.
B. Turner's syndrome
Note:
A karyotype should be performed in all girls presenting in this way. Pedal
lymphoedema at birth is a known presentation. Other associated features
include coarctation of the aorta, infertility, hypothyroidism and horse shoe
kidney.
3) A girl of 6 years has been found to have hypertension and coarctation of the
aorta. She has been referred to the growth clinic because of short stature.
B. Turner's syndrome
Note:
A karyotype should be performed in all girls presenting in this way. Pedal
lymphoedema at birth is a known presentation. Other associated features
include coarctation of the aorta, infertility, hypothyroidism and horse shoe
kidney.

Dr.Kamel Youssef Hassan, Pediatrician Consultant , Palestine - Gaza


E-mail: kyh10557@yahoo.com 62
MCQs PEDIATRICS

7) Theme: PRECOCIOUS PUBERTY


A. Craniopharyngioma
B. Exaggerated adrenarche
C. McCune Albright syndrome
D. Hypothalamic hamartoma
E. Thelarche variant
F. Neurofibromatosis type 1
G. Granulosa-theca cell tumour
H. Hypothyroidism
I. Premature thelarche
J. Congenital adrenal hyperplasia
Which is the most likely diagnosis in the following cases:

1) A 5-year-old girl presents with rapid breast development, breast stage 4, over
a few months and an abdominal mass. There was no pubic hair or cliteromegaly.
G. Granulosa-theca cell tumour
Note:
The rapid onset of puberty is indicative of a tumour. In this case the alpha-
fetoprotein tumour marker will be raised. Symptoms result from the very high
oestradiol levels.

2) An 8 year old boy presents with testicular volumes of 8 mls bilaterally, pubic
hair stage 3 and genital stage 3. He has long-standing constipation, deteriorating
school performance and increased weight.
H. Hypothyroidism
Note:
Long standing hypothyroidism can cause precocious puberty through high levels
of TRH driving gonadotrophin secretion and cross reactivity of the FSH receptor
with TRH.

3) A 2year old girl presents with isolated bilateral breast development.


Examination is otherwise normal as is her height velocity. Subsequent
investigation demonstrates undetectable oestradiol levels and a prepubertal
response to an LHRH test.
I. Premature thelarche
Note:
There may be a slight FSH predominance on the basal gonadotrophin levels.
Premature thelarche is self-limiting and thought to be the result of the body
being exquisitely sensitive to low levels of oestradiol.

Dr.Kamel Youssef Hassan, Pediatrician Consultant , Palestine - Gaza


E-mail: kyh10557@yahoo.com 63
MCQs PEDIATRICS

8) Theme: Drugs During Pregnancy


A. Amino glycosides
B. Aspirin
C. Beta Blockers
D. Lithium
E. Phenytoin
F. Pethidine
G. Sodium Valproate
H. Sulphonamides
I. Thiazide diuretics
J. Warfarin
Select the drug from the above list of options that is most likely to be associated
with the following effects if taken during pregnancy:
1) This drug when taken in late pregnancy may result in the baby developing
jaundice
H. Sulphonamides

2) This drug when taken in pregnancy may result in premature closure of the
ductus arteriosis.
B. Aspirin

3) Prolonged use of this drug may result in a permanent hearing impairment.


A. Aminoglycosides
Comments:
Sulphonamides taken in late gestation may compete with Bilirubin for protein binding
sites resulting in a neonatal hyperbilirubinaemia.
Aspirin has been associated with premature closure of the ductus arteriosis as well as
haemorrhagic disorders.
Aminoglycosides (such as Gentamicin) infused over a prolonged period during
pregnancy will result in ototoxicity in approximately 2-3 % of cases.

Dr.Kamel Youssef Hassan, Pediatrician Consultant , Palestine - Gaza


E-mail: kyh10557@yahoo.com 64
MCQs PEDIATRICS

9) Theme: OBESITY
A. Type I diabetes
B. Pseudohypoparathyroidism
C. Persistent hyperinsulinaemic hypoglycaemia of infancy
D. Asthma treated with Fluticasone
E. Simple obesity
F. Prader Willi Syndrome
G. Type 2 diabetes
H. Hypothalamic tumour
I. Cerebral palsy
J. Hypothyroidism
For each of the following patients, choose the most likely cause for their obesity.
1) A 13-year-old boy presents with polyuria and polydipsia. There have been no
previous concerns regarding his health and development. He has mild asthma
treated with prophylaxis in modest doses over the winter months only. On
examination he has signs of acanthosis nigricans.
G. Type 2 diabetes
2) A 3-month-old baby presents with a generalized convulsion. He has fed
voraciously since birth, consuming in excess of 250 mls/milk/kg/day. Initial
investigations demonstrate a serum calcium concentration of 2.4 mmol/L
(normal) and a serum glucose concentration of 1.4 mmol/L.
C. Persistent hyperinsulinaemic hypoglycaemia of infancy
3) A 9-year-old boy, previously well, presents with a 6-month history of
increasing behavioural disturbances, increasingly severe headache and a
convulsion.
H. Hypothalamic tumour
Comments:
1. Type 1 DM is unlikely to present with obesity but rather weight loss.
Acanthosis nigricans is indicative of insulin resistance and is found typically in
the axillae and around the nape of the neck. These findings suggest type 2 DM
i.e. insulin resistance. However polydipsia and polyuria suggest insulinopenia
and a failing pancreas which may occur in the later stages of type 2 DM. 2. PHHI
is a defect of the K+ ATP channels in the beta cell of the pancreas. There is
autonomous secretion of insulin that is independent of the serum glucose
concentration. These babies will have detectable insulin at the time of
hypoglycaemia, which would not occur normally. Treatment is with diazoxide
and chlorothiazide, some require pancreatectomy. 3. Behavioural disturbance,
headache and seizures suggest intracranial pathology. Coupled with obesity
would suggest a hypothalamic tumour as this area of the brain is involved in
appetite regulation.

Dr.Kamel Youssef Hassan, Pediatrician Consultant , Palestine - Gaza


E-mail: kyh10557@yahoo.com 65
MCQs PEDIATRICS

10) Theme: WEAKNESS IN THE LOWER LIMBS


A. Guillain-Barre syndrome
B. Transverse Myelitis
C. Juvenile Multiple sclerosis
D. Chronic fatigue syndrome
E. Spinal tumour
F. Dermatomyositis
G. Congenital myopathy
H. Duchenne muscular dystrophy
I. Fascioscapulohumeral dystrophy
J. Spinal abscess
For these children with lower limb weakness select the most likely diagnosis.
1) A 15-year-old boy started walking at 18 months of age. He has mild ptosis,
absence of facial expression and neck weakness. His mother has similar
symptoms
I. Fascio-scapulo-humeral dystrophy
Note:
This is a description of fascio-scapular-humeral dystrophy, which typically presents in the
teenage years. The family history does not suggest an X-linked disorder rather autosomal
dominant.
2) A 6-year-old child presents with acute onset symmetrical flaccid paralysis
initially affecting the legs then involving the trunk, and now some mild weakness
in the arms. 2 weeks prior to this he had been unwell with URTI and fever. On
examination there is minimal movement with gravity eliminated in all muscle
groups in the lower limbs, deep tendon reflexes are absent, plantar responses
are up-going. Distal sensory losses are demonstrable. The CSF shows raised
protein but no pleocytosis.
A. Guillain-Barre syndrome
Note:
The description is of Guillain-Barre syndrome, which may be preceded by an URTI. There are
lower motor neurone signs but also with absent reflexes due to progressive distal
demyelination. Sometimes there is cranial nerve involvement (Miller Fisher syndrome)
3) A 9-year-old boy has a gradual onset of proximal weakness in the pelvic
girdle and to a lesser extent in the shoulder girdle also. There is a faint
violaceous rash over the eyelids and some telangiecasia over the finger
knuckles. He has a low-grade fever and the thigh and buttock muscles are
notably tender on examination.
F. Dermatomyositis
Note:
Proximal myopathy, heliotrope rash over eyelids and the description of Gotrens
papules over the knuckles indicate dermatomyositis.
Dr.Kamel Youssef Hassan, Pediatrician Consultant , Palestine - Gaza
E-mail: kyh10557@yahoo.com 66
MCQs PEDIATRICS

11) Theme: DELAYED MILESTONES


A. 4 weeks
B. 2 months
C. 7 months
D. 10 months
E. 12 months
F. 18 months
G. 2 years
H. 3 years
I. 5 years
J. 6 years
At what age would you refer a child for investigation if they were unable to do the
following:-
1) Sit unsupported
D. 10 months
2) Walk independently
F. 18 months
3) Smile
B. 2 months

Comments:

It is important to be familiar with the developmental milestones, particularly up to


the age of 3 years. Milestones are grouped under 4 headings:

 Gross motor
 Fine motor
 Hearing and language
 Social

You should know the age range for each milestone. Developmental milestones
are in Illingworth: 'The Normal Child'.

Dr.Kamel Youssef Hassan, Pediatrician Consultant , Palestine - Gaza


E-mail: kyh10557@yahoo.com 67
MCQs PEDIATRICS

12) Theme: Renal disease


A. Acute nephritis
B. Nephrotic syndrome
C. Diabetes insipidus
D. Urinary tract infection
E. Chronic renal failure
F. Diabetes mellitus
G. Salt poisoning
H. Haemolytic uraemic syndrome
I. Barrter syndrome
J. Cystinosis
Which diagnosis is most appropriate for the following patients?
1) A 2-year-old girl has become puffy in her face and has developed swollen
feet. Her urine dipstick shows proteinuria.
B. Nephrotic syndrome
Note:
Hypoalbuminaemia (serum albumin falls below 25g/L); oedema, proteinuria
(exceed 2g/24 hr. or more than 40 mg/m2/hr.)(In urine dipstick +3 – +4 for
proteinuria) and hypercholestrolaemia (LDL , triglycerides) are features of
nephrotic syndrome. So are peritonitis (pneumococcal peritonitis due to low
immunoglobulins), renal vein thrombosis and hypotension (due to hypovolaemia
& hypercoagulable state with low antthrombin III) .

2) A 15 years old boy has felt tired, lost his appetite and has not grown in the last
year. His serum urea is 20mmol/l and serum creatinine is 575µmol/l. His Hb is
9.6g/dl.
E. Chronic renal failure
Note:
Chronic renal failure may have an insidious onset and may present with slowed
poor growth and delayed puberty.

3) A 2 week old boy presents to casualty with weight loss and inconsolable
crying. His serum sodium is 150mmol/l and his urine osmolality is 100 mosm/kg
(normal > 600).
C. Diabetes insipidus
Note:
Hypernatraemia with concomitant low urine osmolality suggest DI. Remember it
is important to establish whether nephrogenic in origin (treatment indomethacin
or chlorthiazide – thiazides – carbamzepine - chloramphenicol) or cranial
(treatment DDAVP = exogenous ADH = Desmopressin = ADH analogue).

Dr.Kamel Youssef Hassan, Pediatrician Consultant , Palestine - Gaza


E-mail: kyh10557@yahoo.com 68
MCQs PEDIATRICS

13) Theme : Neonatal Jaundice


A. ABO incompatibility
B. Breast milk jaundice
C. Cephalhaematoma
D. Congenital biliary atresia
E. Congenital cytomegalovirus infection
F. Congenital haemolytic anaemia
G. Congenital spherocytosis
H. Galactosaemia
I. Glucose-6-phosphate dehydrogenase deficiency
J. Haemolytic disease of the newborn
K. Hypothyroidism
L. Preterm birth
M. Sepsis

A 6 day old infant is admitted with jaundice (serum bilirubin 300 micromol/l) and
feeding poorly for the past 48 hours. The infant was born at a gestational age of
36 weeks and weighed 2680g at birth. The following are some of the clinical
findings and results of investigations. In each case, select the most likely cause
of the jaundice from the list of options.

1) The infant was born after a difficult forceps delivery and the head appeared to
be swollen in the right parieto-occipital area. Jaundice developed after 24 hours.
The haemoglobin is 12 g/dl; there is indirect hyperbilirubinaemia.
C. Cephalhaematoma

2) The baby has a temperature of 38 oC and tends to bring up its feeds. The
circulating white cell count is 18x109 /l. Microscopy of urine from a suprapubic
aspirate shows large numbers of neutrophil polymorphs.
M. Sepsis

3) The liver and spleen are palpable and there is a family history of lethargy,
hepatosplenomegaly and poor feeding. The raised bilirubin is initially
unconjugated; in the second sample, the conjugated fraction tended to rise.
Urine examination is positive for reducing substances using Clinitest.
H. Galactosaemia

4) On initial assessment, petechiae and hepatosplenomegaly are noted, and on


investigation, thrombocytopenia and partially conjugated hyperbilirubinaemia are

Dr.Kamel Youssef Hassan, Pediatrician Consultant , Palestine - Gaza


E-mail: kyh10557@yahoo.com 69
MCQs PEDIATRICS

found. Chorioretinitis is noted later and intra-cranial calcification is reported on a


CT brain scan.
E. Congenital cytomegalovirus infection

5) The baby was mostly breast fed and 20% of the bilirubin is conjugated.
Formula milk feeds caused a rapid fall in bilirubin levels.
B. Breast milk jaundice

Comments:
1. A cephalhaematoma is a subperiosteal swelling which can arise as a result of
forceps delivery. Complications result from anaemia due to bleeding into the
haematoma, and jaundice due to haemolysis within it
2. Sepsis likely due to urinary tract infection.
3. Galactosaemia is an autosomal recessive condition due to the absence of
galactose-1-phosphate uridyl transferase. This results in the intracellular
accumulation of galactose-1-phosphate which is toxic. Upon commencement of
milk feeds the child develops jaundice, vomiting, diarrhoea and failure to thrive.
4. Congenital Cytomegalovirus infection is typically characterized by the signs
described. CNS damage from congenital CMV infection exceeds that due to any
other infectious cause.
5. Breast milk jaundice is defined as  neonatal jaundice persisting for more than
28 days, with a total bilirubin level greater than 5.9mg/dl, in an otherwise
healthy, thriving, breast-fed infant.

Dr.Kamel Youssef Hassan, Pediatrician Consultant , Palestine - Gaza


E-mail: kyh10557@yahoo.com 70
MCQs PEDIATRICS

14) Theme : Connective tissue disease


A. Ankylosing spondylitis
B. Dermatomyositis
C. Ehlers Danlos syndrome
D. Henoch Schonlein purpura
E. Juvenile chronic arthritis
F. Kawasaki disease
G. Osteogenesis imperfecta
H. Rheumatic fever
I. Scleroderma
J. Systemic lupus erythematosus
Select one of the above diagnoses that would explain the following
presentations:
1) A 10 year old boy presents with arthralgia, an urticarial rash on the lower
limbs and buttocks and abdominal pain.
D. Henoch Schonlein purpura
Note:
Henoch Schonlein purpura also known as anaphylactoid purpura result in non-
thrombocytopenic = (normal plat. Count) purpura nodules, arthritis, abdominal
pain and nephritis.
2) A six year old girl presents with fever, arthritis and a photosensitive rash.
Dipstick of her urine reveals proteinuria.
J. Systemic lupus erythematosus
Note:
describes a child with SLE. A multi system disorder which results in fever,
malaise, arthritis, rashes typically photosensitive rash and a butterfly rash.
Kidneys are also affected resulting in glomerulonephritis. Diagnosis is made by
elevated  anti-nuclear antibody, Double stranded DNA with muscle antibodies.

3) A 4 year old boy is admitted to casualty with fracture of her ulna after minor
injury. She is noted to have blue sclerae and hearing impairment.
G. Osteogenesis imperfecta
Note:
Osteogenesis imperfecta, a syndrome of fragile bones due to defect in the α-chain of type 1collagen, which can
be divided in to 4 separate types. There is widespread osteopaenia resulting in bone fragility, bow legs.
Individuals characteristically have blue sclerae and hearing impairment (conductive deafness). The condition has
variable inheritance with autosomal dominant and recessive forms, the latter being more severe.
Blue sclera – differential diagnosis
1) Marfan syndrome
2) Ehler-Danlos syndrome
3) Osteogenesis imperficta
4)Pseudo-xanthoma elasticum

Dr.Kamel Youssef Hassan, Pediatrician Consultant , Palestine - Gaza


E-mail: kyh10557@yahoo.com 71
MCQs PEDIATRICS

15) Theme : NEPHROLOGY


A. Acute nephritis
B. Nephrotic syndrome
C. Diabetes insipidus
D. Urinary tract infection
E. Chronic renal failure
F. Diabetes mellitus
G. Salt poisoning
H. Haemolytic uraemic syndrome
I. Barrter syndrome
J. Cystinosis
Which diagnosis is most appropriate for the following patients?
1) A 4 1/2 year old girl was admitted to hospital with a 10 day history of
progressive shortness of breath and puffiness of the face. Clinical examination
revealed a raised JVP, pulse rate of 160/min, normal heart sounds,
hepatomegaly 2cm and bilateral basal lung crepitations. She was seen by her
family doctor for a sore throat 2 weeks previously.
A. Acute nephritis
Note:
The picture is one of fluid overload and with a preceding URTI making post-
streptococcal glomerulonephritis the most likely diagnosis. There is often
macroscopic haematuria and hypertension.
2) A 4 month baby boy is admitted because of inconsolable crying. He had been
thriving otherwise and was well previously. He seems settled during
assessment, feeds well and is afebrile. Urinalysis reveals white cells 45/HPF,
red cells 10/hpf, organisms >100,000/ml.
D. Urinary tract infection
Note:
UTI should be ruled out in fractious pyrexial infants as the symptoms are non-specific. It is
important to document whether there has been previous recent antibiotic usage and where
possible a clean catch urine sample should be taken prior to starting antibiotic in this age
group. Pyuria alone is not sufficient to make the diagnosis of UTI.
3) A 9 year old girl has chronic renal failure of unknown aetiology. She has blond
hair and blue eyes and has developed hypothyroidism. She also has cataracts
that are worsening over the last 2 years.
J. Cystinosis
Note:
This is a description of cystinosis which is confirmed by having high white cell
cystine levels. Deposits of cystine may occur in the thyroid and lense. Children
typically have ‫ أﺷﻘر‬blond hair and blue eyes.

Dr.Kamel Youssef Hassan, Pediatrician Consultant , Palestine - Gaza


E-mail: kyh10557@yahoo.com 72
MCQs PEDIATRICS

16) Theme : Personal and Social Development

A. 3 months
B. 8 months
C. 12 months
D. 18 months
E. 2 years
F. 3 years
G. 4 years
H. 5 years
I. 6 years
J. 7 years

For each description of a child's personal and social development choose, from
the list of options, the highest development age the child is most likely to have
achieved.
1) The baby is reliably reacting to sounds and is able to comply with a formal
hearing test.
B. 8 month

2) A child who is starting to seek adequate role models and assumes feelings
and habit of thought.
G. 4 years

3) Is friendly to strangers but is not yet ready to play games such as peek-a-boo.
B. 8 months

Comments:
This question relates to the personal and social aspects of normal development.
1) Between 8- 10 months babies become anxious when separated from the main
carer, usually the mother for example if mother leaves the room.
2) Between 9-10 months the infant is able to wave bye bye and enjoys games
such as peek-a-boo.
3) Between 18-24 months most children start to express their toilet needs.
4) Up until the age of 3 play is usually solitary, playing alongside their peers.
5) By 4 years the children usually start to role play for example pretending to be
the same sex parent and play involves other children in parallel first and then
interactively at around this age.

Dr.Kamel Youssef Hassan, Pediatrician Consultant , Palestine - Gaza


E-mail: kyh10557@yahoo.com 73
MCQs PEDIATRICS

17) Theme : Emergency treatment


A. 100% Oxygen plus nebulised Salbutamol
B. 100% oxygen, adrenaline and hydrocortisone
C. Adrenaline IV
D. Dexamethasone
E. Hyperbaric oxygen
F. Intubation
G. IV Hydrocortisone plus oral anti-Histamines
H. Paralysis with Sodium Thiopentone
I. Skin desensitisation
J. Supportive treatment
Select the most appropriate emergency treatment for the following cases:
1) A 13 year old boy is brought to A&E with a grossly swollen face and difficulty
breathing following a bee sting. ‫ﻟﺳﻌﺔ ﻧﺣﻠﺔ‬
B. 100% oxygen, adrenaline and hydrocortisone

2) A 2 year old boy presents with a barking cough and stridor.


D. Dexamethasone

3) A 4 month old baby with cough fever and mild respiratory distress has
widespread crackles on auscultation.
J. Supportive treatment

Comments:
1) describes an anaphylactic reaction also known as angioneurotic oedema. In
this case there is difficulty breathing i.e. life-threatening. Treatment includes
100% oxygen, IM adrenaline and IV Hydrocortisone. Intubation may be
necessary.
2) The second case has croup as characterized by the barking cough.
Humidified oxygen can help for mild cases but more severe cases require
dexamethasone.
3) The principles of management for bronchiolitis includes supportive treatment
only. There is no specific treatment other than humidified oxygen; broncho
dilator therapy may be of some use

Dr.Kamel Youssef Hassan, Pediatrician Consultant , Palestine - Gaza


E-mail: kyh10557@yahoo.com 74
MCQs PEDIATRICS

18) Theme : Fine Motor Development


A. 3 months
B. 6 months
C. 9 months
D. 12 months
E. 18 months
F. 2 years
G. 3 years
H. 4 years
I. 5 years
J. 6 years
For each description of a child's fine motor development choose, from the list of
options, the highest development age the child is most likely to have achieved.
1) Can copy a circle and a cross and climbs and descends stairs, one foot per
step.
H. 4 years
2) A child can scribble and draw a straight line but not yet a circle.
F. 2 years
3) Spontaneously scribbles ‫ ﺷﺧﺑطﺔ‬with a crayon ‫ طﺑﺎﺷﻳر‬and is starting to build a
tower.
E. 18 months
Comments:
This question relates to the fine motor area of development.
1) At 4 months the infant is able to grasp big objects and move them in to the mid-line.
2) At 6 months the baby can grasp an object such a rattle and starts to transfer it from hand to
hand. He or she also starts to discover the rest of his or her body parts.
3) The pincer grip develops between 6 or 9 months and by 10 months the fine pincer grip is
achieved with the baby being able to pick up small pellet like objects between thumb and
forefinger.
4) At 18 months the baby starts to spontaneously scribble if offered a crayon for example
5) At 24 months the child is able to copy a straight line.
6) By age 3 years the child can copy a circle.
7) By 4 years can copy cross.
8) By 4½ years can copy a square.
9) By 5 years of age can copy a triangle
10) A diamond at about 6 years of age.
11) At the age of 3 the child starts to draw people usually starting as a “head and stick”
extremity person and as development progresses the person becomes more sophisticated.

Dr.Kamel Youssef Hassan, Pediatrician Consultant , Palestine - Gaza


E-mail: kyh10557@yahoo.com 75
MCQs PEDIATRICS

19) Theme : Causes of vomiting


A. Cyclical vomiting
B. Duodenal atresia
C. Over feeding
D. Meckel's diverticulum
E. Ulcerative colitis
F. Meconium ileus
G. Gastro-enteritis
H. Gastro-oesophageal reflux
I. Pyloric stenosis
J. Urinary tract infection
Select one option from the list above that is most suitable for the following
patients
1) A 2 day old breast fed baby is vomiting after each feed. Abdominal x-ray
demonstrates a double bubble.
B. Duodenal atresia
Note:
Duodenal atresia is the most common cause of duodenal obstruction in infancy.
It is frequently found in babies with Down's syndrome. Babies present with
bilious or non-bilious vomiting and a “double bubble” are seen on abdominal x-
ray. This is the result of distension of the stomach and duodenum with a
constricting pylorus between them.

2) A one year old boy with a history of intermittent bleeding pr presents with
features of an interception
D. Meckel's diverticulum
Note:
Meckel's diverticulum is most common under the age of 2. The Meckel's
diverticulum is remnant of the vitilo-intestinal duct and can present with features
of intussusception or diverticulitis. Commonly presents with painless bleeding
and is diagnosed by a Meckel's scan.

3) A 4 month old baby who is thriving has persistent vomiting which is


occasionally blood stained.
H. Gastro-oesophageal reflux
Note:
Gastro-oesophageal reflux. Reflux of gastric contents in to the oesophagus is a
common event in infancy. Once the abdominal segment of the oesophagus
lengthens in the first few months of life reflux improves.

Dr.Kamel Youssef Hassan, Pediatrician Consultant , Palestine - Gaza


E-mail: kyh10557@yahoo.com 76
MCQs PEDIATRICS

20) Theme : Drugs During Pregnancy


A. Aminoglycosides
B. Aspirin
C. Beta Blockers
D. Lithium
E. Phenytoin
F. Pethidine
G. Sodium Valproate
H. Sulphonamides
I. Thiazide diuretics
J. Warfarin
Select the drug from the above list of options that is most likely to be associated
with the following effects if taken during pregnancy:
1) This drug taken in early pregnancy may result in Ebstein's anomaly in 3% of
cases.
D. Lithium

2) This drug may result in a neural tube defect in approximately 2% of cases.


G. Sodium Valproate

3) This drug, when taken in pregnancy may result in a baby being born with
cranio-facial abnormalities, growth impairment and learning difficulties.
E. Phenytoin
Comments:
1) Lithium, a drug used in the treatment of bipolar affective disorder. If given in early
pregnancy is associated with a wide variety of cardiac defects with approximately 8%
having severe cardiac disease, Ebstein's anomaly being the most common (3% of all
cases).
2) Sodium Valproate tends to induce a neural tube defect (spina bifida) in approximately
2% of pregnancies and therefore should be avoided. It is also associated with
abnormalities of the orofacial and digits, compromising the foetal Valproate syndrome.
3) Phenytoin is well known for causing the foetal hydantoin syndrome, compromising
cranio-facial abnormalities and learning difficulties as well growth impairment.

Dr.Kamel Youssef Hassan, Pediatrician Consultant , Palestine - Gaza


E-mail: kyh10557@yahoo.com 77
MCQs PEDIATRICS

21) Theme : Abdominal Pain


A. Acute gastro-enteritis
B. Appendicitis
C. Constipation
D. Diabetes Mellitus
E. Henoch Schonlein Purpura
F. Mesenteric Adenitis
G. Nephroblastoma
H. Pancreatitis
I. Sickle Cell crisis
J. Wilm's Tumour
Select one option from the list above that is most suitable for the following
patients
1) A 12 year old boy with a 3 week history of coryza is brought to A&E
complaining of severe abdominal pain. He has swollen ankles and nodules on
his buttocks, which do not disappear with pressure.
E. Henoch Schonlein Purpura
Note:
Henoch Schonlein purpura also known as anaphylactoid purpura has an unknown aetiology.
Children present with abdominal pain, possibly melaena due to haemorrhage and oedema of
the gut wall. They may have a flitting arthritis affecting large joints. The nephritis presents with
microscopic haematuria and proteinuria and the macular, papular, purple rash is rather
characteristic with buttocks and extensive surfaces of the legs and arms being affected.
2) A 10 year old girl has just returned from Thailand. She is brought to history of
severe diarrhoea associated with abdominal pain. Her mother, a known diabetic,
has similar symptoms.
A. Acute gastro-enteritis
Note:
relates to a child with gastroenteritis. In the developed world gastroenteritis is very common
and usually mild. 60% of cases are due to viruses for example Rotavirus. In bacterial
gastroenteritis; fever is common and children present with colicky abdominal pain, vomiting,
diarrhoea and possibly dehydration.
3) A 12 year old girl with a 2 month history of weight loss and increased appetite
weight loss is brought to Casualty with severe abdominal pain. She denies any
history of vomiting or excessive exercise. Her mother says that the child has
been wetting her bed over the past 4 months.
D. Diabetes Mellitus
Note:
describes a child with diabetes mellitus. The commonest metabolic / endocrine problem in
childhood. Diabetes mellitus results from low Insulin levels resulting in abnormal metabolism
of carbohydrate, protein and fats. Children present with polyuria, polydipsia and polyphagia.
Children sometimes complain of abdominal discomfort and there is a history of weight loss.

Dr.Kamel Youssef Hassan, Pediatrician Consultant , Palestine - Gaza


E-mail: kyh10557@yahoo.com 78
MCQs PEDIATRICS

22) Theme : Causes of Headaches


A. Benign intra-cranial hypertension
B. Cerebral tumour
C. Cluster headache
D. Encephalitis
E. Extradural haematoma
F. Meningitis
G. Migraine
H. Subdural haemorrhage
I. Sub arachnoid haemorrhage
J. Tension headache
Select the most appropriate diagnosis from the above list that would explain the
following cases.
1) A 16 year old girl develops headache and vomiting. She has a fever and
develops a non-blanching rash.
F. Meningitis
2) A 17 year old male with a 4 day history of cough and fever presents with
altered sensorium and vomiting.
D. Encephalitis
3) A 10 year old girl with a 6 month history of episodic abdominal pain presents
with a severe headache, a complete ophthalmoplegia which lasts several days.
G. Migraine
Comments:
1) Bacillus Meningitis is a severe infection that carried serious morbidity and
mortality. It may present suddenly or insidiously. Most commonly symptoms start
off as being non-specific, for example fever, anorexia, irritability and later
develop in to a rash and shock. More specific signs include neck stiffness and
seizures.
2) Encephalitis refers to inflammation of the brain and children present with an
encephalopathy. Initial symptoms may be non-specific or flu-like followed by
changes in mental state, seizures and complications include cerebral oedema,
cardiovascular instability, fluid and electrolyte disturbances. Herpes viruses have
been implicated as well as other viruses and often there is a devastating
consequence.
3) Migraine is a recurrent headache with symptom free interval. Other symptoms
include abdominal pain, nausea or vomiting, aura, visual sensory or motor
(photophobia may also occur). Other visual disturbances may take the form of
blurred vision, fortification spectra and flashing lights. Complicated migraine
occurs when there are neurological signs during a headache that may persist
after the headache has terminated. For example hemiplegic migraine.

Dr.Kamel Youssef Hassan, Pediatrician Consultant , Palestine - Gaza


E-mail: kyh10557@yahoo.com 79
MCQs PEDIATRICS

23) Theme : Dyspnoea in children


A. Acute severe asthma
B. Cardiac failure
C. Costochondritis
D. Diabetic ketoacidosis
E. Hysterical over-breathing
F. Inhalation of foreign body
G. Pleura effusion
H. Pulmonary embolism
I. Tension pneumothorax
J. Pleural effusion
Select the most appropriate diagnosis from the options above that best explains
the following cases:
1) A 3 month old baby presents with tachycardia and tachypnoea with
widespread crepitations
B. Cardiac failure
2) A 3 year old presents with cough and haemoptysis. Examination reveals
reduced breath sounds over one lung and a localized wheeze.
F. Inhalation of foreign body
3) A boy presents with a dry cough, lethargy and shortness of breath.
Examination reveals reduced breath sounds over one side which is dull to
percussion.
J. Pleural effusion

Comments:
Heart failure – A number of congenital heart lesions will result in cardiac failure,
commonly in the first year of life. The baby becomes breathless particularly after
crying or feeding. The baby may be having difficulty completing feeds and as a
result there may be a problem with poor weight gain. Babies also are seen to
have sweating as a prominent symptom. On examination the baby may be
tachycardic and tachypnoenic. A murmur may be present or a gallop rhythm.
Oedema may be seen around the dependant parts of the body. Pulmonary
crepitations may also be heard. It may be possible to pick up heart enlargement
clinically and chest x-ray may reveal cardiomegaly and plethoric lung fields. In
the second case the unilateral nature of the reduced breath sounds and wheeze
strongly suggest inspissation of a foreign body. The final case has again
unilateral signs but on this occasion the dullness to percussion suggest a pleural
effusion.

Dr.Kamel Youssef Hassan, Pediatrician Consultant , Palestine - Gaza


E-mail: kyh10557@yahoo.com 80
MCQs PEDIATRICS

24) Theme : Abdominal diseases of Childhood

A. Aerophagia
B. Alpha 1 antitrypsin deficiency
C. Galactosaemia
D. Hirschsprung's disease
E. Intussusception
F. Lactose intolerance
G. Mesenteric adenitis
H. Reyes syndrome
I. Thalassaemia
J. Wilson's disease
Select one of the above options that is the most appropriate diagnosis for the
following cases:
1) Usually presents with chronic liver disease often in association with
neurological features.
J. Wilson's disease
Note:
describes a child with Wilson's disease also known as hepatolenticular
degeneration. It is an autosomal recessive condition characterized by cirrhosis
of the liver, degenerative changes of the brain and Kayser-Fleischer rings on the
cornea. It is as a result of low Caeruloplasmin and low Copper excretion.
2) A 9 month old baby presents with a sudden onset of abdominal pain and
loose stools which have the appearance of red currant jelly.
E. Intussusception
Note:
Intussusception is an invagination of the bowel in to an adjacent segment. It
presents as paroxysmal pain occasionally with vomiting and palpation may
reveal a sausage shaped mass  in the right upper abdomen. A delay in
diagnosis results in the child passing blood stained mucus rectally some times
known as red currant jelly. Intussusception can sometimes be reduced hydro-
statically, if unsuccessful surgery is required.

3) A 4 day old infant presents with severe abdominal distension and feculent
vomiting
D. Hirschsprung's disease
Note:
Hirschsprung's disease is absence of ganglia cells in part or all of the wall of the
colon. The child presents with bilious or feculent vomiting, constipation and
distension.

Dr.Kamel Youssef Hassan, Pediatrician Consultant , Palestine - Gaza


E-mail: kyh10557@yahoo.com 81
MCQs PEDIATRICS

25) Theme : Gross motor development


A. 1 month
B. 2 months
C. 4 months
D. 6 months
E. 8 months
F. 10 months
G. 1 year
H. 18 months
I. 2 years
J. 3 years
For each description of a child's gross motor development choose, from the list
of options, the highest development age the child is most likely to have
achieved.
1) A baby can crawl, weight bare and is starting to cruise.
F. 10 months
2) A baby rolls from front to back and back to front however is not yet sitting
unsupported.
D. 6 months
3) A child ascends stairs using alternating feet and is able to stand on one foot
for a few seconds.
J. 3 years
Comments:
This question relates to gross motor skills area of development.
1. At 3 months when placed prone an infant can lift his head with his arms extended.
2. By 4 months head lag is lost and this is evident when raised from supine to the sitting
position.
3. At 5 months the baby starts to roll over and
4. At 6 months the baby is starting to sit up with support for example cushions.
5. Sitting unsupported is usually seen by 6-7 months
6. At 9 months the baby can start to crawl.
7. Cruising is usually achieved by about 10 months.
8. At 12 months the baby can raise ‫ ﻳﻘوم‬independently and often starts to take a few steps
alone.
9. At 18 months the toddler can run although this is usually in a tiff fashion. He or she can
ascend stairs with assistance
10. At 20 months the child can climb the stairs holding on to a rail.
At 3 years the child can ascend stairs in an adult fashion using alternating feet per step
however is unable to descend the stairs in such a fashion until 4 years of age.

Dr.Kamel Youssef Hassan, Pediatrician Consultant , Palestine - Gaza


E-mail: kyh10557@yahoo.com 82
MCQs PEDIATRICS

26) Theme : Treatment of Diarrhoeal illness


A. Metronidazole
B. Gluten free diet
C. No action
D. Steroids
E. Lactose free diet
F. Pancreatic enzyme supplements
G. Vitamin D
H. High fibre diet
I. Breast milk allergy
J. Praziquantel

Select one option from the list above that is most suitable for the following
patients
1) A 2 year old girl is taken to the A&E department by her mother who says that
her child has diarrhoea and describes the stool as looking like peas and carrots.
C. No action
Note:
Describes a child with toddler diarrhoea due to a fast gut transit time. Children
between the ages of 6 months and 3 years may have this complaint which
manifests as a chronic diarrhoea. Children are otherwise healthy and thriving
with all investigations carried out being normal.
2) A 7 year old child presents with a 3 month history of abdominal pain. She is
found to have anaemia and colonoscopy reveals a cobble stone appearance.
D. Steroids
Note:
Relates to inflammatory bowel disease, Crohn's disease and ulcerative colitis
respectively. In Crohn's disease the colonoscopy will reveal cobbled stone
mucosal appearances with fistulae where as in ulcerative colitis  (crypt
abscesses + Collar button ulcers)are present.
3) A 12 year old girl whose height and weight are on the 3rd centile complains of
diarrhoea with a history that the stools are difficult to flush away. She has an
itchy rash over her limbs.
B. Gluten free diet
Note:
relates to Coeliac disease. A sensitivity and intolerance to the gliadin protein  in
rice, wheat and barley. Children present with failure to thrive, diarrhoea,
abdominal pain, anorexia and abdominal distension. Diagnosis is suspected by
elevated (anti-gliadin / anti-reticulin / anti-endomysial) antibodies and confirmed
by jejunal biopsies.

Dr.Kamel Youssef Hassan, Pediatrician Consultant , Palestine - Gaza


E-mail: kyh10557@yahoo.com 83
MCQs PEDIATRICS

27) Theme : TREATMENT OF CHILDHOOD CONDITIONS


A. Acyclovir
B. Cefotaxime
C. Erythromycin
D. Gentamicin
E. Immunoglobulin
F. No medication
G. Paracetomol
H. Penicillin
I. Prednisolone
J. Quinine

All of the following children present with a febrile illness. Based on the
symptoms presented below, select the single treatment option most likely to
alter the clinical course of the disease in each case. Each option may be used
once, more than once, or not at all

1) A playful and active 5 year old boy presents with a low grade fever and
bruising. The rest of the examination is normal. His platelet count is 14 x 109 /L
F. No medication
Note:
Childhood thrombocytopenia may occur in the setting of viral illness.

2) A 10 year old girl comes back from school camp with a temperature,
breathlessness and cough, which has been going on for 10 days
C. Erythromycin
Note:
Outbreaks of whooping cough may in school camps. The most vulnerable
children are not immunized.

3) A 5 month old baby presents with a fever, lethargy, poor feeding and a
generalized convulsion. An examination of the cerebrospinal fluid shows an
elevated number of neutrophil polymorphs
B. Cefotaxime
Note:
Meningitis should be suspected in any ill baby and may present with unusual
crying, poor feeding, fever, coma, fits and vomiting. Bulging or tense anterior
fontanelle, stiff neck, opisthotonus and shock may be present. For children more
than three months old cefotaxime is the treatment of choice.

Dr.Kamel Youssef Hassan, Pediatrician Consultant , Palestine - Gaza


E-mail: kyh10557@yahoo.com 84
MCQs PEDIATRICS

4) A 3 year old Asian girl presents with a high temperature for six days, cervical
lymphadenopathy, conjunctivitis and a red tongue. There is a generalized rash,
with desquamation of the skin of the fingers and toes.
E. Immunoglobulin
Note:
Kawasakis disease is characterized by bilateral conjunctival injection, oral
mucosal changes with red sore tongue, polymorphous rash, lymphadenopathy
and desquamation of palms and soles. Inflammatory markers are elevated and
treatment is with intravenous immunoglobulin.

5) A 4 year old girl presents with a temperature of 39oC having just had a
generalized convulsion lasting 3 minutes; twenty minutes later, she is completely
alert again. Her older brother had similar fits at the age of 3 years
G. Paracetomol
Note:
Febrile convulsions are generalized tonic/clonic symmetrical seizures which
occur with rapid rises in temperature in children between the ages 3 months and
5 years who have no history of epilepsy or CNS disease.

Dr.Kamel Youssef Hassan, Pediatrician Consultant , Palestine - Gaza


E-mail: kyh10557@yahoo.com 85
MCQs PEDIATRICS

28) Theme : Fine Motor Development


A. 3 months
B. 6 months
C. 9 months
D. 12 months
E. 18 months
F. 2 years
G. 3 years
H. 4 years
I. 5 years
J. 6 years
For each description of a child's fine motor development choose, from the list of
options, the highest development age the child is most likely to have achieved.

1) A child can copy a square and a triangle but not yet a diamond.
I. 5 years
2) A baby transfers objects to his mouth and has a fine pincer grip using thumb
and forefinger.
D. 12 months
3) Can only copy a circle and builds a tower of 9 cubes.
G. 3 years
Comments:
This question relates to the fine motor area of development.
1) At 4 months the infant is able to grasp big objects and move them in to the
mid-line.
2) At 6 months the baby can grasp an object such a rattle and starts to transfer it
from hand to hand. He or she also starts to discover the rest of his or her body
parts.
3) The pincer grip develops between 6 or 9 months and by 10 months the fine
pincer grip is achieved with the baby being able to pick up small pellet like
objects between thumb and forefinger.
4) At 18 months the baby starts to spontaneously scribble if offered a crayon for
example.
5) At 24 months the child is able to copy a straight line.
6) By age 3 years the child can copy a circle.
7) By age 4 years the child can copy a cross.
8) By 4½ years the child can copy a square
9) By 5 years of age a triangle,
10) A diamond at about 6 years of age.
At the age of 3 the child starts to draw people usually starting as a “head and stick” extremity
person and as development progresses the person becomes more sophisticated.

Dr.Kamel Youssef Hassan, Pediatrician Consultant , Palestine - Gaza


E-mail: kyh10557@yahoo.com 86
MCQs PEDIATRICS

29) Theme : Poisoning


A. Antipsychotics
B. Benzodiazepines
C. Ethylene glycol
D. Methanol
E. Opiates
F. Organophosphates
G. Salicylates
H. Sympathomimetic
I. Tricyclic antidepressants
J. Volatile solvents
Select the most appropriate agent from the above list that would elicit the
following toxic effects:

1) Blindness, pulmonary oedema, metabolic acidosis, shock.


D. Methanol

2) Coma, hypotension, hypoventilation, reduced muscle tone and diminished


reflexes.
B. Benzodiazepines

3) Dysphagia, Dysphoria, Trismus, oculogyric crisis


A. Antipsychotics

Comments:
The relevant feature of the first case is the blindness and acidosis which is
probably due to Methanol, itself a weak acid but it causes lactic acidosis and
optic neuritis. The second case has features of CNS depression and
Benzodiazepines are typical. This should respond rapidly to Flumazenil. The
third scenario with oculogyric crisis suggests antipsychotics.

Dr.Kamel Youssef Hassan, Pediatrician Consultant , Palestine - Gaza


E-mail: kyh10557@yahoo.com 87
MCQs PEDIATRICS

30) Theme : chest pain in children


A. Acute severe asthma
B. Cardiac failure
C. Costochondritis
D. Diabetic ketoacidosis
E. Hysterical over-breathing
F. Inhalation of foreign body
G. Pleura effusion
H. Pulmonary embolism
I. Tension pneumothorax
J. Unilateral pneumonia
Select the most appropriate diagnosis from the options above that best explains
the following cases:
1) 14 year old presents to Accident & Emergency with a sudden onset of chest
pain and shortness of breath. Examination reveals reduced expansion and
hyperresonance over one side with reduced breath sounds.
I. Tension Pneumothorax

2) A 17 year old presents with a sudden onset of chest pain. Examination


reveals a sinus tachycardia and a pleural rub.
H. Pulmonary embolism

3) A 13 year old boy presents with tachypnoea and tachycardia. Examination of


the chest reveals poor air entry, however no crackles or wheeze. He has a
pulsus paradoxus.
A. Acute severe asthma

Comments:
Tension pneumothorax may result following trauma. It is a potentially life threatening event
with air accumulating under the pleura in the pleural space, which pushes the mediastinum
across the chest. Children may be shocked and will be hypoxic with reduced air entry and
hyperresonance. Distended neck veins may also be visible.
Pulmonary embolism is uncommon in infants and children. It may arise in children as a result
of surgery for example scoliosis surgery. It may also occur following prolonged inactivity. In
adolescents drug abuse, recent abortion or the oral contraceptive may be predisposing
problems. Clinical manifestations may suggest pneumonia. Dyspnoea is common associated
with pain. There may be no physical signs although the embolism is sufficiently large there
may be a pleural friction rub.
The clue in the third case the relatively silent chest associated with pulsus paradoxus. This
would suggest severe asthma. Other
Causes of pulsus paradoxus include  (pericardial tamponade /effusion/ constriction and
myocarditis).

Dr.Kamel Youssef Hassan, Pediatrician Consultant , Palestine - Gaza


E-mail: kyh10557@yahoo.com 88
MCQs PEDIATRICS

31) Theme : Abdominal Disease


A. Acute gastro-enteritis
B. Appendicitis
C. Constipation
D. Diabetes Mellitus
E. Henoch Schonlein Purpura
F. Mesenteric Adenitis
G. Nephroblastoma
H. Pancreatitis
I. Sickle Cell crisis
J. Wilm's Tumour
Select one option from the list above that is most suitable for the following
patients
1) A 9 year old Italian girl has severe abdominal pain. She is found to have
sclerae and painful fingers.
I. Sickle Cell crisis
Note:
Sickle cell disease is a haemoglobinopathy caused by substitution of Valine for
Glutamic acid at the 6th position of the Beta chain. It results in a haemolytic
anaemia resulting in vaso-occlusive episodes leading to splenic infarction and
dactylitis.
2) An 11 year old boy with a 12 hour history of anorexia and polyuria complains
of severe abdominal pain. On examination he has a temperature of 38°C and is
tender posteriorly on pr examination. BM is 4mmol per litre.
B. Appendicitis
Note:
describes acute appendicitis. The commonest acute surgical emergency of
childhood. Usually seen in children over the age of 5. Children present with a
temperature, abdominal pain and guarding in the right iliac fossa, rebound
tenderness, fever and vomiting. It is as a result of mucus obstructing the lumen
causing ischaemia of the mucosa and ultimately perforation.
3) An 11 year old boy with cystic fibrosis presents with recurrent attacks of
abdominal pain, vomiting and fever.
H. Pancreatitis
Note:
relates to description of chronic Pancreatitis in a child with cystic fibrosis. It is also a feature of
chronic hyperlipidaemia or congenital anomalies of the biliary retract or pancreas. Children
present with vomiting and abdominal pain which radiates through tot he back. Clinical
suspicion is confirmed by elevated amylase levels and calcification of the pancreas on
abdominal x-ray. Treatment is supportive

Dr.Kamel Youssef Hassan, Pediatrician Consultant , Palestine - Gaza


E-mail: kyh10557@yahoo.com 89
MCQs PEDIATRICS

32) Theme : Abdominal diseases of Childhood


A. Aerophagia
B. Alpha 1 antitrypsin deficiency
C. Galactosaemia
D. Hirschsprung's disease
E. Intussusception
F. Lactose intolerance
G. Mesenteric adenitis
H. Reyes syndrome
I. Thalassaemia
J. Wilson's disease

Select one of the above options that is the most appropriate diagnosis for the
following cases:
1) A 10 year old child develops severe vomiting and becomes encephalopathic
during a viral illness.
H. Reyes syndrome
Note:
Reyes syndrome is an acute disease of the liver often associated with a viral
infection or Salicylate toxicity . Laboratory findings include abnormal liver and
muscle enzymes. Treatment is supportive.

2) A 3 year old child with a 2 day history of headache and pharyngitis complains
of severe abdominal pain.
G. Mesenteric adenitis
Note:
Mesenteric adenitis commonly accompanying upper respiratory tract infections
and is due to inflammation in the abdominal lymph nodes. There is often a
history of pharyngitis and abdominal pain. Examination often reveals cervical
lymphadenopathy and central tenderness.

3) A 28 day old jaundice baby with early onset vomiting and diarrhoea.
C. Galactosaemia
Note:
Galactosaemia results as a result of galactose 1 phosphate uridyl transferase. It
accumulates in kidneys, liver and brain and clinical manifestations include
jaundice, hepatomegaly, hypoglycaemia, vomiting, seizures and poor weight
gain. Diagnosis is made by identifying urinary galactose. Treatment is dietary
adjustment.

Dr.Kamel Youssef Hassan, Pediatrician Consultant , Palestine - Gaza


E-mail: kyh10557@yahoo.com 90
MCQs PEDIATRICS

33) Theme : Skin lesions


A. Café au lait spots
B. Capillary haemangioma
C. Port-wine stain
D. Shagreen patch
E. Adenoma sebaceum
F. Cutis Marmorata
G. Herald's patch
H. Peri-oral pigmentation
I. Hypopigmentation
J. Tuber
Select one option from the list above that is most suitable for the following
patients
1) A child has an abundance of birth marks associated with axillary freckling.
A. Café au lait spots
Note:
relates to Neurofibromatosis type 1. Café au lait lesions are brown pigmented
marks found in abundance in Neurofibromatosis as well as other genetic
conditions for example Tuberous sclerosis.

2) A child with learning difficulties is described as having goose like flesh.


D. Shagreen patch
Note:
relates to Tuberous sclerosis. It is a condition where by hamartomatous lesions
become evident in early childhood. Seizures and learning difficulties are
common and skin manifestations include fibrous angiomatous lesions in the
naso labial folds known as adenoma sebaceum. White macules are seen over
the trunk known as Ash leaf macules and Shagreen patches resemble goose
like flesh. Subungual fibromata are also common.

3) A child with a history of Iron deficiency anaemia presents with per rectal
bleeding and is found to have this characteristic appearance.
H. Peri-oral pigmentation
Note:
refers to Peutz-Jeghers syndrome characterized by peri-oral pigmentation. A
history of Iron deficiency anaemia, colonoscopy may reveal polyps.

Dr.Kamel Youssef Hassan, Pediatrician Consultant , Palestine - Gaza


E-mail: kyh10557@yahoo.com 91
MCQs PEDIATRICS

34) Theme : Personal and Social Development


A. 3 months
B. 8 months
C. 12 months
D. 18 months
E. 2 years
F. 3 years
G. 4 years
H. 5 years
I. 6 years
J. 7 years
For each description of a child's personal and social development choose, from
the list of options, the highest development age the child is most likely to have
achieved.
1) A baby shows stranger awareness, is able to wave and clap his hands and is
starting to spoon feed himself.
C. 12 months
2) Can dress and undress including buttons and can imitate the construction of
bridges and steps with blocks.
G. 4 years
3) A child is starting to indicate the need to sit on the potty, play is usually
solitary at this age and the child tries to control objects over other children.
E. 2 years

Comments:
This question relates to the personal and social aspects of normal development.
1) Between 8- 10 months babies become anxious when separated from the main
carer, usually the mother for example if mother leaves the room.
2) Between 9-10 months the infant is able to wave bye bye and enjoys games
such as peek-a-boo. Between 18-24 months most children start to express their
toilet needs.
3) Up until the age of 3 plays is usually solitary, playing alongside their peers.
5) By 4 years the children usually start to role play for example pretending to be
the same sex parent and play involves other children in parallel first and then
interactively at around this age.

Dr.Kamel Youssef Hassan, Pediatrician Consultant , Palestine - Gaza


E-mail: kyh10557@yahoo.com 92
MCQs PEDIATRICS

35) Theme : Genital disorders


A. Aarskog syndrome
B. Fragile X
C. Gonadal dysgenesis
D. Hermaphroditism
E. Polycystic ovaries
F. Testicular feminisation syndrome
G. XO syndrome
H. XXX syndrome
I. XXY syndrome
J. XYY syndrome
Select the most appropriate diagnosis from the list above that best describes the
following cases
1) A 12 year old girl has seizures, depression and a conduct disorder.
H. XXX syndrome
Note:
describes a girl with XXX syndrome. Individuals are tall and thin. They have
behavioural problems and are mildly dysmorphic with epicanthic folds and
hypertelorism. Seizures are common
2) A 16 year old female presents with obesity, oligomenorrhoea and hirsutism.
E. Polycystic ovaries
Note:
relates to Stein-Leventhal also known as polycystic ovarian syndrome. During
adolescence there is change in body habitus and there is excess hair grown and
chronic and ovulation.

3) A 14 year old phenotypic female presents with primary amenorrhoea. She has
normal female external genitalia but intra-abdominal testicles with an XY
karyotype.
F. Testicular feminisation syndrome

Note:
The third case has testicular feminisation, a condition associated with
testosterone receptor defects that result in insensitivity to testosterone with
resultant female genitalia.

Dr.Kamel Youssef Hassan, Pediatrician Consultant , Palestine - Gaza


E-mail: kyh10557@yahoo.com 93
MCQs PEDIATRICS

36) Theme : NEONATAL JAUNDICE


A. Physiological jaundice
B. Biliary atresia
C. Hypothyroidism
D. Rhesus incompatibility
E. Congenital spherocytosis
F. Congenital cytomegalo virus infection
G. Galactosaemia
H. Glucose-6-phosphate dehydrogenase deficiency
I. Cystic fibrosis
J. Fructose intolerance
For each of the following jaundiced babies, select the most likely cause.
1) A two-week-old full term baby with a birth weight of 1.9 kg. He has gained a
small amount of weight already. On examination he has a white reflex on shining
a bright light into both pupils, moderate jaundice, a pansystolic murmur at the left
lower sternal edge, hepatosplenomegaly and a rash.
F. Congenital cytomegalo virus infection
Note:
Low birth weight, white reflex suggesting congenital cataracts, jaundice and
hepatosplenomegaly are indicative of a congenitally acquired infection. The rash
is likely to be petechiae secondary to bone marrow failure from the infection.
2) A 48 hour old baby who presents with poor feeding. She was delivered at
home following a concealed pregnancy. She was thought to be pale initially but
her skin subsequently became more yellow. Her mother had a late miscarriage
one year previously and is blood group A negative. Investigations show a
haemoglobin of 10 g/dL and an unconjugated bilirubin of 426 mmol/L. The blood
film shows evidence of haemolysis but otherwise normal red cell morphology.
D. Rhesus incompatibility
Note:
The previous miscarriage has sensitized the maternal immune system to the Rhesus antigen.
Haemolysis causes an unconjugated hyperbilirubinaemia. ABO incompatibility unlikely unless
the mother is blood group O.
3) A 5 day old baby appears jaundiced but otherwise well and feeds normally.
The serum bilirubin is checked by the community midwife and found to be
moderately elevated at 298 mmol/L. It is mainly unconjugated bilirubin, and the
haemoglobin is normal.
A. Physiological jaundice
Note:
This is a well baby and with only a slightly elevated unconjugated bilirubin. Levels can
normally increase to 300 mmol/L with no adverse effects on the baby. Some reports say that
levels are safe up to 400 mmol/L in a term well baby.

Dr.Kamel Youssef Hassan, Pediatrician Consultant , Palestine - Gaza


E-mail: kyh10557@yahoo.com 94
MCQs PEDIATRICS

37) Theme : Causes of vomiting


A. Gastro-oesophageal reflux
B. Intussusception
C. Meconium ileus
D. Meconium plug syndrome
E. Meckel's Diverticulum
F. Mid gut volvulus
G. Necrotising enterocolitis
H. Peptic ulcer
I. Pyloric stenosis
J. Tracheo-oesophageal fistula with atresia
Select the most appropriate option from the above list that would account for the
following presentations:
1) A baby on the postnatal ward is seen to have a cyanotic episode during
feeding.
J. Tracheo-oesophageal fistula with atresia
Note:
Tracheo-oesophageal fistula with oesophageal atresia results in excessive oral
secretions. The baby often chokes or coughs with feeds and becomes cyanotic.
There is an inability to pass a nasogastric tube and there is often a history of
maternal polyhydramnios.
2) A baby born at 33 weeks develops vomiting and abdominal distension on day
3. By day 4 the abdomen appears erythematous and the baby becomes
apnoeic.
G. Necrotising enterocolitis
Note:
Necrotising enterocolitis is a life threatening condition resulting in mucosal and
transmural necrosis. It is most prevalent in low birth weight babies and the
babies present with abdominal distension, gastric retention, bloody stools and
signs of a sepsis. Abdominal x-ray may reveal pneumatosis intestinalis.
3) A term baby is admitted to Special Care Baby unit with hypoglycaemic
episodes. His mother is a poorly controlled diabetic. By day 4 the hypoglycaemia
has resolved but the baby has not yet opened his bowels.
D. Meconium plug syndrome
Note:
Meconium plug syndrome is as a result of a plug of meconium causing
obstruction. There is usually an underlying disease such as cystic fibrosis,
Hirschsprung's disease, infant of a diabetic mother or maternal drug abuse.

Dr.Kamel Youssef Hassan, Pediatrician Consultant , Palestine - Gaza


E-mail: kyh10557@yahoo.com 95
MCQs PEDIATRICS

38) Theme : Treatment of Diarrhoeal illness


A. Metronidazole
B. Gluten free diet
C. No action
D. Steroids
E. Lactose free diet
F. Pancreatic enzyme supplements
G. Vitamin D
H. High fibre diet
I. Breast milk allergy
J. Praziquantel
Select one option from the list above that is most suitable for the following
patients:

1) A 7 month old baby has diarrhoea, which mother feels has been brought on
by the introduction of weaning. Mum is a known asthmatic.
E. Lactose free diet
Note:
refers to Cow's milk protein intolerance is mainly a clinical diagnosis with a
history that symptoms appeared at the time of cow's milk ingestion. The disorder
is usually temporary and can be managed by dietary adjustment.

2) A 13 year old Irish girl complains of diarrhoea. She is Iron deficient and
denies any history of GI upset after eating rice or potatoes. She has high anti-
reticulin antibodies
B. Gluten free diet
Note:
refers to Coeliac disease where a gluten free diet is appropriate. High reticulin or
more specifically now anti-TTG antibodies are expected. The diagnosis is
confirmed by jejunal biopsy.

3) A 10 year old boy presents with faecal urgency and bloody diarrhoea. He is
also under the care of ophthalmologists for a chronic iritis.
D. Steroids
Note:
relates to inflammatory bowel disease, Crohn's disease and ulcerative colitis
respectively. In Crohn's disease  the colonoscopy will reveal cobbled stone
mucosal appearances with fistulae where as in ulcerative colitis  crypt
abscesses are present.

Dr.Kamel Youssef Hassan, Pediatrician Consultant , Palestine - Gaza


E-mail: kyh10557@yahoo.com 96
MCQs PEDIATRICS

39) Theme : Causes of vomiting


A. Cyclical vomiting
B. Duodenal atresia
C. Over feeding
D. Meckel's diverticulum
E. Ulcerative colitis
F. Meconium ileus
G. Gastro-enteritis
H. Gastro-oesophageal reflux
I. Pyloric stenosis
J. Urinary tract infection
Select one option from the list above that is most suitable for the following
patients
1) An 8 year old girl shows signs of moderate dehydration. She has vomited all
fluids of 24 hours and the vomit is not bile stained. Her abdomen is soft and she
has had 3 similar episode in the past year.
A. Cyclical vomiting
Note:
Cyclical vomiting. This is thought to be a variant of migraine. The child presents
with bouts of vomiting which may persist for days and at times a headache is
associated.

2) A 12 week old infant vomits after each feed. He is developmentally normal


and is fed by bottle at 260mls per kilo per day.
C. Over feeding
Note:
Over feeding frequently results in regurgitation and vomiting. This may also be
accompanied by an excessive weight gain and dietary adjustment is indicated.

3) An 11 year old boy with a 3 month history of abdominal pain and weight loss
presents with bloody diarrhoea.
E. Ulcerative colitis
Note:
The last question relates to ulcerative colitis a form of inflammatory bowel
disease. In this case the child presents with recurrent bouts of profuse bloody
diarrhoea often with mucus and abdominal pain. Associated features include
arthritis, pyoderma gangrenosum, erythema nodosum and anaemia.

Dr.Kamel Youssef Hassan, Pediatrician Consultant , Palestine - Gaza


E-mail: kyh10557@yahoo.com 97
MCQs PEDIATRICS

40) Theme : Drugs to be avoided during breastfeeding


A. Amiodarone
B. Aspirin
C. Benzodiazepine
D. Chloramphenicol
E. Heparin
F. Lactulose
G. Lithium
H. Sulfasalazine
I. Sulphonamides
J. Warfarin
Which of the above drugs when prescribed to the mother is associated with the
following abnormalities in the breast-fed infant:

1) May result in neonatal hypothyroidism.


A. Amiodarone

2) May result in drowsiness and failure to thrive.


C. Benzodiazepine

3) May result in neonatal diarrhoea.


F. Lactulose

Comments:
Administration of certain drugs to breast feeding mums may result in toxicity in
the infants if the drug enters the milk in pharmacologically significant quantities.
Some drugs also inhibit the babies sucking reflex. Other drugs inhibit lactation.
Amiodarone needs to be avoided as it may result in neonatal thyroid
abnormalities as there is a theoretical risk from the release of Iodine.
Benzodiazepine may render the infant drowsy resulting in poor growth.
Laxatives if used over a prolonged period may enter the breast milk resulting in
the baby developing diarrhoea.

Dr.Kamel Youssef Hassan, Pediatrician Consultant , Palestine - Gaza


E-mail: kyh10557@yahoo.com 98
MCQs PEDIATRICS

41) Theme : Paediatric endocrinology


A. 3 Beta hydroxysteroid dehydrogenase deficiency
B. 17 Hydroxylase deficiency
C. 21 Hydroxylase deficiency
D. Adrenal hyperplasia
E. Craniopharyngioma
F. Hypoadrenalism
G. Phaeochromocytoma
H. Panhypopituitarism
I. Polyglandular endocrinopathy
J. Primary aldosteronism
Select the most appropriate diagnosis from the above options to explain the
following presentations:
1) An 8 year old boy presents with a history of enuresis. Examination reveals a
bi-temporal hemianopia and papilloedema.
E. Craniopharyngioma
Note:
refers to a child with a Craniopharyngioma which is the commonest
supratentorial tumour found in children. Children present with accelerated head
growth or headaches and examination findings include papilloedema and a bi-
temporal hemianopia This tumour may result in diabetes insipidus,
hypothyroidism and growth hormone deficiency.

2) A term baby is born with ambiguous genitalia. Examination reveals a phallus


with pigmented labial scrotal folds and the baby is found to be hyponatraemic.
C. 21 Hydroxylase deficiency
Note:
describes a baby with congenital adrenal hyperplasia. 95% of cases are due to
21 Hydroxylase deficiency and can be diagnosed by increased levels of 17OHP.
The salt wasting results in hyponatraemia (Na ) and hyperkalaemia (k ).
3) A 10 year old girl complains of headaches and vomiting. On examination she
is noted to be pale and her blood pressure measures 135/90. Numerous café au
lait lesions are found over her trunk.
G. Phaeochromocytoma
Note:
describes a child with Neurofibromatosis complicated by  phaeochromocytoma,
which is a tumour arising from the adrenal medulla or chromaffin cells. Children
present with headaches, palpitations, pallor, vomiting and night sweats. They
are found to be hypertensive.

Dr.Kamel Youssef Hassan, Pediatrician Consultant , Palestine - Gaza


E-mail: kyh10557@yahoo.com 99
MCQs PEDIATRICS

42) Theme : Drugs to be avoided during breastfeeding

A. Amiodarone
B. Aspirin
C. Benzodiazepine
D. Chloramphenicol
E. Heparin
F. Lactulose
G. Lithium
H. Sulfasalazine
I. Sulphonamides
J. Warfarin

Which of the above drugs, when prescribed to the mother, is associated with the
following abnormalities in the breast-fed infant:

1) May result in hypotonia and cyanosis.


G. Lithium

2) May result in prolonged jaundice.


I. Sulphonamides

3) May result in platelet dysfunction.


B. Aspirin

Comments:
Administration of certain drugs to breast-feeding mums may result in toxicity in
the infants if the drug enters the milk in pharmacologically significant quantities.
Some drugs also inhibit the babies sucking reflex. Other drugs inhibit lactation.
Lithium is also to be avoided as it may result in reduced tone and cyanotic
episodes.
Aspirin may result in a possible risk of Reyes syndrome; regular use may impair
platelet function, as well as producing a hypoprothrombinaemia, especially if the
infant is low in Vitamin K stores.

Dr.Kamel Youssef Hassan, Pediatrician Consultant , Palestine - Gaza


E-mail: kyh10557@yahoo.com 100
MCQs PEDIATRICS

43) Theme : Genital disorders


A. Aarskog syndrome
B. Fragile X
C. Gonadal dysgenesis
D. Hermaphroditism
E. Polycystic ovaries
F. Testicular feminisation syndrome
G. XO syndrome
H. XXX syndrome
I. XXY syndrome
J. XYY syndrome

Select the most appropriate diagnosis from the list above that best describes the
following cases

1) A young child is noted to have Cryptorchidism and a shawl scrotum.


A. Aarskog syndrome
Note:
describes a boy with Aarskog syndrome. Features include mild – moderate
learning difficulties, short stature, facial dysmorphic features include
hypertelorism anteverted nostrils and a webbed neck. Genital abnormalities
include shawl ‫ ﺷﺎل‬scrotum and cryptorchidism ► (hidden or obscure testis).

2) A 12 year old boy is noted to have a long thin face, micropenis, hypospadias
and has learning difficultie
J. XYY syndrome s
Note:
relates to a child with XYY syndrome. Individuals are usually tall and have
learning difficulties and abhorrent ‫ ﻣﺷﻣﺋز‬behaviour.
3) A child is noted to have macrocephaly and macroorchidism.
B. Fragile X
Note:
describes a boy with Fragile X syndrome. Features include moderate learning
difficulties, speech delay and autistic features. They have large heads and
prominent ears as well as testicular enlargement.

Dr.Kamel Youssef Hassan, Pediatrician Consultant , Palestine - Gaza


E-mail: kyh10557@yahoo.com 101
MCQs PEDIATRICS

44) Theme : ABDOMINAL PAIN


A. Peritonitis
B. Appendicitis
C. Constipation
D. Urinary tract infection
E. Mesenteric lymphadenitis
F. Migraine
G. Intussusception
H. Henoch-Schonlein vasculitis
I. Psychosomatic
J. Shigella dysentry
For the following patients with abdominal pain, choose the most likely diagnosis.

1) A 7 year old girl who has felt anorexic since the previous evening and has
vomited 3 times since. She has abdominal pain, which was made worse by the
bumpy car journey to hospital.
B. Appendicitis
Note:
A typical description of appendicitis. In the younger child/ baby there is often a
later presentation with perforation of the appendix being relatively more
common, as these children are not able to articulate their pain and localization of
pain is more difficult to discern with examination.

2) A 4 year old boy who has a sore throat, mild fever and peri-umbilical
abdominal pain.
E. Mesenteric lymphadenitis
Note:
Preceding sore throat and periumbilical pain are typical of mesenteric
lymphadenitis in a relatively well child.

3) A 9 year old boy has intermittent severe generalized colicky abdominal pain
which is getting worse. He has started to vomit over the last 24 hours. He has a
history of recurrent abdominal pain. Examination reveals a mass in the left iliac
fossa.
C. Constipation
Note:
This is a description of chronic severe constipation, which in extreme cases can
cause obstruction. There is faecal loading implying the chronicity. When it is this
severe it is important to rule out other pathology and consider the possibility that
is a manifestation of child abuse

Dr.Kamel Youssef Hassan, Pediatrician Consultant , Palestine - Gaza


E-mail: kyh10557@yahoo.com 102
MCQs PEDIATRICS

45) Theme : PRECOCIOUS PUBERTY


A. Craniopharyngioma
B. Exaggerated adrenarche
C. McCune-Albright syndrome
D. Hypothalamic hamartoma
E. Thelarche variant
F. Neurofibromatosis type 1
G. Granulosa-theca cell tumour
H. Hypothyroidism
I. Premature thelarche
J. Congenital adrenal hyperplasia
Which is the most likely diagnosis in the following cases:
1) A 4 year old girl present s with vaginal bleeding and café –au-lait pigmentation
which stops at the midline. Subsequent investigations demonstrate
gonadotrophin-independent puberty
C. McCune-Albright syndrome
Note:
McCune Albright syndrome is caused by an activating mutation of the GS alpha
subunit. This results in autonomous secretion of certain hormones e.g.
thyroxine, cortisol, oestradiol. There may be associated skin pigmentation and
fibrous dysplasia.
2) A 3 year old girl presents with pubic hair development, cliteromegaly and
acnes. There was an acceleration in her height velocity and no symptoms of
oestrogen excess. Neurological examination was normal.
J. Congenital adrenal hyperplasia
Note:
Virilising Congenital adrenal hyperplasia (CAH) typically presents with
symptoms of androgen excess. There is often behavioural disturbance with
mood swings. Diagnosis is confirmed by a raised 17 hydroxyprogesterone.
Cliteromegaly and a sustained increase in height velocity does not occur in
adrenarche.
3) A 2year old girl presents with isolated bilateral breast development.
Examination is otherwise normal as is her height velocity. Subsequent
investigation demonstrates undetectable oestradiol levels and a prepubertal
response to an LHRH test.
I. Premature thelarche
Note:
There may be a slight FSH predominance on the basal gonadotrophin levels.
Premature thelarche is self limiting and thought to be the result of the body being
exquisitely sensitive to low levels of oestradiol.

Dr.Kamel Youssef Hassan, Pediatrician Consultant , Palestine - Gaza


E-mail: kyh10557@yahoo.com 103
MCQs PEDIATRICS

46) Theme : Causes of vomiting


A. Gastro-oesophageal reflux
B. Intussusception
C. Meconium ileus
D. Meconium plug syndrome
E. Meckel's Diverticulum
F. Mid gut volvulus
G. Necrotising enterocolitis
H. Peptic ulcer
I. Pyloric stenosis
J. Tracheo-oesophageal fistula with atresia
Select the most appropriate option from the above list that would account for the
following presentations:
1) 5 week old breast fed baby is admitted with a 2 week history of vomiting and
dehydration. On examination an olive sized mass is palpable.
I. Pyloric stenosis
Note:
Pyloric stenosis is most prevalent amongst first born male and age of onset is
usually between 3-6 weeks of age. Babies present with projectile vomiting and
examination may reveal visible peristaltic waves and an olive sized mass may
be palpable in the upper epigastrium.

2) A 3 month old baby boy is admitted with episodic screaming. On examination


he is mottled and has a delayed capillary refill time. Palpation of the abdomen
reveals a sausage shaped mass.
B. Intussusception
Note:
Intussusception – a segment of bowel becomes telescoped in to an adjacent of
bowel often in areas where the bowel is aperistaltic, for example Peyer's
patches, Meckel's diverticulum, tumour or an area of oedema secondary to
Henoch Schonlein purpura. The child presents with pain, vomiting and signs of
shock. A mass may be palpable in the right upper quadrant.

3) A baby with cystic fibrosis develops vomiting and abdominal distension. A


ground glass appearance is seen on x-ray.
C. Meconium ileus
Note:
Meconium ileus syndrome is the commonest form of small bowel obstruction
and the baby presents with distension, vomiting and constipation. An x-ray may
reveal a ground glass appearance.

Dr.Kamel Youssef Hassan, Pediatrician Consultant , Palestine - Gaza


E-mail: kyh10557@yahoo.com 104
MCQs PEDIATRICS

47) Theme : Gross motor development


A. 1 month
B. 2 months
C. 4 months
D. 6 months
E. 8 months
F. 10 months
G. 1 year
H. 18 months
I. 2 years
J. 3 years
For each description of a child's gross motor development choose, from the list
of options, the highest development age the child is most likely to have
achieved.
1) A baby has good head control with no head lag when pulled from supine to
sitting.
C. 4 months
2) A baby can rise independently and stands momentarily alone. Is starting to
take 1-2 steps.
G. 1 year
3) A child who is walking and runs stiffly can climb stairs if hand is held.
I. 2 years
Comments:
This question tests your knowledge of the development stages of gross motor
skills.
1) At 3 months when placed prone an infant can lift his head with his arms extended.
2) By 4 months head lag is lost and this is evident when raised from supine to the sitting
position.
3) At 5 months the baby starts to roll over.
4) At 6 months the baby is starting to sit up with support for example cushions.
5) Sitting unsupported is usually seen by 6-7 months and
6) At 9 months the baby can start to crawl.
7) Cruising is usually achieved by about 10 months.
8) At 12 months the baby can rise independently and often starts to take a few steps
alone.
9) At 18 months the toddler can run although this is usually in a tiff fashion. He or she can
ascend stairs with assistance.
10)At 20 months the child can climb the stairs holding on to a rail.
11)At 3 the child can ascend stairs in an adult fashion using alternating feet per step,
however is unable to descend the stairs in such a fashion until 4 years of age.

Dr.Kamel Youssef Hassan, Pediatrician Consultant , Palestine - Gaza


E-mail: kyh10557@yahoo.com 105
MCQs PEDIATRICS

48) Theme : SHORT STATURE


A. Constitutional delay of growth
B. Growth hormone deficiency
C. Psychosocial deprivation
D. Familial short stature
E. Skeletal dysplasia
F. Hypothyroidism
G. Craniopharyngioma
H. Turner’s syndrome
I. Cushing’s syndrome
Which is the most likely diagnosis in the following cases?
1) A previously abused 4 year old child presents with growth failure over the last
12 months. She was placed in foster care for the last 18 months and is said to
much happier. Her height velocity has slowed down and she has gained weight.
She suffers with constipation and her teacher has noticed her attention span has
got worse recently.
F. Hypothyroidism
Note:
Height velocity will improve once abuse is removed. Constipation and poor
attention span are common symptoms of hypothyroidism in children.
2) A 7 year old boy was referred to clinic for investigation of short stature. He is
well and apart from occasional headaches has no other complaints. His height is
below the 0.4 th centile and within the target centile range calculated from his
parents heights. Previous height measurements from his GP show no evidence
of a reduction in height velocity.
D. Familial short stature
Note:
Short stature with no evidence of growth failure and a height within the target
centile range indicate familial short stature which requires no further
investigation.
3) A 3 year old girl presents with stature well below the 0.4 th centile. She had a
past history of pedal oedema at birth and uncomplicated repaired coarctation of
the aorta. She has been treated for constipation in the past but this has resolved,
other wise she is very well. An insulin tolerance test demonstrates a peak growth
hormone level of 25 mU/l.
H. Turner’s syndrome
Note:
The insulin tolerance test should only be performed if there is growth data that suggests
growth failure and other causes of short stature have been ruled out. GH deficiency is not
suggested here. She is very short and also well i.e. other pathology unlikely. A previous
history of coarctation and pedal oedema suggests Turner’s syndrome.
Dr.Kamel Youssef Hassan, Pediatrician Consultant , Palestine - Gaza
E-mail: kyh10557@yahoo.com 106
MCQs PEDIATRICS

49) Theme : Childhood malignancies

A. Burkitt's nasopharyngeal carcinoma


B. Ewing sarcoma
C. Hepatoblastoma
D. Hodgkin's lymphoma
E. Melanoma
F. Nesidioblastosis
G. Neuroblastoma
H. Teratoma
I. Wilm's tumour
J. Xeroderma pigmentosum
Select the most appropriate oncological diagnosis from the above list of options
to explain the following scenarios.

1) A baby is born with a solitary mass over the right buttock.


H. Teratoma
2) A young child presents with fevers and a mass in the femur is noted and a
diagnosis of malignancy is made. This malignancy is 30 times more common in
white children compared to black children.
B. Ewing sarcoma
3) A child of Nigerian parents presents with a unilateral tender cervical node. A
diagnosis of malignancy is made with this malignancy being 7 times more
common in Black Children compared with White children.
A. Burkitt's nasopharyngeal carcinoma
Comments:
Item 1 relates to a baby born with a sacrococcygeal teratoma. A solid tumour
found in new-borns with girls being affected more than boys. It is a mixed germ
cell tumour and may be associated with gastrointestinal or genital urinary
symptoms. Ewing sarcoma is a tumour of the bone, usually found in long bones
particularly the femur. It may present with pain, fever and tenderness and
sometimes pathological fractures. There is a poor prognosis. It is 30 times more
common in white children than black children. This genetic predisposition is
contrary to the Burkitt's nasopharyngeal carcinoma which is more common in
black children than white (by 7 fold). This tumour has been associated with the
Epstein-Barr virus and children present with sore throats, unilateral tenderness,
cervical lymphnodes, trismus and weight loss.

Dr.Kamel Youssef Hassan, Pediatrician Consultant , Palestine - Gaza


E-mail: kyh10557@yahoo.com 107
MCQs PEDIATRICS

50) Theme : Cranial Nerves


A. Abducens
B. Auditory
C. Facial
D. Glossopharyngeal
E. Oculomotor
F. Olfactory
G. Optic
H. Trochlear
I. Trigeminal
J. Vagus
Which of the above cranial nerves is affected in the following cases?
1) A 3 year old girl recovering from a pneumococcal meningitis complains of a
painful right eye. The pupil is enlarged and the eyes deviated inferiorly and
laterally. There is a right sided ptosis.
E. Oculomotor

2) A 12 year old boy complains of a numb feeling to the left side of his face. He
is unable to whistle or show is teeth on that side.
C. Facial

3) A 15 year old girl on treatment for Nephrotic syndrome presents with a


vesicular rash involving the right conjunctiva and right forehead, which is very
tender to touch.
I. Trigeminal
Comments:
The oculomotor nerve innovates the superior inferior and medial rectus as well
as the inferior oblique and the levator palpebrae muscles. Complete paralysis of
the oculomotor nerve causes ptosis, dilatation of the pupil and displacement of
the eye outwards and downwards.
An upper motor neurone facial nerve palsy is diagnosed by decreased voluntary
movement of the lower face with flattening of the nasal folds, where as a lower
motor neurone lesion tends to involve the upper and lower facial muscles.
The Trigeminal nerve – sensory distribution to the face is divided in to
ophthalmic, maxillary and the mandibular routes and motor function may be
tested by examining the masseter and temporalis muscles.

Dr.Kamel Youssef Hassan, Pediatrician Consultant , Palestine - Gaza


E-mail: kyh10557@yahoo.com 108
MCQs PEDIATRICS

51) Theme : Causes of Headaches


A. Benign intra-cranial hypertension
B. Cerebral tumour
C. Cluster headache
D. Encephalitis
E. Pseudotumour Cerebri
F. Meningitis
G. Migraine
H. Subdural haemorrhage
I. Sub arachnoid haemorrhage
J. Tension headache

Select the most appropriate diagnosis from the above list that would explain the
following cases.
1) A 6 year old girl with a history of Still's disease presents with a 2 month
history of headache and diplopia She is admitted to hospital. Investigations
carried out include a normal CT scan, normal CSF profile and normal opening
pressures.
E. Pseudotumour Cerebri
2) A 4 month old infant presents with poor feeding and irritability. On
examination he has a torticollis and his head circumference has increased form
the 50th – 98th centile since his last measurement 6 weeks ago.
I. Sub arachnoid haemorrhage

3) A 16 year old obese girl with a long history of headaches is found to have
papilloedema. The rest of the neurological examination is normal as is the CT
scan.
A. Benign intra-cranial hypertension
Comments:
Intra-cranial tumours- brain tumours are second to leukaemia as the most prevalent
malignancy in childhood. Children present with either signs of increased intra-cranial pressure
or with focal neurological signs. Infratentorial tumours are more common in the paediatric age
group than supratentorial tumours and the MRI scan is the best radiological tool for
delineating brain tumours. Pseudotumour cerebri is a clinical syndrome mimicking brain
tumours and characterised by normal CSF pressure and cell count and a normal structure to
the brain. Causes of which are multiple for example metabolic disorders, infections, drugs,
haematological disorders. Treatment focuses towards the underlying aetiology and pseudo
tumour cerebri is mainly self limiting. Benign intracranial hypertension is associated with
pregnancy as well as drugs such as the oral contraceptive, tetracyclines and rarely Growth
Hormone therapy. It is more common in association with obesity. LP usually reveals high
opening pressures and treatment includes removal of any precipitant with possible serial LP
and removal of CSF.

Dr.Kamel Youssef Hassan, Pediatrician Consultant , Palestine - Gaza


E-mail: kyh10557@yahoo.com 109
MCQs PEDIATRICS

52) Theme : Causes of abdominal pain


A. Acute appendicitis
B. Inflammatory bowel disease
C. Irritable bowel syndrome
D. Meckel's diverticulum
E. Muscle strain
F. Ovarian cysts
G. Pelvic inflammatory disease
H. Psoas haematoma
I. Pyelonephritis
J. Ureteric calculus
Select the most appropriate diagnosis from the above list for the following
presentations
1) A 13 year old boy with umbilical pain for the last 4 days presents with
anorexia, nausea and has not passed a bowel motion for 24 hours.
A. Acute appendicitis
Note:
The pain of acute appendicitis starts off as a peri-umbilical pain which then
locates to the right iliac fossa and is some times referred to the back. It is sharp
and steady; it results in nausea, emesis and local tenderness.

2) A 10 year old boy with lower abdominal pain for the last 10 days presents with
a history of passing 6-8 loose stools. Temperature is 38.8°C. He is tender in the
right lower quadrant and has an anal fistula.
B. Inflammatory bowel disease
Note:
Inflammatory bowel disease results in recurrent lower abdominal pain, often
radiating to the back. The pain is dull and crampy in nature and is associated
with fever, weight loss and tenesmus.

3) A 14 year old haemophiliac patient presents with lower abdominal pain and a
limp. Attempts to straighten his leg results in excruciating pain
H. Psoas haematoma
Note:
Bleed into the Psoas muscle which may be due to coagulopathy or secondary to
sepsis (DIC) results in back pain radiating to the hip with painful flexion at the hip
joint.

Dr.Kamel Youssef Hassan, Pediatrician Consultant , Palestine - Gaza


E-mail: kyh10557@yahoo.com 110
MCQs PEDIATRICS

53) Theme : NORMAL DEVELOPMENT


A. 4 weeks
B. 2 months
C. 7 months
D. 10 months
E. 12 months
F. 18 months
G. 2 years
H. 3 years
I. 4 years
J. 5 years
At what age would you expect the average child to achieve the following
milestones:-
1) Show casting, able to pick up small objects with a neat pincer grasp and have
developed concept of object permanence.
E. 12 months
2) Able to skip, draw a triangle from copy, name the heavier of 2 weights, count
up to ten and ask questions about meaning of words.
J. 5 years

3) Able to identify colours


H. 3 years

Comments:

It is important to be familiar with the developmental milestones, particularly up to


the age of 3 years.

Milestones are grouped under 4 headings:

 Gross motor
 Fine motor
 Hearing and language
 social

You should know the age range for each milestone. Developmental milestones
are in Illingworth: 'The Normal Child'.

Dr.Kamel Youssef Hassan, Pediatrician Consultant , Palestine - Gaza


E-mail: kyh10557@yahoo.com 111
MCQs PEDIATRICS

54) Theme : Chemotherapy


A. 6-Mercaptopurine
B. Actomyosin D
C. Asparaginase
D. Bleomycin
E. Cisplatin
F. Cyclophosphamide
G. Cytosine arabinoside
H. Doxorubicin
I. Methotrexate
J. Vincristine
Select the most appropriate chemotherapeutic agent from the above list that
corresponds with the following actions and side effects:

1) An alkylating agent resulting in haemorrhagic cystitis.


F. Cyclophosphamide

2) A drug inhibiting protein synthesis may result in a dose related Pancreatitis.


C. Asparaginas

3) An anthracycline with a serious risk of cardio-toxicity.


H. Doxorubicin
Comments:
All cytotoxic agents have adverse effects. General side-effects include nausea, vomiting
and bone marrow suppression, alopecia and stomatitis. More specific side-effects are
listed in the questions above. Alkylating agents such as Cyclophosphamide may cause
a haemorrhagic cystitis. Asparaginase results in a dose related Pancreatitis.
The Anthracyclines e.g. (Doxorubicin and Daunorubicin) are cardiotoxic, which is
often very difficult to detect. Monitoring with echocardiograms is advised.
Vincristine is an alkaloid agent and results in sensory motor neuropathy with long-term
use. It also has an affect on the autonomic system resulting in severe constipation and
paralytic ileus. It may also result in sensory changes with parathesis progressing to loss
of tendon reflexes.
Bleomycin is the main drug resulting in lung damage and occurs in up to 10% of
patients. The damage is dose related.

Dr.Kamel Youssef Hassan, Pediatrician Consultant , Palestine - Gaza


E-mail: kyh10557@yahoo.com 112
MCQs PEDIATRICS

55) Theme : SYNDROMES


A. Down's syndrome
B. Turner's syndrome
C. Noonan's syndrome
D. Klinefelter's syndrome
E. Treacher-Collins syndrome
F. Marfan's syndrome
G. Sotos syndrome
H. Klippel-feil syndrome
I. Laurence Moon Biedl syndrome
J. MCune Albright syndrome
Which is the most applicable diagnosis for the following children?
1) A boy with learning difficulties and short stature is found to have pulmonary
stenosis when investigated for a murmur. He has some dysmorphic features
including wide carrying angle of the arms.
C. Noonan's syndrome
Note:
Children with Noonan’s syndrome have short stature and may have delayed
puberty. There is also associated short sightedness. Pulmonary stenosis is the
most frequent congenital heart lesion.

2) Boy is seen in cardiology clinic for investigation of a murmur heard on routine


check. His father died at an early age from aortic dissection. He is noted to be
above the 98th centile in height for his age.
F. Marfan's syndrome
Note:
Marfans syndrome is also associated with a high arched palate, arachnodactly
and a predisposition to pneumothoraces.
3) A child with learning difficulties was noticed to be long at birth. He has a large
head, downslanting eyes, bossed forehead and prominent jaw. He is clumsy and
previously been investigated for hypotonia.
G. Sotos syndrome
Note:
This is a typical description of Sotos syndrome which is one of the overgrowth
syndromes.

Dr.Kamel Youssef Hassan, Pediatrician Consultant , Palestine - Gaza


E-mail: kyh10557@yahoo.com 113
MCQs PEDIATRICS

56) Theme : Connective tissue disease


A. Ankylosing spondylitis
B. Dermatomyositis
C. Ehlers Danlos syndrome
D. Henoch Schonlein purpura
E. Juvenile chronic arthritis
F. Kawasaki disease
G. Osteogenesis imperfecta
H. Rheumatic fever
I. Scleroderma
J. Systemic lupus erythematosus
Select one of the above diagnoses that would explain the following
presentations:
1) A 12 year old girl presents with oesophageal dysmotility, interstitial
pneumonitis and has cardiac conduction abnormalities (arrhythmias).
I. Scleroderma
Note:
describes a child with scleroderma, a chronic fibrotic disturbance which tends to
be relapsing and remitting. It affects the GI tract, heart lungs and kidneys.

2) A 5 year old presents with conjunctivitis fever and desquamation of the


periphery.
F. Kawasaki diseas
Note:
Kawasaki's disease  is also known as mucocutaneous lymph node syndrome. It
is a clinical diagnosis made by a history of fever for 5 days, bilateral
conjunctivitis, mucosal involvement and peripheral changes including erythema
and desquamation. Cardiac involvement is present in approximately 30% of
cases and this is a serious complication.

3) A 10 year old girl presents with weight loss, peri-orbital oedema, weakness
and is noted to have a photosensitive rash.
B. Dermatomyositis
Note:
relates to the diagnosis of Dermatomyositis, a multi system disease where
individuals have a violaceous heliotrope rash around the eyes, Guttron’s patches
on the knuckles and inflammation of muscles and joints resulting in weakness in
muscles and arthritis. The condition is often associated with underlying
malignancy.

Dr.Kamel Youssef Hassan, Pediatrician Consultant , Palestine - Gaza


E-mail: kyh10557@yahoo.com 114
MCQs PEDIATRICS

57) Theme : Causes of abdominal pain


A. Acute appendicitis
B. Inflammatory bowel disease
C. Irritable bowel syndrome
D. Meckel's diverticulum
E. Muscle strain
F. Ovarian cysts
G. Pelvic inflammatory disease
H. Psoas haematoma
I. Pyelonephritis
J. Ureteric calculus
Select the most appropriate diagnosis from the above list for the following
presentations
1) A 7 year old boy presents with a 2 weeks history of flank pain and rigors,
intermittent fevers.
I. Pyelonephritis
Note:
Pyelonephritis - an infection of the renal tract causes acute sudden pain in the
lower abdomen and back. This is associated with urinary frequency and dysuria,
costochondral tenderness and fever.

2) A 13 year old girl presents with recurrent abdominal pain and haematuria.
She is apyrexial
J. Ureteric calculus
Note:
Urolithiasis or urinary tract calculi present with haematuria (macro or
microscopic) abdominal pain, dysuria and voiding abnormalities.

3) An 8 year old girl with a history of recurrent central abdominal pain presents
with pallor and an Iron deficiency anaemia.
D. Meckel's diverticulum
Note:
Meckel's diverticulum results in peri-umbilical sharp pain associated with
haematochezia
Haematochezia (bright red blood per rectum)

Dr.Kamel Youssef Hassan, Pediatrician Consultant , Palestine - Gaza


E-mail: kyh10557@yahoo.com 115
MCQs PEDIATRICS

58) Theme : Causes of headache and fever


A. Bacterial meningitis
B. Congenital rubella
C. Congenital toxoplasmosis
D. Herpes simplex encephalitis
E. Leukaemic infiltration
F. Meningococcal meningitis
G. Pneumococcal meningitis
H. Status epilepticus
I. Tuberculous meningitis
J. Varicella cerebellitis
Select the most likely diagnosis from the above list that would account for the
following presentations
1) A child is admitted with alternating levels of consciousness and a high
temperature. EEG shows periodic, lateralised epileptiform discharges.
D. Herpes simplex encephalitis
2) A baby is born at term weighing 2.5kilos. He is noted to have microcephaly.
An ultrasound scan of his head reveals calcification.
C. Congenital toxoplasmosis
3) A baby of 6 months has a full septic screen. His CSF shows 10 white cells
predominantly lymphocytes, high protein and low glucose.
I. Tuberculous meningitis
Comments:
Examination of the cerebral spinal fluid is essential in confirming the diagnosis of conditions
such as meningitis, encephalitis and sub arachnoid haemorrhage. The main contraindication
for performing a lumbar puncture is any condition that may lead to “Coning” or Herniation of
intra-cranial contents. Therefore fundoscopy to exclude the presence of papilloedema is
essential. Routine CSF study includes red cell count, white cell count, glucose protein and
bacterial cultures.
In meningitis typical CSF findings include A neutrophilic pleocytosis with an elevated protein
and reduced glucose. Gram stains reveal organisms in approximately 80% of cases of
bacterial meningitis.
In encephalitis CSF is typically  normal. In Tuberculous meningitis  the cell count is elevated
and the predominance of lymphocytes. Glucose is reduced (1/3 the serum) however the
protein concentration is elevated (0.5-6 ) (normal protein g/l 0.2-0.4). With Leukaemic
infiltration of the central nervous system atypical  Leukaemic cells may be present in the
CSF. Further cytopathology studies may be indicated.
Congenital toxoplasmosis affected infants may have fever, rash, petechiae,
hepatosplenomegaly. They may have dysmorphic features such as microcephaly or
hydrocephalus, microphthalmia and chorioretinitis. Cerebral calcification is common.
Prognosis is guarded as affected patients may have persistent neurological disability. (Brain
Calcification in CMV infection periventricular , in toxoplasmosis  widespread No calcification
in rubella)

Dr.Kamel Youssef Hassan, Pediatrician Consultant , Palestine - Gaza


E-mail: kyh10557@yahoo.com 116
MCQs PEDIATRICS

59) Theme : Cranial Nerves


A. Abducens
B. Auditory
C. Facial
D. Glossopharyngeal
E. Oculomotor
F. Olfactory
G. Optic
H. Trochlear
I. Trigeminal
J. Vagus
Which of the above cranial nerves is affected in the following cases?

1) A 16 year old female who is pregnant complains of headaches and has


blurring of the disc margin on fundoscopy.
G. Optic

2) A baby is born with an ipsilateral diffuse birth mark over the right side of his
face. He develops seizures in the second week of life.
I. Trigeminal

3) A 15 year old boy recovering from a head injury is unable to abduct his right
eye beyond the mid-line.
A. Abducens
Comments:
The first case has probable benign intracranial hypertension causing
papilloedema. The optic nerve is examined by fundoscopy as well as visual
acuity and visual fields. The Trigeminal nerve – sensory distribution to the face is
divided in to ophthalmic, maxillary and the mandibular routes and motor function
may be tested by examining the masseter and temporalis muscles. The
abducens nerve innovates bilateral rectus muscle of the eye therefore paralysis
causes medial; deviation of the eye and inability to abduct the eye beyond the
mid-line.

Dr.Kamel Youssef Hassan, Pediatrician Consultant , Palestine - Gaza


E-mail: kyh10557@yahoo.com 117
MCQs PEDIATRICS

60) Theme: Childhood syndromes.


A. Alport syndrome
B. Down’s syndrome
C. Duchenne muscular dystrophy
D. Edward’s syndrome
E. Gaucher’s syndrome
F. Klinefelter’s syndrome
G. Pierre robin syndrome
H. Sturge webber syndrome
I. Treacher collins syndrome
J. Turner’s syndrome
For each patient below choose from the list above the single most likely diagnosis from the list
above. Each option may be chosen more than once or not at all.
1) A neonate is seen with short chin cleft palate and feeding and respiratory difficulties.
G. Pierre robin syndrome
Note:
Pierre Robin syndrome. Results in neonatal difficulty in feeding, micrognathia (short chin due
to hypoplasia of mandible) ± cleft palate and eye abnormality. These babies should be nursed
prone to prevent obstructive apnoea.
2) A 9 month old baby with low short receding chin low set ears, deafness and notching of
lower eye lids.
I. Treacher collins syndrome
Note:
Treacher Collins syndrome. Due developmental abnormalities of first and second pharyngeal
pouch. Low set ears (80%), deafness 40%, under developed mandible.
3) A 16 year old girl presents with primary amenorrhoea, short stature, and radio-femoral
delay.
J. Turner’s syndrome
Note:
Turner’s syndrome. 45XO Chromosomal abnormality. Associated with short stature (<130
cm), primary amenorrhoea, rudimentary gonads, wide carrying angle, webbing of the neck
cardiac abnormalities.
4) A 6 week old baby with low set ears, receding chin umbilical hernia and rocker bottom feet.
D. Edward’s syndrome
Note:
Edward’s syndrome (Trisomy 18). Rigid baby, flexion of limbs, mental retardation. Mean
survival less than 10 months.
5) A 5 year old boy presents with waddling gait and positive Gower’s sign.
C. Duchenne muscular dystrophy
Note:
Duchenne’s muscular dystrophy X-linked recessive .

Dr.Kamel Youssef Hassan, Pediatrician Consultant , Palestine - Gaza


E-mail: kyh10557@yahoo.com 118
MCQs PEDIATRICS

61) Theme : Childhood Breathlessness


A. Asthma
B. Bronchiolitis
C. Bronchiectasis
D. Broncho pulmonary dysplasia
E. Cardiac failure
F. Laryngomalacia
G. Tracheal haemangioma
H. Tracheal oesophageal fistula
I. Vitamin D deficiency
J. Vocal cord palsy
Select the most appropriate choice from the above list that is the most
appropriate diagnosis in the following cases
1) A premature baby presents with a cough and cyanotic episodes during feeds.
H. Tracheal oesophageal fistula

2) A girl with hypogammaglobulinaemia develops haemoptysis and a purulent


sputum.
C. Bronchiectasis

3) A baby is born by a ventouse delivery following failed forceps. She is born in


poor condition and stridor is noted shortly after birth.
J. Vocal cord palsy
Comments:
Item 1describes a baby with tracheo-oesophageal fistula, which typically
presents with a cough and choking cyanotic episodes during feeds.
Bronchiectasis typically produces clubbing and haemoptysis as well as
persistent crepitations in the chest. This process may follow a severe pneumonia
for example pertussis, measles, TB. It can also complicate immune-deficiency
states and Cystic fibrosis. Kartagener's syndrome is a triad of 1) bronchiectasis
2) sinusitis 3) dextrocardia. The features result from abnormal ciliary function.
Vocal cord paralysis is the answer to Item 6. Stridor from birth may occur as a
result of birth trauma or an abnormality of the central nervous system, such as
hydrocephalus.

Dr.Kamel Youssef Hassan, Pediatrician Consultant , Palestine - Gaza


E-mail: kyh10557@yahoo.com 119
MCQs PEDIATRICS

62) Theme : Paediatric endocrinology


A. 3 Beta hydroxysteroid dehydrogenase deficiency
B. 17 Hydroxylase deficiency
C. 21 Hydroxylase deficiency
D. Adrenal hyperplasia
E. Craniopharyngioma
F. Hypoadrenalism
G. Phaeochromocytoma
H. Panhypopituitarism
I. Polyglandular endocrinopathy
J. Primary aldosteronism
Select the most appropriate diagnosis from the above options to explain the
following presentations:
1) A child is admitted to ITU with meningococcaemia. On arrival hyponatraemia
and hyperkalaemia is revealed. Urinary electrolytes reveal high levels of Sodium
and Chloride and low levels of Potassium.
F. Hypoadrenalism
Note:
describes a child with meningococcaemia complicated by hypoadrenalism of Waterhouse-
Friderichsen syndrome. This is due to haemorrhage into the adrenal glands. If the condition is
unrecognised adrenal crisis may result and therefore treatment must be immediate and
vigorous (Hydrocortisone, IV fluids and Glucose).
2) A child attending diabetic clinic complains of feeling tired. She is found to
have thin hair, patches of vitiligo and the tongue is coated in white. Blood tests
reveal low Calcium and high Phosphate.
I. Polyglandular endocrinopathy
Note:
describes a child with Polyglandular endocrinopathy. In this case a child with diabetes, vitiligo,
Addison's disease, hypoparathyroidism and chronic mucocutaneous Candidiasis (type 1
Polyglandular endocrinopathy.) Type 2 Polyglandular endocrinopathy is diagnosed if there is
history of hyperthyroid disease or diabetes plus Addison's disease.
3) A baby is born at term. Birth weight is on the 50th centile. Within the first few weeks
hypoglycaemia is diagnosed. The growth of the baby is monitored closely and By 9 months of
age parameters have fallen to the second centile.
H. Panhypopituitarism
Note:
The last question describes a baby with growth hormone deficiency. From the above option
Panhypopituitarism is the answer. Babies with growth hormone deficiency present with
episodes of hypoglycaemia and may present as an emergency with apnoeic episodes or
cyanotic episodes. They are usually normal in birth size; however by 1 year of age growth has
tailed off. In Panhypopituitarism the deficiency may be of growth hormone only or of multiple
hormones and therefore laboratory investigations should include thyroid function tests, cortisol
and ACTH, gonadotrophin's and prolactin as well as growth hormone.
Dr.Kamel Youssef Hassan, Pediatrician Consultant , Palestine - Gaza
E-mail: kyh10557@yahoo.com 120
MCQs PEDIATRICS

63) Theme : WEAKNESS IN THE LOWER LIMBS

A. Guillain-Barre syndrome
B. Transverse Myelitis
C. Multiple sclerosis
D. Chronic fatigue syndrome
E. Spinal tumour
F. Dermatomyositis
G. Congenital myopathy
H. Duchenne muscular dystrophy
I. Fascioscapulohumeral dystrophy
J. Spinal abscess

For these children with lower limb weakness select the most likely diagnosis

1) A 4 year old boy is brought to clinic by his mother who feels he has weak legs
and is finding climbing the stairs difficult. He walked independently at 20 months
and has received speech therapy for a mild speech delay. On examination his
lower limb reflexes are normal, his muscles are well developed distally although
weak proximally.
H. Duchenne muscular dystrophy

Note:
DMD occurs in boys(X-linked recessive) and presents with delayed walking i.e.>
18months. Speech delay, cardiomyopathy, scoliosis and calf pseudohypertrophy
are also features.

2) A 12 year old girl presents with a 5 day history of progressive weakness in her
legs and is now unable to walk. She has been passing urine frequently and with
difficulty. On examination there is power with gravity eliminated in her legs, very
brisk ankle and knee jerks and upgoing plantar responses. Her bladder is
palpable up to the umbilicus. Sensation is reduced to touch below T5, vibration
and position sense are preserved 3 weeks previously she had a presumed viral
illness. MRI brain is normal but shows swelling and increased signal over the
spinal cord T5-7 on T1- weighted images. Lumbar puncture shows 33
lymphocytes and monocytes per microlitre and protein 570 mg/L in the CSF.
B. Transverse Myelitis

Note:
The features are suggestive of a lower motor neurone lesion which could also be

Dr.Kamel Youssef Hassan, Pediatrician Consultant , Palestine - Gaza


E-mail: kyh10557@yahoo.com 121
MCQs PEDIATRICS

caused by a spinal tumour / abscess however the MRI would demonstrate these
latter pathologies. The CSF lymphocytosis and increased protein may be seen in
transverse myelitis. Transverse myelitis has unknown aetiology but has been
postulated to have a viral or mycoplasma origin.

3) A 14 year old girl has been off school for 4 months with headaches, tiredness
and weakness of her legs. In the initial weeks of her illness she had a low-grade
fever, sore throat and cervical lymphadenopathy. A blood count showed atypical
lymphocytes and liver function tests show a raised AST. These were re-checked
1 month later when they had normalised and thyroid function, urine culture urea
& electrolytes were also normal. It is difficult to get her full co-operation for
examination but there are no objective signs of weakness and reflexes are all
present, symmetrical with downgoing plantar responses.
D. Chronic fatigue syndrome

Note:
The previous viral illness and subsequent lethargy with no consistent abnormal
investigations suggest chronic fatigue syndrome.

Dr.Kamel Youssef Hassan, Pediatrician Consultant , Palestine - Gaza


E-mail: kyh10557@yahoo.com 122
MCQs PEDIATRICS

64) Theme : FRACTURES IN CHILDREN


A. Accidental fracture
B. Child physical abuse
C. Osteoporosis
D. Rickets
E. Metabolic bone disease of prematurity
F. Hypoparathyroidism
G. Hypophosphatemic rickets
H. Pseudohypoparathyroidism
I. Osteogenesis imperfecta
J. Osteopetrosis
For each patient select the most likely diagnosis.
1) A 15 month girl is in foster care following child protection procedures when
she presented with a spiral fracture at the age of 11 months. Her mother is
single, poorly supported and had been suffering from postnatal depression at the
time. There had been no explanation offered for the original injury. She now
presents with a greenstick fracture of the femur following a trivial fall in the care
of the experienced foster-mother who has been known to social services for
many years. She is just walking independently. Skull X-ray shows several
Wormian bones.
I. Osteogenesis imperfecta
Note:
Wormian bones are indicative of osteogenesis impefecta. This condition is an
important differential of NAI.
2) An 11 month old girl is brought to casualty refusing to move her left arm. An x-
ray demonstrates a spiral fracture of the humerus. The mother says that 30
minutes previously she lifted her daughter out of the bath and lost grip of her
right hand and her body momentarily twisted round her left arm.
A. Accidental fracture
Note:
There is no delay in presentation and the mechanism of injury fit the description
of the episode.
3) A 10 year old girl presents with back pain. A spinal X ray shows collapsed
lumbar vertebrae at L3 and L4 and generalized osteopenia. She has a history of
3 previous limb fractures after relatively trivial falls. She is otherwise healthy and
well-grown for her age.
I. Osteogenesis imperfecta
Note:
Collapsed vertebra and osteopenia are uncommon in children. Coupled with low
trauma fractures and no dysmorphism, type 1 osteogenesis imperfecta is likely.

Dr.Kamel Youssef Hassan, Pediatrician Consultant , Palestine - Gaza


E-mail: kyh10557@yahoo.com 123
MCQs PEDIATRICS

65) Theme : FRACTURES IN CHILDREN


A. Accidental fracture
B. Child physical abuse
C. Osteoporosis
D. Rickets
E. Metabolic bone disease of prematurity
F. Hypoparathyroidism
G. Hypophosphatemic rickets
H. Pseudohypoparathyroidism
I. Osteogenesis imperfecta
J. Osteopetrosis
For each patient select the most likely diagnosis.
1) A toddler of 3 years presents to the accident and emergency department with
swelling of his leg and is found to have a spiral fracture of the tibia. His mother
reports that he had tripped and fallen the previous day but she had not noticed
any sign of injury at the time. She is a single parent with little family support. The
child is not on the child protection register.
B. Child physical abuse
Note:
It is imperative to determine whether the mechanism of injury is compatible with
the history. There is delay in presentation and also spiral fracture indicates
“twisting injury” rather than falling over.
2) An infant is admitted with symptoms and signs of respiratory infection and is
found to have several posterior rib fractures on chest radiograph. He was born
prematurely at 37 weeks gestation and was observed overnight on the special
care baby unit for tachypnoea which settled by the following day. On
assessment it is also apparent that his head circumference has increased at an
excessive rate and has crossed 3 centiles since birth.
B. Child physical abuse
Note:
Posterior rib fractures are highly suggestive of non accidental injury (NAI). The
excessive head growth is likely to be secondary to a previous shaking injury
which has resulted in an intracerebral bleed and hydrocephalus.
3) A 4 month baby girl is presented with swelling of her right arm and is found to
have a spiral fracture of the humerus. She had been in the care of her mother’s
boyfriend who reported that he had nearly dropped her that day when reaching
for her bottle and had inadvertently pulled on her arm to save her.
A. Accidental fracture
Note:
This history could be compatible with an accidental injury.

Dr.Kamel Youssef Hassan, Pediatrician Consultant , Palestine - Gaza


E-mail: kyh10557@yahoo.com 124
MCQs PEDIATRICS

66) Theme : DELAYED MILESTONES


A. 4 weeks
B. 2 months
C. 7 months
D. 10 months
E. 12 months
F. 18 months
G. 2 years
H. 3 years
I. 5 years
J. 6 years
At what age would you refer a child for investigation if they were unable to do the
following:-
1) Speak in intelligible short sentences
G. 2 years
2) Be reliably dry at night (i.e. primary nocturnal enuresis) whilst continent of
urine and faeces by day and otherwise well and asymptomatic.
J. 6 years
3) Be able to sit unsupported
D. 10 months

Comments:

It is important to be familiar with the developmental milestones, particularly up to


the age of 3 years.

Milestones are grouped under 4 headings:

 Gross motor
 Fine motor
 Hearing and language
 Social

You should know the age range for each milestone. Developmental milestones
are in Illingworth: 'The Normal Child'.

Dr.Kamel Youssef Hassan, Pediatrician Consultant , Palestine - Gaza


E-mail: kyh10557@yahoo.com 125
MCQs PEDIATRICS

67-Theme : GASTRO-INTESTINAL DISORDERS IN NEONATES

A. Ano-rectal atresia
B. Hirschsprung’s disease
C. Infantile hypertrophic pyloric stenosis
D. Intestinal atresia
E. Intussusception
F. Meconium ileus
G. Necrotizing enterocolitis
H. Tracheo-oesophageal fistula
I. Volvulus neonatorum

Select the most appropriate diagnosis from the list above that would explain the
presentation of the following neonates:

1) A newborn baby girl presents with gross abdominal distension and bilious
vomiting. She has also got cystic fibrosis and her abdominal x-ray shows
distended coils of bowel, but no fluid levels.
F. Meconium ileus

2) A premature infant (31-week gestation) presents with distended and tense


abdomen. She is passing blood and mucus per rectum, and she is also
manifesting signs of sepsis.
G. Necrotizing enterocolitis

3) A newborn baby boy presents with mild abdominal distension and failure to
pass meconium after 24 hours. X-ray reveals dilated loops of bowel with fluid
levels.
B. Hirschsprung’s disease

Comments:
One in 15 000 newborns will have a distal small bowel obstruction secondary to
abnormal bulky and viscid meconium. Ninety percent of these infants will have
cystic fibrosis and the abnormal meconium is the result of deficient intestinal
secretions. This condition presents during the first days of life with gross
abdominal distension and bilious vomiting. X-ray of the abdomen shows
distended coils of bowel and typical mottled ground glass appearance. Fluid
levels are scarce as the meconium is viscid. Necrotizing enterocolitis is more
common in premature infants. Mesenteric ischemia causes bacterial invasion of

Dr.Kamel Youssef Hassan, Pediatrician Consultant , Palestine - Gaza


E-mail: kyh10557@yahoo.com 126
MCQs PEDIATRICS

the mucosa leading to sepsis. Terminal ileum, caecum and the distal colon are
commonly affected. The abdomen is distended and tense, and the infant passes
blood and mucus per rectum. X-rays of the abdomen shows distended loops of
intestine and gas bubbles may be seen in the bowel wall. Hirschsprung’s
disease is an absence of ganglion cells in the neural plexus of the intestinal wall.
It is more common in boys than girls. The delayed passage of meconium
together with distension of abdomen is the usual clinical presentation. A plain
abdominal x-ray will demonstrate dilated loops of bowel with fluid levels and a
barium enema can be helpful when it demonstrates a cone with dilated
ganglionic proximal colon and the distal aganglionic bowel failing to distend.

Dr.Kamel Youssef Hassan, Pediatrician Consultant , Palestine - Gaza


E-mail: kyh10557@yahoo.com 127
MCQs PEDIATRICS

7- PEDIATRICS BEST REPONSE.


1) The following laboratory results were returned in a 6 week old boy admitted
with 6 days of severe projectile vomiting:

pH 7.51 (7.35-7.45)
PO2 12 KPa (95 mmHg)
PCO2 4.7 KPa (35 mmHg)
Blood Urea 11 mmol/l
Sodium 131 mmol/l
Potassium 3 mmol/l
Chloride 83 mmol/l

Which of the following is true concerning this patient?

1-He has respiratory alkalosis


2-He is likely to have a bulging anterior fontanelle
3-He should be resuscitated immediately with normal saline
4-He should be commenced immediately on half strength soy protein, low
lactose formula
5-X-ray of abdomen is likely to show dilated loops of small bowel
Comments:
He has a metabolic, rather than respiratory alkalosis as CO2 is not reduced. He is likely
to have a sunken fontanelle as he is very dehydrated.
Pyloric stenosis would highly likely as because of the age and hypochloraemic,
hypokalaemic metabolic alkalosis. He should be resuscitated with normal saline
first. Dilated bowel would not be expected, as pyloric rather than small bowel
obstruction is present.

Dr.Kamel Youssef Hassan, Pediatrician Consultant , Palestine - Gaza


E-mail: kyh10557@yahoo.com 128
MCQs PEDIATRICS

2) A 7 year old girl presents with a 3 day history of rash and ankle swelling. She
had a cold 4 weeks previously, but has otherwise been healthy. 39+1/40,
2.96kg. No neonatal problems. No drugs or medications. Fully immunized. On
examination she has palpable non-blanching purple spots 1-4 mm in diameter
especially over the shins and buttocks. Her left ankle is swollen, warm and
tender, with restricted movement. What is the most likely diagnosis?

1-Viral infection
2-Trauma/ child abuse
3-Streptococcal infection
4-Vasculitis eg HSP
5-Thrombocytopaenia
Comments:
The history is of preceding URTI followed by vasculitis on the shins and
buttocks, and ankle swelling. This is classical of Henoch-Schonlein purpura. The
pathological lesion is a vasculitis, hence the lesions are often palpable. In
contrast thrombocytopaenic purpura are not raised. The classical features are
rash, joint swelling, haematuria, and GI symptoms (vomiting, abdominal pain,
PR bleeding, occasional intussusception).

3) A 2 month old baby is admitted with seizures and failure to thrive. Follwoing a
seizure he is found to have a blood glucose concentration of 1.2 mmol/l. Other
biochemistry reveals a lactic acidosis and hyperlipidaemia. Examination reveals
hepatomegaly with palpable kidneys. What is the most likely diagnosis?
1-Gaucher's disease
2-McArdle's disease
3-Niemann-Pick disease
4-Pompe Disease
5-Von Gierke disease
Comments:
This baby has hypoglycaemia accounting for fits, associated with lactic acidosis
and hepatomegaly which would point to a diagnosis of von Gierke's disease.
This is a glycogen storage disease due to deficiency of glucose-6-phosphatase.
Glycogen is stored in both liver and kidney causing enlargement.
Both Gaucher and Niemann-Pick disease are lipid storage disorders.
McArdle's disease is a glycogen storage disease affecting muscle rather than
liver as is Pompe's disease. Neither would be expected to produce profound
hypoglycaemia.

Dr.Kamel Youssef Hassan, Pediatrician Consultant , Palestine - Gaza


E-mail: kyh10557@yahoo.com 129
MCQs PEDIATRICS

4) A 15 month old child with flexural eczema presents with a 3 day history of
cough and wheeze. There is a family history of hay fever. On examination his
saturations are 92% in air, with a respiratory rate of 55/min and moderate
recession. The heart rate is 150/min.

What is the most likely diagnosis?

1-Gastroesophageal reflux
2-Allergic rhinitis
3-Sinusitis
4-Asthma
5-Croup

Comments:
This child has virus-induced wheeze on an atopic background. He may well be an early
asthmatic, in which case recurrent episodes of cough and wheeze can be anticipated.

5) An overweight twelve year old boy attends A+E complaining of hip and knee
pain after a minor fall from his bike. On examination his knee appears normal
but there is restricted range of movement at the hip. The likely diagnosis is:

1-Chondromalaichae patella
2-Osteosarcoma
3-Perthe’s disease
4-Septic arthritis
5-Slipped upper femoral epiphysis

Comments:
Slipped upper femoral epyphesis is most common in obese, adolecent boys with a
positive family history. 25% are bilateral. Presentation may be with a coxalgic externally
rotated gait, decreased internal rotation, thigh atrophy and hip, thigh and knee pain.

Dr.Kamel Youssef Hassan, Pediatrician Consultant , Palestine - Gaza


E-mail: kyh10557@yahoo.com 130
MCQs PEDIATRICS

6) A 15 year old boy presents with a painful lower right arm after falling from his
motorbike. X-ray reveals a fracture of the distal radius. What is the name for this
fracture?
1-Bennett's fracture
2-Colle's fracture
3-Galeazzi' s fracture
4-Monteggia' s fracture
5-Pott's fracture

Comments:
Bennett's fracture is an intra-articular fracture of the base of the first metacarpal.
Galeazzi's fracture involves the radial shaft with dislocation of the distal
radioulnar joint.
Pott's fracture is a general term applied to fractures around the ankle.

7) A 12 day old girl presents with poor feeding, vomiting and lethargy. Born at
37+6/40, 2.98kg, no neonatal problems. Uncomplicated pregnancy and delivery.
No drugs or medications. No immunizations. No FH/SH of note.

On examination she is thin and wasted. She has occasional twitching of the
eyelids and mouth. She responds slightly to voice. Weight <3%, OFC 25%,
temperature is 36.5°C (tympanic), RR 40/min, HR 150/min. Chest clear, no
murmur. No organomegaly. No rash or dysmorphic features.

What is the most likely diagnosis?

1-Congenital adrenal hyperplasia


2-Congenital heart disease
3-Inborn errors of metabolism
4-Sepsis
Comments:
This child is dehydrated, vomiting and lethargic. Examination suggests possible subtle
fits. Although infection and a cardiac abnormality are possible, a metabolic condition
should be strongly suspected. Evaluation should include electrolytes, NH4, blood gases,
serum and urine aminoacids, and urine organic acids.

Dr.Kamel Youssef Hassan, Pediatrician Consultant , Palestine - Gaza


E-mail: kyh10557@yahoo.com 131
MCQs PEDIATRICS

8) 7 year old girl presents with high fever and severe left-sided throat pain. She
has had difficulty in swallowing over the last 2 days, and has been finding it
increasingly uncomfortable to open her mouth. Her voice is muffled and she
dribbles saliva. She was born at 41/40 gestation weighing 4.0kg and there were
no neonatal problems.

On examination she looks ill. Her temperature is 40.2°C (tympanic), RR 15/min


and HR 100/min. ENT examination shows left tonsillar enlargement and
exudate, with a uvula deviated to the right.

What is the most likely diagnosis?

1-Retropharyngeal abscess
2-Foreign body aspiration
3-Anaphylaxis
4-Croup
5-Peritonsillar abscess
Comments:
The history suggests a peritonsillar abscess (quinsy) on the left side, a complication of
Group A Strep. sore throat. A fever greater than 39.4°C is associated with severe
disease, and treatment is by surgical drainage.

9) Feeding difficulties in the neonatal period may be due to all of the following
except:

1-Prematurity.
2-Hiatus hernia.
3-Cerebral birth trauma.
4-Physiological jaundice
5-Congenital heart disease
Comments:
All of the options are associated with feeding difficulty except for physiological
jaundice which is benign, shortlived and not generally associated with
symptoms.

Dr.Kamel Youssef Hassan, Pediatrician Consultant , Palestine - Gaza


E-mail: kyh10557@yahoo.com 132
MCQs PEDIATRICS

10) Features of Down’s syndrome include all of the following except:


1-A third fontanelle.
2-Duodenal atresia.
3-Ataxic gait.
4-Increased incidence of hypothyroidism.
5-Susceptibility to acute leukaemia
Comments:
Cerebellar dysfunction is not a feature of Down’s syndrome, however an
Alzheimer’s like syndrome of memory loss and cognitive decline develops when
patients reach the mid thirties.

11) Which of the following statements regarding messenger RNA (mRNA) is


correct?

1-mRNA never contains introns.


2-mRNA is translated into proteins in the nucleus.
3-mRNA contains the bases cytosine and thymine.
4-Reverse transcriptase uses mRNA as a template to produce complementary
DNA.
5-mRNA is used in the Southern blotting technique.
Comments:
The structure of mRNA is similar to DNA except that uracil replaces thymine as
one of the bases. Both coding (exons) and non-coding regions of DNA are
initially transcribed into mRNA. Splicing is required for mature mRNA to be
produced only consisting of introns. Translation occurs in the cytoplasm.
Southern blotting is a technique that uses denatured fragments of DNA in a gel
to bind to DNA probes in order to detect the presence of particular genes or
sequences of DNA. The enzyme reverse transcriptase can be used by viruses to
insert viral mRNA into the host genome.

Dr.Kamel Youssef Hassan, Pediatrician Consultant , Palestine - Gaza


E-mail: kyh10557@yahoo.com 133
MCQs PEDIATRICS

12) Which of the following is a recognized treatment for complications of cystic


fibrosis?
1-DNAase to assist in reinflating collapsed lung segments.
2-Rectal pull-through and anastamosis for rectal prolapse.
3-Pancreatic transplant for diabetes mellitus.
4-Nebulised Tobramycin for pseudomonas colonization of the lower respiratory
tract.
5-Hypotonic saline drinks for hypernatraemic dehydration.
Comments:
Human recombinant DNAase given as a single daily aerosol seems to improve
pulmonary function, decrease the frequency of chest exacerbations, and promotes a
sense of well-being in patients with mild to moderate disease with purulent secretions.
This may be because, in the inflamed airway, the nuclei from dead cells accounts for
much of the viscidity of secretions. Rectal prolapse is usually idiopathic, occurring
between 1 and 5 years. Intestinal parasites, malnutrition, acute diarrhoea, ulcerative
colitis, pertussis, Ehler's Danlos Syndrome, meningocele, cystic fibrosis, and chronic
constipation can also predispose to it. Following defecation the prolapse usually resolves
spontaneously, or through manual reinsertion by the patient or parent. Nebulised
Tobramycin or Gentamicin may be given when airway pathogens are resistant to oral
antibiotics, or where infection is difficult to control at home. Hypernatramic dehydration
should be treated in the usual way.

13) Characteristic features of physiological jaundice include all of the following


except:

1-Onset after the first 24 hours of life.


2-Disappearance by the tenth day of life.
3-A serum bilirubin level below 250 mmol/L.
4-Normal feeding behaviour
5-Associated anaemia.
Comments:
Physiological jaundice occurs in 90% of infants and manifests after the first 2-3
days of life it is related to increased red cell breakdown and relative ‘immaturity’
of hepatic function. Anaemia if present would imply pathological cause of
jaundice eg haemolytic anaemia.

Dr.Kamel Youssef Hassan, Pediatrician Consultant , Palestine - Gaza


E-mail: kyh10557@yahoo.com 134
MCQs PEDIATRICS

14) A 3 year old by presents with a 2d history of coryza and cough, particularly
at night. On examination he has Harrison's sulci and is on the 3rd centile for
weight and height. Chest auscultation reveals scattered wheeze.

What is the most likely diagnosis?

1-Gastroesophageal reflux
2-Allergic rhinitis
3-Sinusitis
4-Asthma
5-Croup
Comments:
The examination findings suggest chronic undertreated asthma. If he does not respond
rapidly to asthma therapy a sweat test may be indicated to exclude CF. It is likely that he
will respond to regular inhaled low-dose steroids.

15) A 4 year old girl is referred with behavioural problems. Her speech is
generally well-formed. However, she finds it difficult to sustain conversations
because she talks obsessively about her own interests. She avoids eye contact.
She plays on her own for hours with her dolls, dressing and undressing them
repeatedly. Attempts to play with others results in tantrums ‫ﻧوﺑﺎت ﻏﺿب‬. Born at
39+6/40, 3.18kg no neonatal problems. No drugs, allergies. Fully immunized. No
FH/SH of note.

On examination she is on the 50% for height and weight. There are no specific
abnormalities to find.

What is the most likely diagnosis?


1-Speech delay, isolated
2-Pervasive developmental disorder
3-Global developmental delay
4-Impaired hearing
5-Autism
Comments:
This child has problems with reciprocal social interaction, with repetitive obsessive play.
Her speech is normal in form. These symptoms are characteristic of Asperger
syndrome. This is part of the spectrum of Pervasive Developmental Disorders, and is
regarded by some as 'high-functioning autism'. Such children may be regarded as
eccentric by their peers.

Dr.Kamel Youssef Hassan, Pediatrician Consultant , Palestine - Gaza


E-mail: kyh10557@yahoo.com 135
MCQs PEDIATRICS

16) A 9 year old boy presents with a history of headache and persistent green
nasal discharge. At night he has a cough and snores loudly. The headache is
exacerbated by leaning forwards.

On examination he is apyrexial, but has a persistent nasal obstruction and nasal


speech. He is tender over the maxillae and forehead.

What is the most likely diagnosis?

1-Gastroesophageal reflux
2-Allergic rhinitis
3-Sinusitis
4-Asthma
5-Croup
Comments:
The picture is one of upper airways obstruction associated with nasal discharge, most
likely due to sinusitis. In this case the maxillary and frontal sinuses are most likely to be
involved.

17) 15 year old boy is knocked out for 2 minutes while playing rugby. He
recovers rapidly, but cannot recall the incident. Apart from mild asthma requiring
beta-agonists before sport he is otherwise well.

On examination his temperature is 36.5°C (tympanic), RR 12/min and HR


65/min. His GCS is 15/15 and no focal neurological abnormalities are present.

What is the most likely diagnosis?

1-Child abuse
2-Head injury
3-Hypoglycaemia
4-Diabetic ketoacidosis
5-Substance abuse

Comments:
He has had a traumatic concussion, which refers to a diffuse brain injury. It is not
necessary to be knocked out to have concussion. The Colorado Medical Society
guidelines grade such injuries into 3 groups:

Dr.Kamel Youssef Hassan, Pediatrician Consultant , Palestine - Gaza


E-mail: kyh10557@yahoo.com 136
MCQs PEDIATRICS

1. Confusion alone
2. Confusion and amnesia
3. Confusion, amnesia and loss of consciousness

To avoid the Second Impact Syndrome return to sport should be allowed after
20 mins (group 1), 1 week (group 2) or 1 month (group 3) respectively.

18) A 17 year old student presented with recurrent attacks of dizziness. Which
one of the following additional features is most suggestive that she has an
anxiety disorder?

1-elevated diastolic blood pressure


2-nocturia
3-paraesthesiae in the hands
4-rotational vertigo
5-tinnitus
Comments:
Parasthesiae is often associated with anxiety of any description. Rotational
vertigo and tinnitus suggest an organic disorder, whilst anxiety disorder would
normally be associated with raised systolic blood pressure

19) A young boy is presents with aches and pains. Which of the following
features would argue against a diagnosis of osteomalacia?

1-Bow legs
2-raised alkaline phosphatase
3-Low serum phosphate level
4-Normal serum calcium
5-advanced bone age

Comments:
Clinical features include: bowing of tibia and fibula, craniotabes (soft skull),
thickening of forearm at the wrist, and of the costo chondral junction (rachitic
rosary). Dietary Vitamin D deficiency stimulates secondary hyperparathyroidism,
which causes increased renal excretion of phosphate, low serum phosphate,
and reduced urinary calcium excretion. Calcium concentrations may be low or
low normal. Raised alkaline phosphatase results from osteoblasts forming
unmineralised matrix. Short stature and delayed bone age are associated, rather
than advanced bone age.

Dr.Kamel Youssef Hassan, Pediatrician Consultant , Palestine - Gaza


E-mail: kyh10557@yahoo.com 137
MCQs PEDIATRICS

20) A 16 year old girl presents with a 2 day history of deteriorating


breathlessness and dyspnoea. Blood gas analysis shows a pH of 7.25, a PCO2
of 7.0kPa, a PO2 of 8.5kPa, and a base excess of -4. Which of the following
interpretations is correct?

1-Results are consistent with bronchopulmonary dysplasia.


2-Blood gases suggest type 1 respiratory failure.
3-Immediate intubation is required.
4-Results are consistent with late severe asthma.
5-Bicarbonate may be necessary to correct the acidosis.

Comments:
In interpreting blood gas results, the following sequence may be useful:

 Inspect the pH: Is it low, normal or high?


 Inspect the CO2: Is it low, normal or high?
 Inspect the PO2: Is it low, normal or high?

If the pH is low then an acidosis is present, and inspecting the CO2 will enable
you to determine whether this is due to respiratory or metabolic causes.
Inspecting the PO2 will tell you whether the patient is hypoxic or not. In this case,
the pH is reduced, and the CO2 is high, with a base deficit of only -4, insufficient
to explain the acidosis from metabolic causes. This is, therefore, a respiratory
acidosis, and the PO2 is also low suggesting type 2 respiratory failure. Possible
causes would include severe pneumonia, end stage asthma or neurogenic
causes such as guillain-Barre. In asthma, the initial stages show a low CO2, with
this climbing only to accompany failing respiration. The results would therefore
be consistent with late severe asthma. In bronchopulmonary dysplasia, there is
usually long-term CO2 retention with compensatory increase in bicarbonate
leading to a positive base excess and normal pH. Bicarbonate is usually only
considered if the base deficit exceeds about -8.

Dr.Kamel Youssef Hassan, Pediatrician Consultant , Palestine - Gaza


E-mail: kyh10557@yahoo.com 138
MCQs PEDIATRICS

21) An 18 month old boy because of maternal concerns about delayed speech.
He was born at 39+4/40 weighing 2.6kg and there were no neonatal problems.
He sat at 12/12, and now pulls to stand. He is able to make a tower of 2 inch-
high bricks. He is saying 2-3 single words which his mother can understand and
seems to hear well. He drinks from a bottle. He is fully immunized, on no
medications, and there is no FH/SH of note.

On examination he looks well. He is on the 25th centile for height and weight
and OFC. There are no specific findings of note.

What is the most likely diagnosis?

1-Speech delay, isolated


2-Pervasive developmental disorder
3-Global developmental delay
4-Impaired hearing
5-Autism
Comments:
The history suggests moderate global developmental delay. He is functioning at about
the 12 month level in gross motor, fine motor and vision, hearing and speech, and social
development. The history gives no indication of an underlying cause for the delay. A
detailed developmental assessment should be performed plus investigations for an
underlying cause. At minimum these should include TFTs and karyotype to rule out
Fragile X Syndrome. Other investigations may be indicated if a specific disorder is
suspected.

Dr.Kamel Youssef Hassan, Pediatrician Consultant , Palestine - Gaza


E-mail: kyh10557@yahoo.com 139
MCQs PEDIATRICS

22) A 15 month old girl presents with stridor and respiratory distress. She has
had a crusty nasal discharge for 2 days with low grade fever. She went to bed as
usual at 8pm, but awoke 2 hours later. Previous history was unremarkable.

On examination she has a fever of 37.8°C (tympanic), RR of 55/min (crying),


and HR of 150/min. She has marked stridor with moderate recession and a
barking cough. O2 saturations are 95% in air.

What is the most likely diagnosis?


1-Retropharyngeal abscess
2-Foreign body aspiration
3-Anaphylaxis
4-Croup
5-Peritonsillar abscess
Comments:
The history suggests a viral croup (laryngotracheobronchitis). Oral dexamethasone or
inhaled budesonide are effective in reducing symptoms, but are not required in mild
cases. Reassurance of the mother often calms down both child and mother.

23) A 14 year old girl presents with a history of cough and breathlessness on
exercise. She has seasonal rhinitis, and admits to have started smoking. Clinical
examination is unremarkable.

What is the most likely diagnosis?


1-Gastroesophageal reflux
2-Allergic rhinitis
3-Sinusitis
4-Asthma
5-Croup
Comments:
A typical history of asthma in later childhood, with exercise-induced symptoms and a
general deterioration on commencement of smoking. Unfortunately this is all too
common these days.

Dr.Kamel Youssef Hassan, Pediatrician Consultant , Palestine - Gaza


E-mail: kyh10557@yahoo.com 140
MCQs PEDIATRICS

24) A 10 year old boy attends A+E with a two day history of a limp. He has
recently been unwell. On examination he is apyrexial and movement at his hip is
uncomfortable.
1-Congenetal dislocation.
2-Perthes disease
3-Septic arthritis
4-Slipped upper femoral epiphesis
5-transient synovitis

Comments:
Transient synovitis is the most common cause of hip pain in childhood but other more
serious causes should be excluded before it is diagnosed. The cause is unknown but it
can be related to viral infection, allergic reaction or trauma.

25) A 9 week old infant is brought in unresponsive. Mother says she has well
apart from episodes of screaming every evening, which last several hours at a
time. She was born at 38+2/40 weighing 3.1kg and there were no neonatal
problems. Parents are unmarried teenagers.

On examination she has 3 small possible bruises on her face. She has a
temperature of 34.5°C, RR 30/min (irregular pattern), HR 130/min. She is floppy
with a full fontanelle. Fundoscopy shows flame-shaped haemorrhages. BM stix
is 3.9 mmol/l.

What is the most likely diagnosis?

1-Child abuse
2-Hepatic failure
3-Hypoglycaemia
4-Diabetic ketoacidosis
5-Substance abuse

Comments:
The history suggests a previously healthy child with colic of inexperienced parents. The
facial bruising suggests grip marks. Given the full fontanelle, comatose state of the
child, and flame-shaped haemorrhages, Shaken baby syndrome is most likely. It is
important to exclude infection with DIC or a coagulopathy.

Dr.Kamel Youssef Hassan, Pediatrician Consultant , Palestine - Gaza


E-mail: kyh10557@yahoo.com 141
MCQs PEDIATRICS

26) A 2 year old child presents with delayed language development. He was
born at term weighing 3.21kg and there were no neonatal problems. Gross
motor, vision and fine motor development are normal. Mother complains that he
'doesn't listen' especially when he is in another room. His speech is restricted to
single words, and only mother understands them.

On examination he is apyrexial, RR 16/min, HR 90/min. No dysmorphic features.


Chest is clear, there is no murmur nor organomegaly. ENT examination reveals
normal tonsils, clear nose and bilaterally dull drums.

What is the most likely diagnosis?


1-Speech delay, isolated
2-Pervasive developmental disorder
3-Global developmental delay
4-Impaired hearing
5-Autism
Comments:
The child has hearing impairment and isolated language delay. The examination
suggests bilateral 'glue ear' (Otitis media with effusion). This usually resolves with time,
though it may require tympanostomy tube insertion. These children often need Speech
Therapy input.

27) A 3 year old girl presents with a 5 day history of neck pain and fever, with
increasing difficulty in swallowing. Over the previous night she woke several
times with difficulty in breathing. She was born at 36+6/40 weighing 2.8kg and
there were no neonatal problems.

On examination her head held to the right. She has audible stridor when
agitated, and she has very tender glands in the neck. Her temperature is 38.6°C,
RR 20/min and HR 100/min. She has moderate neck stiffness.

What is the most likely diagnosis?

1-Retropharyngeal abscess
2-Foreign body aspiration
3-Anaphylaxis
4-Croup
5-Peritonsillar abscess

Dr.Kamel Youssef Hassan, Pediatrician Consultant , Palestine - Gaza


E-mail: kyh10557@yahoo.com 142
MCQs PEDIATRICS

Comments:
The history suggests a tonsillitis progressing to retropharyngeal abscess with increasing
upper airways obstruction. ENT examination in children with stridor should take place
where facilities are available for immediate intubation. In this case it revealed a
massively enlarged right tonsil with profuse exudate. This required surgical drainage and
removal with antibiotic cover.

28) A 2.5 year old boy is referred with behaviour problems and poor speech. He
speaks very seldom, and is unable to sustain a conversation. He always prefers
to play on his own, lining up his Thomas the Tank engine set repetitively,
ignoring his older and younger siblings. He throws tantrums when his routine is
disturned. He was born at 37+3/40 weighing 3.51kg and there were no neonatal
problems. Pregnancy had been uncomplicated.

On examination he is apyrexial, and there are no specific abnormalities to find.

What is the most likely diagnosis?


1-Speech delay, isolated
2-Pervasive developmental disorder
3-Global developmental delay
4-Impaired hearing
5-Autism
Comments:
The history is of poor speech development, ritualistic behaviour and lack of reciprocal
social interaction. The most likely diagnosis is Autism, which starts before 30 months of
age and is characterized by a qualitative impairment in verbal and nonverbal
communication, in imaginative activity, and in reciprocal social interactions. It is part of
the wider spectrum of pervasive developmental disorders. This includes: autism,
Aspergers, childhood disintegrative disorder, and Rett's syndrome.

Dr.Kamel Youssef Hassan, Pediatrician Consultant , Palestine - Gaza


E-mail: kyh10557@yahoo.com 143
MCQs PEDIATRICS

29) A 12 year old boy complains of persistent clear nasal discharge during the
spring and summer. He constantly rubs his nose and his eyes. Sometimes he
has an associated cough. The symptoms usually get better in the autumn. He
was diagnosed with asthma at 2 years, but this has improved with age. He rarely
requires beta agonist inhaler.

On examination he has a clear nasal discharge, nasal speech, and pink non-
purulent conjunctivae.

What is the most likely diagnosis?


1-Gastroesophageal reflux
2-Allergic rhinitis
3-Sinusitis
4-Asthma
5-Croup
Comments:
This atopic boy has classical symptoms of seasonal rhinitis. Although skin prick testing
may be helpful in resistant cases, it is usual to treat sufferers symptomatically with oral
antihistamines or topical steroids to eyes and nose.

30) 5 month old boy has had a mild coryza for 2 days. Mother hears him making
odd noises on the baby monitor. When she investigates she finds him floppy,
pale and not breathing. She stimulates him, attempts mouth-to-mouth
resuscitation and calls an ambulance. They give bag ventilation and he starts
breathing again. O2 is given on the way to hospital. Born at 32/40 gestation
weighing 1.7kg he required 3d ventilation for surfactant-deficient lung disease.
Since discharge he has been thriving.

On arrival at hospital he is self-ventilating in facemask oxygen, with saturations


of 94%. Temperature is 37.8°C, with RR 35/min, mild recession, and HR of
140/min. Scattered coarse crepitations are audible in both lung fields.

What is the most likely diagnosis?


1-Acute life-threatening event
2-Cardiac dysrhythmias
3-Seizures
4-Sudden infant death syndrome
Comments:

Dr.Kamel Youssef Hassan, Pediatrician Consultant , Palestine - Gaza


E-mail: kyh10557@yahoo.com 144
MCQs PEDIATRICS
This child has RTI followed by acute life-threatening event (ALTE). There are a large
number of potential causes. In this case bronchiolitis is most likely, given the
examination findings.

31) A 3 year old child presents with delayed speech. Although he seems to
understand full sentences and commands he says only occasional single words.
He was born at 41/40 weighing 3.4kg and there were no neonatal problems. He
is fully immunised, on no medications, and comes from a middle class articulate
family.

On examination he is on the 25th centile for height, weight and OFC. There are
no specific abnormalities to find. ENT examination is unremarkable.

What is the most likely diagnosis?

1-Speech delay, isolated


2-Pervasive developmental disorder
3-Global developmental delay
4-Impaired hearing
5-Autism
Comments:
The history is of isolated speech delay in a child who apparently hears normally.
Hearing should be formally checked, as it is the commonest reason for speech delay.In
this case the problem involves mainly expression rather than reception of speech. Such
speech problems may interfere significantly with schooling.

32) The parents of an eight year child divorce. The child lives with his mother
with his father visiting at weekends. On his father's visits, the child is sullen and
irritable but when his father leaves the boy cries, clings to his father and askes
his father to come back to live with his mother. What is the most appropriate
response that the father can make during these episodes?
1-Big boys don't cry
2-I left your mother but I am not leaving you.
3-I will see you next weekend
4-You take care of the house now I've left

Dr.Kamel Youssef Hassan, Pediatrician Consultant , Palestine - Gaza


E-mail: kyh10557@yahoo.com 145
MCQs PEDIATRICS

5-Sorry son but I just couldn't live with your mother


Comments:
The child's reactions typically represent a feeling of abandonment and it is
important that his father reaffirms that he is not leaving his son.

33) An infant is diagnosed with pyloric stenosis. Which of the following is true of
this diagnosis?

1-Typically presents between 6 and 12 months of age.


2-Is more common in females.
3-There is a strong familial predisposition.
4-Is due to hypertrophy of the longitudinal muscle layer of the pylorus.
5-Presents with bile-stained projectile vomiting.
Comments:
Infantile hypertrophic pyloric stenosis presents most commonly at approximately
6 weeks of age. It is more common in boys with a male: female ratio of 4:1.
There is a strong familial predisposition with 5% of affected infants having a
mother who also developed the problem. Pathologically, the pyloric 'tumour'
arises from hypertrophy the circular muscle layer. Patients present with
projectile vomiting that is not bile stained. Following resusciation and correction
of any electrolyte abnormality, surgical treatment is by pyloromyotomy.

34) A study has been designed to investigate whether a certain drug plus
physiotherapy treatment is better than drug treatment alone in the management
of rheumatoid arthritis. After randomizing the patients a small proportion of the
drug plus physiotherapy group decide to drop out of the study or omit some
treatment sessions specified in the research protocol. What is the correct way of
analysing the subsequent data?

1-Assume the patients have withdrawn their consent


2-Exclude these patients from all analysis
3-Extend the trial recruitment to make up the numbers
4-Include these patient outcomes in the drug plus physiotherapy group
5-Interview the patients and report their group separately

Dr.Kamel Youssef Hassan, Pediatrician Consultant , Palestine - Gaza


E-mail: kyh10557@yahoo.com 146
MCQs PEDIATRICS

Comments:
This is the principle of 'intention to treat'. It is possible that the physiotherapy
intervention was harmful to the patients and this is why they left. Intention to
treat helps to reduce bias by sticking to the original allocation of treatment and
analyzing the patient in that treatment group even (and concentrate for this bit)
even if they don''t get it!

35) A 17 year old female presents with acute breathlessness. She has had
asthma for approximately 3 years and recently commenced new therapy. Which
agent may be responsible for this exacerbation?
1-Salmeterol
2-Theophylline
3-Beclomethasone
4-Ipratropium bromide
5-Monteleukast
Comments:
Salmeterol has been reported to produce an acute exacerbation of asthma,
possibly through an acute hypersensitivity reaction.

36) A 17 year old male is brought to clinic as his parents are concerned about
changes in his behaviour. Which of the following suggest a diagnosis of
Schizophrenia?

1-incongruity of affect ‫ﻣﺷﺎﻋر ﻣﺗﺿﺎرﺑﺔ – ﺗﺿﺎرب اﻟﻣﺷﺎﻋر‬


2-auditory hallucinations with clouding of consciousness
3-memory impairment
4-feelings of panic in buses and shops
5-grandiose ideations

Comments:
Incongruity of affect is emotion inappropriate to circumstances. There may be
intellectual defects from prolonged institutionization ‫ اﻹﻗﺎﻣﺔ ﺑﻣؤﺳﺳﺎت ذوى اﻟﮫﺎھﺎت‬or
treatment rather than the illness itself. Auditory hallucinations with clouding of
conciousness suggest delirium and memory impairment possible organic brain
disease. The panic in crowds suggests an anxiety disorder and grandiose
ideations suggest hypomania

Dr.Kamel Youssef Hassan, Pediatrician Consultant , Palestine - Gaza


E-mail: kyh10557@yahoo.com 147
MCQs PEDIATRICS

37) An 18 year old female is reluctant to eat food that is prepared for her. Which
one of the following would be most consistent with a diagnosis of anorexia
nervosa?
1-she believes the food is poisoned
2-she has a full-time job
3-she has bouts of heavy drinking
4-she regards herself as ill
5-she secretly abuses anabolic steroids

Comments:
Anorexia nervosa is associated with the abnormal perception of body image.
They generally feel well despite the protestations of others who feel that they
look awful. They exercise avidly and until the very late stages of the disease
hold down full time jobs. There is no delusion with regard to the food being
poisoned. Heavy drinking associated with food refusal would suggest alcoholism
and alcoholic gastritis. The secretive abuse of laxatives would fit with the
diagnosis rather than anabolic agents.

38) Which of the following organelles contains enzymes responsible for the
digestion of constituents of cells and tissues?
1-endoplasmic reticulum
2-Gogli apparatus
3-lysosomes
4-microtubules
5-mitochondria
Comments:
The lysosomes contain the enzymes and molecules such as oxidases, free radical etc
responsible for the breakdown of intracellular components. Micortubules are involved in
mitotic processes and intracellular transportation. The mitochondria produce energy for
cellular functions.

Dr.Kamel Youssef Hassan, Pediatrician Consultant , Palestine - Gaza


E-mail: kyh10557@yahoo.com 148
MCQs PEDIATRICS

39) Which one of the following is true regarding protein metabolism?


1-There are 5 essential amino acids.
2-The absence of any of the essential amino acids results in a negative nitrogen
balance
3-Protein constitutes about 30-45% of total daily calorie intake in infants and
children
4-Essential amino acids can be synthesized in the liver and kidneys
5-Human milk as the only food provides an insufficient source of protein for a 1
year old healthy infant

Comments:
There are 9 essential amino acids; all are required to maintain nitrogen balance.
Carbohydrates constitute 30-45% of total caloric intake in children. Essential
amino acids cannot be synthesized and must be obtained from the diet. Human
milk contains the proteins casein, lactoglobulin and lactalbumin, which provide a
sufficient source of protein.

40) Apoptosis is the process of programmed cell death and occurs in cells that
have damaged DNA. A mediator of this process is a tumour suppressor gene
that inhibits mitosis and promotes apoptosis. This gene is:-

1-bcl-2
2-caspases
3-fas (CD95)
4-p53
5-ras

Comments:
bcl-2 is an inhibitor of apoptosis. fas is a cell receptor and caspases are present
in all cells both promote apoptosis but are not tumour suppressor genes. ras is
an oncogene.

Dr.Kamel Youssef Hassan, Pediatrician Consultant , Palestine - Gaza


E-mail: kyh10557@yahoo.com 149
MCQs PEDIATRICS

41)Which one of the following conditions is transmitted by means of an X-linked


recessive inheritance :
1-Cystic fibrosis
2-Galactosaemia
3-Huntington’s chorea
4-Duchenne muscular dystrophy
5-Facio-humero-scapular dystrophy
Comments:
Cystic fibrosis and Galactossaemia are autosomal recessively inherited
conditions. Haemophilia and Duchenne muscular dystrophy are X- linked
recessive and Facio-humero-scapular dystrophy and Huntington’s chorea are
inherited in an autosomal dominant manner.

42)A 10 year old child is found to have neurofibromatosis. Which of the following
statements correctly applies:
1-She will most probably be mentally retarded
2-The finding of 2 café au lait spots in her 6 year old sibling suggests he may be
affected
3-Her condition was inherited in an autosomal recessive fashion
4-Freckling of the back would be expected
5-There may be no family history of the condition
Comments:
Mental retardation may be a clinical feature of the condition but is not a usual
clinical feature. Six or more café au lait spots before the age of puberty would
assist in making a diagnosis, along with other clinical findings such as Lisch
nodules, and optic nerve gliomas. The condition is autosomal dominant.
Freckling of the inguinal and axillary regions occur. 30-50% of new cases are
due to spontaneous mutations.

Dr.Kamel Youssef Hassan, Pediatrician Consultant , Palestine - Gaza


E-mail: kyh10557@yahoo.com 150
MCQs PEDIATRICS

43)Which of the following statements applies to fragile X syndrome?

1-Mental retardation is found in all males


2-Mental retardation is not found in females
3-Cytogenetic finding of fragile-X is induced in a vitamin B12 deficiency culture
medium
4-Affected children are taller than average
5-The condition can only be diagnosed after birth.
Comments:
Affected males are usually but not always mentally retarded .One third of
females with the mutation will be mentally retarded. The diagnosis of fragile X
syndrome was originally based on the expression of a folate- sensitive fragile X
site (X q27.3) induced in cell culture under conditions of folate deprivation.
Affected children are taller and may have high arched palate, long ears, a long
face and macro orchidism. Diagnosis can be made by detection of the mutant
FMR 1 gene by chorionic villus sampling , confirmatory amniocentesis may be
required in some cases.

Dr.Kamel Youssef Hassan, Pediatrician Consultant , Palestine - Gaza


E-mail: kyh10557@yahoo.com 151
MCQs PEDIATRICS

1- A 6-month-old previously healthy infant presents with a 2-day history of cough and
difficulty breathing. On physical examination, she appears lethargic, pale, and poorly
perfused. She exhibits marked tachypnea, and auscultation of the chest reveals decreased
breath sounds bilaterally and poor aeration. She has supraclavicular and intercostal
retractions.
Of the following, the MOST appropriate therapy for this infant is
A. continuous positive airway pressure via face mask
B. endotracheal administration of surfactant
C. endotracheal intubation and positive pressure ventilation
D. negative pressure ventilation
E. nitric oxide

2- A previously healthy 12-year-old girl presents to the emergency department with


sudden onset of wheezing after running through the woods. Findings on physical
examination are normal except for wheezing only in the mid-right lung field.
Of the following, the MOST likely diagnosis is
A. bacterial pneumonia
B. exercise-induced asthma
C. inhaled foreign body
D. poorly controlled asthma
E. vascular ring

3- During the health supervision visit for a 5-year-old child, his father states that the boy
is "always coughing." His mother dismisses this, stating that her son "keeps a cold."
Further questioning reveals that the child coughs when he has an upper respiratory tract
infection, during exercise, upon exposure to the neighbor's cat, and almost every night.
Results of the physical examination are normal.
Of the following, the MOST likely diagnosis for this child is
A. asthma
B. chronic sinusitis
C. cystic fibrosis
D. no abnormality
E. postnasal drip

Dr.Kamel Youssef Hassan, Pediatrician Consultant , Palestine - Gaza


E-mail: kyh10557@yahoo.com 152
MCQs PEDIATRICS
4- A 7-year-old girl coughs frequently during a health supervision visit. On further
questioning, her mother reports that over the past several years the child has coughed at
night, during exercise, and whenever she has an upper respiratory tract infection. A
thorough history and physical examination is otherwise unrevealing.
In addition to chest radiography and a sweat test, the MOST appropriate initial
diagnostic study to determine the etiology of this child's cough is
A. allergy testing
B. barium swallow
C. bronchoscopy
D. pulmonary function testing
E. skin testing for tuberculosis

5- A 9-year-old girl presents to the emergency depart-ment with an exacerbation of her


asthma. She felt warm to touch at home, but is afebrile. Physical examination reveals
mild respiratory distress and diffuse expiratory wheezing. Chest radiography shows a
consolidation between the fissures on the right side with volume loss.
Of the following, the MOST likely interpretation of this radiographic finding would be
a(n)
A. atelectasis of the right middle lobe
B. hyperinflated chest without infiltrate
C. pleural effusion
D. pneumothorax
E. right middle lobe pneumonia

6- The father of a 12-year-old girl is concerned about her recent lack of interest in
physical activity. She says that it is "just too hard, and I get out of breath." Findings
from a complete history, physical examination, and chest radiography are normal. An
exercise test is terminated when fatigue and breathlessness are induced, but the results of
pulmonary function testing remained normal throughout the test.
Of the following, The MOST likely explanation for these symptoms is
A. asthma
B. cardiac disease
C. depression
D. muscle weakness
E. sarcoidosis

Dr.Kamel Youssef Hassan, Pediatrician Consultant , Palestine - Gaza


E-mail: kyh10557@yahoo.com 153
MCQs PEDIATRICS
7- A 4-month-old previously healthy infant presents in January with a 2-day history of
coughing and breathing difficulty and a 1-day history of refusing to take oral fluids. He
has no history of choking. Physical examination reveals a pale infant who has dry
mucous membranes, retractions, a respiratory rate of 70 breaths/min, and breath sounds
decreased bilaterally with diffuse expiratory wheezing.
Of the following, the BEST management is
A. administration of intramuscular ceftriaxone
B. administration of nebulized albuterol at home
C. administration of oral albuterol
D. hospital admission for cardiorespiratory monitoring

8- A 12-year-old girl presents with signs and symptoms suggestive of asthma. You order
pulmonary function studies.
Of the following, spirometry is MOST useful to measure
A. airway resistance
B. expiratory flow rate
C. functional residual capacity
D. intrapulmonary shunting
E. static compliance

9- A 2-year-old child who has asthma that is well controlled with home nebulizer
therapy suddenly develops coughing after playing with a small plastic toy. Physical
examination reveals unlabored breathing, with an intermittent, nonproductive cough and
diffuse expiratory wheezing that is greater on the right side.
Of the following, the appropriate NEXT step in determining the cause of this patient's
symptoms is to
A. administer a nebulized bronchodilator
B. arrange for a pH probe study
C. initiate a trial of oral antibiotics
D. obtain inspiratory and expiratory chest radiographs
E. perform pulmonary function spirometry

Dr.Kamel Youssef Hassan, Pediatrician Consultant , Palestine - Gaza


E-mail: kyh10557@yahoo.com 154
MCQs PEDIATRICS
10- You are asked to assess a hospitalized 3-year-old girl who has asthma and
respiratory distress without cyanosis while receiving 3 L/min of oxygen by nasal
cannula. Blood pressure and hemoglobin concentration are normal.
Of the following, the MOST important factor affecting your decision whether to intubate
the child for assisted ventilation is
A. arterial PO2
B. heart rate
C. magnitude of pulsus paradoxus
D. respiratory effort
E. respiratory rate

11- A 7-year-old child has had persistent nighttime cough for 1 year. Administration of
over-the-counter cough syrups has not been helpful.
Among the following, the MOST likely cause of the cough is
A. foreign body
B. hyperresponsive airway disease
C. immotile cilia syndrome
D. psychogenic cough
E. tuberculosis

Dr.Kamel Youssef Hassan, Pediatrician Consultant , Palestine - Gaza


E-mail: kyh10557@yahoo.com 155

You might also like